■ちょっとした疑問や質問はここに書いてね113■

このエントリーをはてなブックマークに追加
1ご冗談でしょう?名無しさん
前スレ
■ちょっとした疑問や質問はここに書いてね112■
http://science6.2ch.net/test/read.cgi/sci/1243866126/

質問する前に
教科書や参考書をよく読もう
http://www.google.com/ などの検索サイトを利用し、各自で調べること
宿題を聞くときは、どこまでやってみてどこが分からないのかを書くこと。
丸投げはダメだからね
(丸投げ君は完全無視。答えるだけ無駄。)

質問に対する返答には、何かしらの返答ちょうだいね
★書き込む前に>>2の注意事項を読んでね
★数式の書き方(参考)はこちら>>3
(予備リンク:>>2-10
荒らし厳禁、煽りは黙殺、忘れないうちに定期age
単発質問スレを発見したらこのスレッドへの誘導をよろしくね。

定番FAQ
英語最新版
http://math.ucr.edu/home/baez/physics/
旧版日本語訳
http://research.kek.jp/people/morita/phys-faq/
他にも参考にあるサイトなどあればぜひ。
例)http://ja.wikipedia.org/
※wikipedia内の説明はすべてが正確なわけではありません。
このスレでの受け答えもそうですが。相互に補完しつつ精度を高めましょう。
2ご冗談でしょう?名無しさん:2009/06/22(月) 20:42:35 ID:??? BE:255612839-DIA(281444)
書き込む際の注意

1.)
板の性格上、UNIX端末からの閲覧も多いと推察されます。
機種依存文字(ローマ数字、丸数字等)は避けて下さい。

2.)
以下のような質問に物理板住人は飽き飽きしているので、たぶん無視されます。
しないで下さい。
「相対性理論は間違っています」「量子力学は間違っています」
「宇宙論は間違っています」「シュレディンガーの猫は変です」
「永久機関を作りました」「タイムマシンについて教えて」
「どうして〜?」関連(→「どのようにして〜?」と質問すること)
「なぜ〜?」関連(たいてい、物理学の対象ではないため)

「どうして・なぜ」:http://academy6.2ch.net/philo/
(哲学板・雑談板のほうがむいている場合が多いです。)
雑談は雑談スレで:http://science6.2ch.net/test/read.cgi/sci/1181386663/

3.)
宿題を聞くときは、どこまでやってみてどこが分からないのかを書くこと。
丸投げはダメよ。丸投げに答えるのもダメよ。 丸投げを相手にする人はお馬鹿さん。
せめてポインタを示す程度に留めましょう。

4.)
厨房を放置できない奴も厨房
3ご冗談でしょう?名無しさん:2009/06/22(月) 20:42:58 ID:??? BE:530158278-DIA(281444)
数式の書き方(参考)
●スカラー:a,b,...,z, A,...,Z, α,β,...,ω, Α,Β,...,Ω,...(「ぎりしゃ」「あるふぁ〜おめが」で変換)
●ベクトル:V=[v1,v2,...], |V>,V↑,vector(V) (混同しないならスカラーの記号でいい。通常は縦ベクトル)
●テンソル:T^[i,j,k...]_[p,q,r,...], T[i,j,k,...;p,q,r,...]  (上下付き1成分表示)
●行列  M[i,j], I[i,j]=δ_[i,j]  M=[[M[1,1],M[2,1],...],[M[1,2],M[2,2],...],...], I=[[1,0,0,...],[0,1,0,...],...]
(右は全成分表示。行または列ごとに表示する。例:M=[[1,-1],[3,2]])
●転置行列・随伴行列:M ',tM, M†("†"は「きごう」で変換可) ●行列式・トレース:|A|=det(A), tr(A)
●複号:a±b("±"は「きごう」で変換可)
●内積・外積・3重積:a・b, a×b, a・(b×c)=(a×b)・c=det([a,b,c]), a×(b×c)
●関数・数列:f(x), f[x] a(n), a[n], a_n
●平方根:√(a+b)=(a+b)^(1/2)=sqrt(a+b) ("√"は「るーと」で変換可)
●指数関数・対数関数:exp(x+y)=e^(x+y) ln(x/2)=log[e](x/2)(exp(x)はeのx乗、lnは自然対数)
●三角比:sin(a), cos(x+y), tan(x/2)
●絶対値:|x|  ●共役複素数:z~ ●ガウス記号:[x] (関数の変数表示と混同しないよう注意)
●階乗:n!=n*(n-1)*(n-2)*...*2*1, n!!=n*(n-2)*(n-4)*...
●順列・組合せ:P[n,k]=nPk, C[n,k]=nCk, Π[n,k]=nΠk, H[n,k]=nHk ("Π"は「ぱい」で変換可)
4ご冗談でしょう?名無しさん:2009/06/22(月) 20:43:22 ID:??? BE:37868922-DIA(281444)
数式の書き方続き(参考)
●微分・偏微分:dy/dx=y', ∂y/∂x=y_x ("∂"は「きごう」で変換可)
●ベクトル微分:∇f=grad(f), ∇・A=div(A),∇xA=rot(A), (∇^2)f=Δf ("∇"は「きごう」,"Δ"は「でるた」で変換可.)
●積分:∫[0,1]f(x)dx=F(x)|_[x=0,1], ∫[y=0,x]f(x,y)dy, ∬[D]f(x,y)dxdy, ∬[C]f(r)dl ("∫"は「いんてぐらる」,"∬"は「きご

う」で変換可)
●数列和・数列積:Σ[k=1,n]a(k), Π[k=1,n]a(k) ("Σ"は「しぐま」,"Π"は「ぱい」で変換可)
●極限:lim[x→∞]f(x) ("∞"は「むげんだい」で変換可)
●図形:"△"は「さんかく」 "∠"は「かく」 "⊥"は「すいちょく」 "≡"は「ごうどう」 "∽"は「きごう」
●論理・集合:"⇔⇒∀∃∧∨¬∈∋⊆⊇⊂⊃∪∩"は「きごう」で変換
●等号・不等号:"≠≒<>≦≧≪≫"は「きごう」で変換
5ご冗談でしょう?名無しさん:2009/06/23(火) 00:07:36 ID:1lM6UYSL
1さんありがと&お疲れさま
窓の外に雨音が響いています
6ご冗談でしょう?名無しさん:2009/06/23(火) 00:09:55 ID:???
オーロラと流星ってどっちの方が高度が高い?
7ご冗談でしょう?名無しさん:2009/06/23(火) 00:13:37 ID:???
バネ定数1N/mのバネに質量1kgの重りをぶら下げて、微笑変位0.05mを与えた時の変位xの式ってどうなりますか?
8ご冗談でしょう?名無しさん:2009/06/23(火) 00:15:20 ID:???
>>7
>>2の3)を読め
9ご冗談でしょう?名無しさん:2009/06/23(火) 00:34:16 ID:???
994 +1:ご冗談でしょう?名無しさん :sage:2009/06/22(月) 23:15:14
>989
ボソンのスピンは0か1か2だけど
10ご冗談でしょう?名無しさん:2009/06/23(火) 00:47:40 ID:xVY79hOM
>>9

957 名前:ご冗談でしょう?名無しさん[[email protected]] 投稿日:2009/06/22(月) 04:33:15 ID:???
 宇宙船内の気圧をA、宇宙宙船内の気温D、外気の気温E、外気の気圧F

 {(A×D)+{(F−A)×E}÷F=宇宙船内に外気を注入した時の宇宙船の室内気温

 よって
 宇宙船内の気圧を1atm、宇宙宙船内の気温100K、外気の気温200K、外気の気圧0.01atm
 のとき

 宇宙船内に外気を注入した時の宇宙船の室内気温
 = ((1*100+(0.01-1)*200)/0.01 = -9800 K

 これで絶対零度など存在しないことがお分かりになっただろう
11897:2009/06/23(火) 06:25:14 ID:???
>>990
> >>989
> > ただ、この考えだと、フェルミオンとボソンがどう種類存在する
> > 超対称性ではゼロ点エネルギーを打ち消せません。

> kwsk
スピン1/2は+1/2,-1/2の2種類
スピン1は+1,0,-1の3種類と、ボソンとフェルミオンの自由度が異なったら
たとえ同種類存在しても打ち消しあうことができないでしょう?

>>994
>ボソンのスピンは0か1か2だけど、一種類の粒子はそのどれかしか持たないだろ
>自由度とは言わないな

ということはスピン1/2は+1/2,-1/2の2種類とスピン1は+1,0,-1の3種類は
ゼロ点エネルギーの種類には反映しないってことですか?
12ご冗談でしょう?名無しさん:2009/06/23(火) 08:37:00 ID:???
>>11
普通、複素でスピン1/2のフェルミオン1種(1/2,-1/2の2モード)と、
masslessのスピン1の粒子(-1,1の2モード)がモードの数が同じになって、
零点エネルギーが消し合う。
massiveなスピン1と、複素スピン1/2だと消し合わない。


13ご冗談でしょう?名無しさん:2009/06/23(火) 09:33:40 ID:???
■ちょっとした疑悶や膣悶はめこすじに掻いてね69■
14ご冗談でしょう?名無しさん:2009/06/23(火) 09:43:23 ID:???
>>10正しくは{(A×D)+(F−A)×E}÷F=宇宙船に外気を注入を注入した時の温度
のまちがいでした。
15ご冗談でしょう?名無しさん:2009/06/23(火) 11:00:20 ID:CLv4v5q5
馬鹿野郎!!参考書読め。
16ご冗談でしょう?名無しさん:2009/06/23(火) 11:12:26 ID:cZJJ+lNx
粒子(とか)の固有時は量子論ではどう(いう式で)表されるのですか?
17ご冗談でしょう?名無しさん:2009/06/23(火) 12:19:07 ID:3O8xQEjK
特定の日時(秒まで)の特定の場所(緯度経度高度) の地球磁場の強さと偏角を求めたいのですが、教授にキーワードは「IGRF」と言われたんですが調べでもIGRFがなにを意味しているのかさっぱり分かりません。どなたかご教授願います。
18ご冗談でしょう?名無しさん:2009/06/23(火) 12:26:43 ID:???
>>10正しくは{(A×D)+(F−A)×E}÷F=宇宙船に外気を注入を注入した時の温度
のまちがいでした。

宇宙船内の気圧A 宇宙船内の気温D  外気の気温E 外気の気圧F
{(A×D)+(F−A)×E}÷F=宇宙船に外気を注入した時の宇宙船の船内気温

よって
宇宙船内の気圧を1atm、宇宙宙船内の気温100K、外気の気温200K、外気の気圧0.01atm
のとき

{(1×100)+(0.01−1)×200}÷0.01=宇宙船に外気を注入した時の宇宙船の船内気温=ー9800

これで絶対零度など存在しないことがお分かりになっただろう
19ご冗談でしょう?名無しさん:2009/06/23(火) 12:29:18 ID:???
>>17
釣ってくれてありがとう。
これは面白いな。

http://swdcwww.kugi.kyoto-u.ac.jp/igrf/point/index-j.html

些末ながら、一行が長いので適宜改行を入れた方がいいぞ。
20ご冗談でしょう?名無しさん:2009/06/23(火) 12:45:28 ID:3O8xQEjK
それは見つけたんですが、それの計算方法を自分でプログラムしてこいっていうんですよ。
つまり「年月日時分秒」を入力して「磁場強度と偏角」を出力するプログラムを自分で制作しろと言うんです。
で、IGRFの意味が分からなくて詰んでる状態ですorz
21ご冗談でしょう?名無しさん:2009/06/23(火) 12:49:42 ID:???
>>20 なら、下記に Cと FORTRANがおいてあるよ。

http://www.ngdc.noaa.gov/IAGA/vmod/igrf.html
22ご冗談でしょう?名無しさん:2009/06/23(火) 13:05:09 ID:???
それもダウンロードしてみたんですが、自分が使っているプログラム言語はC++で
しかもまだ勉強始めたばかりなので肝心のやり方については全く分かりませんでした…
IGRFをどのように使ってどのように計算するのかが今の問題です。
23ご冗談でしょう?名無しさん:2009/06/23(火) 13:07:36 ID:???
じゃあお前プログラム板の初心者スレ行けよ!
つーかCとC++なんて違い殆ど無いだろ!
24ご冗談でしょう?名無しさん:2009/06/23(火) 13:10:24 ID:???
ぷっ、解説もできないくせに、文句だけ。
25ご冗談でしょう?名無しさん:2009/06/23(火) 13:27:17 ID:???
教えてクン養成マニュアルのお手本のような香具師だな
26ご冗談でしょう?名無しさん:2009/06/23(火) 13:46:28 ID:???
>>11
> 949 名前:ご冗談でしょう?名無しさん メェル:sage 投稿日:2009/06/22(月) 00:58:32 ID:???
> >>897
> > 多くの論文には真空はボソニック真空であり、
> > +∞の発散問題があると書かれています
> 例えば?
早く例出せよ。多くの論文とかいうのはでたらめか?
27ご冗談でしょう?名無しさん:2009/06/23(火) 13:59:50 ID:QoGZ+L31
半径aの薄い円盤の端に長さ2aに拘束した軽いバネをつけ角速度ω1で回転させる。その後、拘束を解除したところバネは伸び縮みをしたのち、長さ3b、角速度ω2で回転するようになった。

@ω1とω2の比を求めよ。A失われた力学的エネルギーを求めよ。

という問題で、Aが複雑で分かりません。やり方は

@始めと後の状態の慣性モーメントをそれぞれI1、I2とすると、
I1=(1/2)Ma^2+M(2a+a)^2
=(19/2)Ma^2
I2=(1/2)Ma^2+M(3a+a)^2
=(33/2)Ma^2
また角運動量保存則より
I1・ω1=I2・ω2
∴(ω2/ω1)=(I1/I2)
=19/33

A始めと後の円盤の重心の速さそれぞれV1、V2、エネルギーの減少量をΔUとすると、
ΔU=(1/2){MV2^2+I2・ω2^2}−(1/2){MV1^2+I1・ω1^2}

というやり方でよろしいでしょうか?
28ご冗談でしょう?名無しさん:2009/06/23(火) 14:13:01 ID:QoGZ+L31
>>27問題文訂正
長さ3b ×
長さ3a 〇

すみません。
29ご冗談でしょう?名無しさん:2009/06/23(火) 14:56:37 ID:???
>>27
バネがどこにどう憑いてるのかさっぱりわからないんだが?
30ご冗談でしょう?名無しさん:2009/06/23(火) 15:11:59 ID:???
大方半径aの円盤の外周じゃね?
31ご冗談でしょう?名無しさん:2009/06/23(火) 15:18:20 ID:???
おまえの見解とか心の底からどうでもいい
32ご冗談でしょう?名無しさん:2009/06/23(火) 15:24:21 ID:QoGZ+L31
>>29円盤の端です

―――〇

というような感じです!
33ご冗談でしょう?名無しさん:2009/06/23(火) 15:25:24 ID:???
>>29
オカルト板ででも訊いてこい
34ご冗談でしょう?名無しさん:2009/06/23(火) 15:31:20 ID:+jB9vv8H
X線は電場の影響を受けますか?
どなたか教えてください
35ご冗談でしょう?名無しさん:2009/06/23(火) 16:29:07 ID:???
レート方程式について詳しく書いてある論文でお勧めってありますか?
できればレート方程式に焦点を当てた論文がいいです
スレチだったらすいません
36ご冗談でしょう?名無しさん:2009/06/23(火) 16:37:03 ID:???
X線に限らず電磁波は電荷を持たないから、電場の影響も磁界の影響も受けないよ
37ご冗談でしょう?名無しさん:2009/06/23(火) 16:51:24 ID:???
カー効果は?
38ご冗談でしょう?名無しさん:2009/06/23(火) 16:53:33 ID:+jB9vv8H
>>36
ありがとうございます。

1.X線は電磁波である
2.紫外線より長波長にX線がある
3.ガンマ線は電磁波である
4.X線は電場の影響をうける

以上の選択肢で正しいものの組み合わせを選べという問題で悩んでいます。
A.134 B.12 C.23 D.4 E.1234
39ご冗談でしょう?名無しさん:2009/06/23(火) 18:12:51 ID:???
>>38
入試に出たら全員正解扱いってやつだな
40ご冗談でしょう?名無しさん:2009/06/23(火) 18:14:17 ID:???
>>38
丸投げ禁止だよ
でも先生ブン殴ってもいいよ
41ご冗談でしょう?名無しさん:2009/06/23(火) 18:42:31 ID:???
>>21
ごめん英語出来ないんだが何それコンパイラ?
42ご冗談でしょう?名無しさん:2009/06/23(火) 18:46:57 ID:+jB9vv8H
>>39ー40
その反応で安心しました、どうもありがとうございます。
43ご冗談でしょう?名無しさん:2009/06/23(火) 19:11:13 ID:???
>>20
プログラミングの知識はどの位有るの?
44ご冗談でしょう?名無しさん:2009/06/23(火) 19:39:15 ID:???
>>38
その問題出した「先生」って高校の物理の教員?
それとなく出身大学とか聞き出してみてよ。
45ご冗談でしょう?名無しさん:2009/06/23(火) 19:42:46 ID:???
>27
力学的エネルギーの変化ってことはバネが減衰振動して熱エネルギーとかに変わることまで考えるということ?
熱エネルギーに変わるとしたら、角運動量保存するんだっけ?
ところでMって何?
46897:2009/06/23(火) 19:45:26 ID:???
>>12
>masslessのスピン1の粒子(-1,1の2モード)がモードの数が同じになって

スピン1の0(スカラーフォトンまたは縦波フォトンは)は
計算に入れなくっていいっていうことですか??

>>26
>早く例出せよ。多くの論文とかいうのはでたらめか?

なんで怒ってるのかは全く不明だが、とりあえずDEとゼロ点エナジー
を絡ませて論じているものはプラスの発散問題をどうにかしようとしている。
あとゼルドヴィッチだっけ?induced重力っていうアインシュタイン方程式に
ゼロ点エナジーをカップリングさせた理論も正の発散問題の
解決の提案なんだな。
47ご冗談でしょう?名無しさん:2009/06/23(火) 20:35:08 ID:???
顔はマンゲツ☆ さん、mixiに帰りなよ。
ここは茶化して楽しんでる奴ばかりだから。
http://mixi.jp/view_bbs.pl?page=13&comm_id=63364&id=42163151
48ご冗談でしょう?名無しさん:2009/06/23(火) 20:35:12 ID:???
>>46
arXivの番号か何か一つ挙げてもらえる? >+∞の発散問題
49サル:2009/06/23(火) 20:39:32 ID:yZStCEtD
二つの未知定数に対して二つの境界条件があってどうして波動性が成立するんだ?
50ご冗談でしょう?名無しさん:2009/06/23(火) 21:11:34 ID:QoGZ+L31
>>45熱エネルギーか分かりませんが、減少を求めればいいみたいです!Mは円板の質量です!
51ご冗談でしょう?名無しさん:2009/06/23(火) 21:19:32 ID:+jB9vv8H
>>44
僕一応国立医学部の学生です。
なのでこの問題を作った先生も一応医者なんだとは思います。
授業には出てなくて、どの先生が講義した内容かわからないので出身を聞くことはできないです。
52ご冗談でしょう?名無しさん:2009/06/23(火) 21:20:50 ID:???
放物の問題?
53ご冗談でしょう?名無しさん:2009/06/23(火) 21:37:58 ID:???
>27
減衰も内力だけから生じるだろうから、角運動量は保存するか、、、。

でもバネの伸びは一様じゃないはず。慣性モーメントそんなに簡単か?
そもそもMが円盤の質量ならバネのは?
54ご冗談でしょう?名無しさん:2009/06/23(火) 21:41:40 ID:???
「軽いバネ」は質量無視の符丁でしょ
55ご冗談でしょう?名無しさん:2009/06/23(火) 22:00:57 ID:QoGZ+L31
>>53慣性モーメントは円盤の中心の慣性モーメント(1/2)Ma^2に平行軸の定理でM(2a+a)^2を加えたもので、合ってると思うのですが…。
56ご冗談でしょう?名無しさん:2009/06/23(火) 22:16:24 ID:???
>>55
バネの質量無視してるのに慣性モーメント変わるの?
57ご冗談でしょう?名無しさん:2009/06/23(火) 22:22:32 ID:???
明記されてないが、状況設定は「バネの一方の端に円盤を取り付け、もう一方の端を中心に回転させた」だと思う。
58ご冗談でしょう?名無しさん:2009/06/23(火) 22:30:51 ID:???
>>27
「力学的エネルギー」だから運動エネルギーだけじゃなく、バネの弾性エネルギー ku^2/2 が必要じゃないか?
kが明示されてないので求まるのかどうかよくわからんが。
59ご冗談でしょう?名無しさん:2009/06/23(火) 22:57:59 ID:/1CaD/Nf
>>58問題文すべてかきます!
質量Mの円板が質量の無視できるバネを介してXYZ座標軸の原点に固定され、バネと円板は一体となりXY平面内においてZ軸まわりを回転する。
最初このバネは自然長2aで固定され、角速度はω2であった。固定を解除したところ、バネは伸び縮み運動を開始した。
これに伴う力学的エネルギーの損失により、十分時間がたった後は、バネの長さは3aとなり、一定角速度ω2で回転することとなった。
ただし、バネの一端はZ軸周りに摩擦無く回転でき、他端は円板の円周上の一点に固定されているものとする。
60ご冗談でしょう?名無しさん:2009/06/23(火) 22:59:54 ID:???
>>57
きっとそうだ! そうだろ>27?

話が噛み合ないはずだ。問題が妙に難しいと思った、、、。
61ご冗談でしょう?名無しさん:2009/06/23(火) 23:05:54 ID:/1CaD/Nf
>>60はい!そのとおりです!明記してなくてすみませんでした!
62ご冗談でしょう?名無しさん:2009/06/24(水) 00:03:45 ID:???
>>38
これはひどすぎだろwww
63ご冗談でしょう?名無しさん:2009/06/24(水) 00:14:39 ID:???
カルノーの定理が禁じているのは不可逆機関の熱効率が可逆機関の熱効率を越えることであって、
可逆機関の熱効率と等しい熱効率をもつ不可逆機関の存在までは否定していない、で合っていますか?
64ご冗談でしょう?名無しさん:2009/06/24(水) 00:19:36 ID:???
否定してないどころか、理論上は有ると示唆してる。
65ご冗談でしょう?名無しさん:2009/06/24(水) 00:24:21 ID:???
>>63
不可逆過程ではd'Q/T<dSになるから無理だろう
66ご冗談でしょう?名無しさん:2009/06/24(水) 00:31:01 ID:???
>>27
2番目は運動エネルギーを重複して数えているように見える
67ご冗談でしょう?名無しさん:2009/06/24(水) 00:39:27 ID:???
>>51
国立医学部で、医者が物理を教えているのか??
普通は物理は物理の教員が教えると思うが、そんなにスタッフの足りない
医学部あるのか??
68ご冗談でしょう?名無しさん:2009/06/24(水) 00:45:40 ID:???
>>64>>65
ありがとうございます

熱力学に関してもうひとつ質問があります
エントロピー増大則の例として、
高温物体(T_2)と低温物体(T_1)を接触させたとき、
ΔS=Q(1/T_1 - 1/T_2)であってT_1<T_2よりΔS>0である、
というのが複数の教科書に載っていました

しかしこれは不可逆過程なのだから、正しくはΔS>Q(1/T_1 - 1/T_2)だと思うんですが、違っていますか
ΔS>0という結論には違いがないから適当に書いているだけですか?
69ご冗談でしょう?名無しさん:2009/06/24(水) 01:01:55 ID:???
>>68
教科書が正しい。断熱過程でΔS=Q(1/T_1 - 1/T_2)>0
だから不可逆過程だと判定できるわけだ。
70ご冗談でしょう?名無しさん:2009/06/24(水) 01:09:29 ID:???
>>65>>69
お前本当に熱力学分かってんの?
71ご冗談でしょう?名無しさん:2009/06/24(水) 01:14:50 ID:???
熱力学は分かってるつもりでも実は分かってない人の多い分野
72ご冗談でしょう?名無しさん:2009/06/24(水) 01:16:26 ID:???
>>69
そこらへんを何度か考えたんですが、
ΔS = ΔQ / T が成り立つのは可逆過程だけですよね?

少し考え直して、>>68で自分が考えたのとは異なりますが、以下のようになるんではないでしょうか
高温物体に関して(可逆だろうと不可逆だろうと)ΔS_2 ≧ -Q / T_2
同様にΔS_1 ≧ Q / T_1
さらに全体としてみると断熱系だからΔS≧0
等号は全て可逆過程の時に成立
さらにΔS=ΔS1+ΔS2だから、
ΔS≧ Q/T_1 - Q/T_2 > 0
よってΔS > 0
よってこの過程は可逆ではありえない。

あくまでΔS≧ Q/T_1 - Q/T_2ではないかと思うんです
=が成り立つ理由が分かりません……
73ご冗談でしょう?名無しさん:2009/06/24(水) 01:17:13 ID:???
>>70
お前よりはね。
74ご冗談でしょう?名無しさん:2009/06/24(水) 01:48:08 ID:???
>>72
>=が成り立つ理由が分かりません……

Sは状態量だから途中の過程によらない。
だから2つの状態を結ぶ適当な可逆過程を考えて計算すればいい。
具体的にはカルノーサイクルとか。
75ご冗談でしょう?名無しさん:2009/06/24(水) 01:56:59 ID:???
>>72
>高温物体に関して(可逆だろうと不可逆だろうと)ΔS_2 ≧ -Q / T_2

ついでにつっこんどくと、d'Q/T≧dSのTは「外界(熱源)の温度」なので
その不等式はΔS_2>-Q/T_1が正しい。
76ご冗談でしょう?名無しさん:2009/06/24(水) 02:09:59 ID:???
>>74-75
やっと分かりました!ありがとうございます!
可逆的に熱量Qを高温物体T_2から奪うとき、
外界の温度もT_2だからΔS_2=Q/T_2になって、以下同様ってことですよね
僕がやったのだと>>75の通りΔS>Q/T_2-Q/T_1<0だから不可逆過程であることは示せない
77ご冗談でしょう?名無しさん:2009/06/24(水) 03:30:06 ID:???
333hPa60℃の宇宙船に999hPa20℃の外気を注入すれば。およそ33℃になる。
思考実験でもわかるだろう
78ご冗談でしょう?名無しさん:2009/06/24(水) 07:48:35 ID:???
そして1hPa100℃の宇宙船に0.01hPa200℃の外気を注入すれば。およそ−9800℃になる。
これで絶対零度など存在しないことがお分かりになっただろう
7958:2009/06/24(水) 09:17:08 ID:???
あと
> ΔU=(1/2){MV2^2+I2・ω2^2}−(1/2){MV1^2+I1・ω1^2}

運動エネルギーの分だけとしてもこれおかしいよね。I1, I2 は円板の重心が軸から離れてる分も
入ってるんだから、MV1^2/2、MV2^2/2 は余計だ。

 ΔU=(1/2)I2・ω2^2−(1/2)I1・ω1^2



 ΔU=(1/2){MV2^2+I0・ω2^2}−(1/2){MV1^2+I0・ω1^2}
  (I0 = (1/2)Ma^2)

じゃないと。
80ご冗談でしょう?名無しさん:2009/06/24(水) 10:13:10 ID:???
>>78
   f`::'ー 、,-、-、_ _,....-- 、_  _,....-=―ヽ―-、-、_        、ミ川川川彡
   ,.r'‐'゙´ヽ,r'  ヽ \ー、_:::::::::/,´:::::::::::,:::::::,::::::::ヽ::\`ー、     ミ       彡
 ,〃ィ  ,rヽ'-ヽ i 、 、 ヾ,、 `'y',ィ´/::::::::/|::::::ハ_::::::::ト、::::\ \  三  ギ  そ  三
r'/〃//    | i! |, \、_`ー!rf.,イ-,ィ/u ノ::::/ |::::`::::|iヽ::::::ヽ  ヽ. 三   ャ れ  三
iヾ!l i /,.=ヽ  i,ケ ハ,i', Y't=ラ゙,〉'|::::r'|! 彡´ ,!--、 |i!|::|::i::::::',   ', 三  グ  は  三
  {ヾllッ-,   〃ノ'-'、||ii i|i| |-/! /^ヽ    ´   ヾ|从ノ::i::::|   ||三  で     三
  >|゙! 0ヽ ノ' ´ 0 レノWノi |,.、!/ 0       0  ',' レ|,イ::::i,,_ | !三  言  ひ  三
  ',i ヽ- ,      _, "  |i| | |             ´ '´ハ',Y  .!三  っ  ょ 三
  /l   `        !| | i `´ r     'ー‐' u   (-, ' |   !三  て  っ  三
  /久        U  |! i|'´'、u              z_,ノ/ .i |三  る  と  三
 /イ |ヽ '==..‐_、      |!,'|Y´,ヽ  ___        ハ_ ,/i | |三  の  し 三
 |ト|、',::::\     _,.-‐イ//-'´::::!\'ー‐--ニュ     / ト_、 _| _!,=,|三  か て 三
  〉:ヾ_'、::::`ー‐r<   //イ|::::::_、:::`7i\ __,..-‐'´  .| |`゙"::"::|-"三  !?    三
 'ー‐'´¨`'ー、/,rケ  /,'1ノ人'-‐'`y'/::::, i| ,!,      |`iiイ:::::::::|  彡     三
    ,〃7,‐/ {   ´_,-'´ ,,‐!、=,/.〃::::i i|kハ    / ,ヾ、::::::|.|!  彡川川川ミ
   ,ッ'、_〃'f /゙-<´  r〃 〃 /イ::::,!ッ'/  ',  / /   |ト、:|:リ
  rir' 〃,y'、久_,.rヽ/〃  川/iケir'〃/ ,-'水´  /    〃  \
  f::}'ー'〃_i| /::::f|::::',  .〃 r/if |||ir' f| レ' r'o | .〃-、 〃     "i
  ,);'ーッ゙-. レ:::::/_|::::::',_,〃=_、!!|| !i/ ||:,ri   !o ∨/_)_〃        |
81ご冗談でしょう?名無しさん:2009/06/24(水) 10:23:00 ID:???
>>78
お前面白いな
好きだわ
82ご冗談でしょう?名無しさん:2009/06/24(水) 11:08:26 ID:???
>>58
バネ常数kは角速度はω2になったときの釣り合いから
k a = 4 a M ω2^2
で求まりそうだ。そうすると、結論は、、、
83ご冗談でしょう?名無しさん:2009/06/24(水) 11:22:07 ID:???
(sinx)^2=sin2x

でよかったんでしたっけ?
それとも積分結果がsin2xになるんだっけ
84ご冗談でしょう?名無しさん:2009/06/24(水) 11:34:00 ID:???
>>83
xが実数なら左辺≧0だから、右辺とは違うのは明らか。
積分すれば違うのもほぼ自明、、。
85ご冗談でしょう?名無しさん:2009/06/24(水) 11:54:36 ID:PTnLjZ42
コマ、地球ゴマについて質問です。
まだ、原理的に、何故立ち続けられるか、解明されきっていないというのは本当ですか?
たとえば、紐の先に横に立って回転し続ける地球ゴマなんかは、
そのメカニズムが解明されていないと言う話を聞きましたが本当ですか?
86ご冗談でしょう?名無しさん:2009/06/24(水) 11:55:17 ID:???
(sinx)^2を積分結果はsin2xで合ってるよね?
87ご冗談でしょう?名無しさん:2009/06/24(水) 11:56:04 ID:???
× (sinx)^2を積分結果はsin2xで合ってるよね?
○ (sinx)^2の積分結果はsin2xで合ってるよね?
88ご冗談でしょう?名無しさん:2009/06/24(水) 12:13:37 ID:???
d(sinx)^2/dx = 2sinxcosx = sin2x
89ご冗談でしょう?名無しさん:2009/06/24(水) 12:15:16 ID:???
>>85
うそ

>>87
微分だろう
90ご冗談でしょう?名無しさん:2009/06/24(水) 12:21:07 ID:zOZkJxfE
>>82ということは、
始めの力学的エネルギーは回転と運動で、
後の力学的エネルギーは回転と運動とバネののびということですか?
91ご冗談でしょう?名無しさん:2009/06/24(水) 12:35:07 ID:???
>>85
剛体の力学で必ずやるくらいに解明されまくり
92ご冗談でしょう?名無しさん:2009/06/24(水) 12:37:48 ID:???
>>90
回転エネルギーなんて使う?運動エネルギーだろ?
93ご冗談でしょう?名無しさん:2009/06/24(水) 13:26:42 ID:zOZkJxfE
>>92円板の運動エネルギーと、原点を中心とした回転のエネルギーを考えなきゃいけないのではないのでしょうか?
94ご冗談でしょう?名無しさん:2009/06/24(水) 13:41:07 ID:???
>>93
その二つは違うものなのか?
95ご冗談でしょう?名無しさん:2009/06/24(水) 13:41:11 ID:???
糸を伝わる波の速度は
v=√(張力S/糸の線密度ρ)
と高校で習いましたが
これは経験則でしょうか?
96ご冗談でしょう?名無しさん:2009/06/24(水) 13:46:35 ID:???
経験則とそうでない法則を各々定義してくれ
97ご冗談でしょう?名無しさん:2009/06/24(水) 13:47:32 ID:???
ちょっとした発展問題が載ってる問題集なら、張力と線密度から、
高校物理レベルのモデルを使って近似で波の速度を導く問題とかあったような。
98ご冗談でしょう?名無しさん:2009/06/24(水) 13:52:04 ID:PTnLjZ42
ジャイロ効果を打ち消し合うと言う、2つの回転体を繋げる方法について教えてください。

これは、つまり、一本の棒の両端に2つの回転体が付いているという事なのですか?
すんません。物理的にどんな風に繋がってるのかわかりません。
教えてください。
99ご冗談でしょう?名無しさん:2009/06/24(水) 17:04:02 ID:???
どんな繋がりかたでもいいからとにかくトータルの角運動量が0になるように繋げりゃいいのでは?
もちろん
>一本の棒の両端に2つの回転体が付いている
それもひとつの方法
100ご冗談でしょう?名無しさん:2009/06/24(水) 18:04:11 ID:???
ちょうど疑問に思ってたので便乗質問。
たとえば二つの地球ゴマを、軸が一直線状になるようにくっつけて、上下で逆回しにしたとする。
この連ゴマは立たずにパタンと倒れるんだよね?
101ご冗談でしょう?名無しさん:2009/06/24(水) 20:07:41 ID:PTnLjZ42
ふんじゃ、2つの地球ゴマを逆回転させたうえで、
縦に乗せたらどうなるのか?
102ご冗談でしょう?名無しさん:2009/06/24(水) 20:09:54 ID:PTnLjZ42
しかし、何故、逆向きに繋げたらジャイロ効果が打ち消し合うのですか?
103ご冗談でしょう?名無しさん:2009/06/24(水) 20:11:52 ID:PTnLjZ42
理論上は、どちら向きの回転であろうと、
傾けた力に対して反発する力が発生するはず。
つまり、どちら向きの回転であろうと、
力の方向は全く同じ方向の力が発生するはず。

では、無いのか?
104ご冗談でしょう?名無しさん:2009/06/24(水) 20:43:21 ID:???
歳差 でぐぐれ
105ご冗談でしょう?名無しさん:2009/06/24(水) 20:44:28 ID:???
>>103
>理論上は
何の理論だよ。
106ご冗談でしょう?名無しさん:2009/06/24(水) 20:45:02 ID:???
>>100  倒れる
107ご冗談でしょう?名無しさん:2009/06/24(水) 20:57:28 ID:???
>>76
ご名答。熱力学はどの分野に進んでも必ず役に立つから、
しっかり勉強することを勧めるよ。
108ご冗談でしょう?名無しさん:2009/06/24(水) 21:08:20 ID:PTnLjZ42
あ゛あ゛っ!!
ワレコラッ!!
109ご冗談でしょう?名無しさん:2009/06/24(水) 21:11:23 ID:PTnLjZ42
ジャイロ効果を得るための物質は、なんでも良いというわけではないですよね。
たとえば、ゴムより鉄の方が、同じ重さ、回転力であったとしても、
ジャイロ効果は高くなりますよね。
110ご冗談でしょう?名無しさん:2009/06/24(水) 21:15:59 ID:Nv8xNSRs
NG ID:PTnLjZ42
111ご冗談でしょう?名無しさん:2009/06/24(水) 22:01:24 ID:???
エネルギー結晶に、シリコンウエーバーを差し込むといいことがあるぞ
112ご冗談でしょう?名無しさん:2009/06/24(水) 22:08:31 ID:???
> 理論上は、どちら向きの回転であろうと、

どういう「理論」?

> 傾けた力に対して反発する力が発生するはず。

普通の「理論 (=ニュートン力学)」ではそんな力は発生しないんだよ。各運動量の変化率に応じた
モーメントが生じるだけでね。
113ご冗談でしょう?名無しさん:2009/06/24(水) 22:39:54 ID:???
運動方程式について聞きたいんけど

ma=F

m(dv/dt)=F

dm/dt=0のとき

d(mv)/dt=F @


ここまでは分かる

@は(dm/dt)v+m(dv/dt)=F に変形できて
mが変数のときにも使えるから@の式の方がより厳密な運動方程式なんだ!
って教授言ってたんだけどさ

でもmが定数って条件で@が導出されてるんだから
mが変数のときは@成り立たなくないか?

114ご冗談でしょう?名無しさん:2009/06/24(水) 22:48:28 ID:???
dm/dt≠0 のときは d(mv)/dt=F が成り立って ma=F が成り立たないと言いたいんだろう。より厳密というより、より一般的という感じ。
115ご冗談でしょう?名無しさん:2009/06/24(水) 23:39:06 ID:5zjoEnTW
普通、何keVのX線なら0.1mmのアルミ透過後は50%に減るとか表現しますが、
X線粒子のエネルギーが減って波長が変わる訳じゃないですよね。
数が減るだけという考えで合ってますか?
116ご冗談でしょう?名無しさん:2009/06/25(木) 00:14:19 ID:???
>>115
コンプトン散乱も起こりそうだけど…?
117ご冗談でしょう?名無しさん:2009/06/25(木) 00:20:11 ID:???
原子が1×10のマイナス10乗メートルの大きさとして
地球を原子の大きさとすると
1メートルはどのくらいの大きさになりますか?
118ご冗談でしょう?名無しさん:2009/06/25(木) 00:28:39 ID:???
>>116
まだ勉強中なのですが、コンプトン散乱すなわちターゲットの電子と
エネルギーのやりとりが有るとX線の波長が変わる訳ですね。
それは分かるのですが、普通に10keVくらいのX線がアルミ箔を
通る場合はどのような過程が支配的なのでしょうか?
119ご冗談でしょう?名無しさん:2009/06/25(木) 01:44:03 ID:???
>>93

>>79でも書いたけどそれじゃだめだってば
120ご冗談でしょう?名無しさん:2009/06/25(木) 02:19:52 ID:lZ8Satkc
こんばんは。
ヘリウムの励起状態(1s2s)のエネルギーを変分法で求めるにあたり、
クーロン積分・交換積分を行いたいのですが、積分が上手くいきません。

どなたか解法が載っているサイトや文献ご存じないでしょうか?

よろしくお願いします。
121ご冗談でしょう?名無しさん:2009/06/25(木) 05:39:27 ID:RNWclz3Z
>>27>>59結論が出なかったのでお願いします!
Aは運動エネルギー、原点に対する円板の回転エネルギー、男性エネルギーを考えればよいのでしょうか?
122ご冗談でしょう?名無しさん:2009/06/25(木) 08:28:53 ID:???
だから…

> 運動エネルギー、原点に対する円板の回転エネルギー

君が言ってる「運動エネルギー」って

   円板質量×円板中心 (重心) の線速度^2 / 2
   始 MV1^2/2 = M(3aω1)^2/2
   終 MV2^2/2 = M(4aω2)^2/2

で「原点に対する円板の回転エネルギー」って

   原点周りの慣性モーメント×角速度^2 / 2
   始 I1・ω1^2/2 = {Ma^2/2 + M(3a)^2}・ω1^2/2 = Ma^2/2・ω1^2 + M(3aω1)^2/2
   終 I2・ω2^2/2 = {Ma^2/2 + M(4a)^2}・ω2^2/2 = Ma^2/2・ω2^2 + M(4aω2)^2/2

でしょ? 「回転エネルギー」の2項目が「運動エネルギー」と重複してるって気づかない?
123ご冗談でしょう?名無しさん:2009/06/25(木) 08:54:04 ID:???
>>114
@がdm/dt≠0でも成り立つならそれも納得できるんだけどさ・・・
124ご冗談でしょう?名無しさん:2009/06/25(木) 09:00:57 ID:???
>>123
dm/dt≠0 のときは m・dv/dt = F は成り立たないけど d(mv)/dt = F なら成り立つ、って話でしょ。

>>113 の式は、

  dm/dt = 0 のときは両者が一致する

って話であって

  dm/dt = 0 という条件で m・dv/dt =F から d(mv)/dt = F が導出される

という話ではない
125ご冗談でしょう?名無しさん:2009/06/25(木) 09:31:03 ID:RNWclz3Z
>>122
やっとわかりました!運動エネルギーと弾性エネルギーを考えれば良いのですね!
ありがとうございました!
126ご冗談でしょう?名無しさん:2009/06/25(木) 10:08:36 ID:eTbYyEWV
>>124

F=maからは@は導出はできないって解釈しておk?
127ご冗談でしょう?名無しさん:2009/06/25(木) 10:08:42 ID:???
>>118
そのエネルギーなら光電効果が支配的
128ご冗談でしょう?名無しさん:2009/06/25(木) 11:07:51 ID:???
とても大きな物体がは秒速10万キロメートル(適当に速い速度)で動く慣性系であるとして、
その慣性系の物体上で同じく秒速10万キロメートルで動くやや大きな物体があるとします。
そんな感じのことを繰り返して、それで方向も上手い具合に合ってたとしたら、
光速は超えられないにしても亜光速で動くものを
現実的に人工的に作ることができると思ったのですが、どうでしょうか?

言葉の使い方が間違ってるかもしれませんが、本当に素人なので許してください。でも科学はとても好きです。
129ご冗談でしょう?名無しさん:2009/06/25(木) 11:24:57 ID:???
>>128
べつにそんな妙な物体を考えなくても、加速器では電子や陽子を光速に近い速度まで実際に加速しているわけだが
130ご冗談でしょう?名無しさん:2009/06/25(木) 11:39:08 ID:HodCJRf1
質問です。s1+s2=sで、s1=s2=1/2なのでs=1,0と本に書いてあったのですがs=0はなんでですか?高校生に分かるように教えて下さい。
131ご冗談でしょう?名無しさん:2009/06/25(木) 11:49:45 ID:zw76xkLf
>>67
亀ですがこの問題は物理の試験に出てきた問題ではないです。
機械とか超音波とかそこらへんの工学の科目です。
物理は物理専門の教員が教えてましたよ。
132ご冗談でしょう?名無しさん:2009/06/25(木) 12:13:26 ID:JMO4oBcf
>>129
大きな物体でも、光速近くまで加速させることは、できますか?よろしくお願いします。
133ご冗談でしょう?名無しさん:2009/06/25(木) 12:15:28 ID:???
思考実験なら際限なく光速に近づけるよ。
134ご冗談でしょう?名無しさん:2009/06/25(木) 12:19:32 ID:???
>>127
ありがとうございます。
光電効果、コンプトン効果で検索したらそういう解説ページが出てきました。
光電効果ならアルミをすり抜けてきたX線もやはり10keVと考えて良い訳ですね。
135ご冗談でしょう?名無しさん:2009/06/25(木) 12:55:47 ID:???
>>132
予算の続く限りどこまでも
136ご冗談でしょう?名無しさん:2009/06/25(木) 13:12:36 ID:???
>>132
二段噴射、三段噴射なら可能かも。
137ご冗談でしょう?名無しさん:2009/06/25(木) 13:26:54 ID:???
>>130
大きさが同じ二つのベクトルを足したらゼロになることもあるでしょ
138ご冗談でしょう?名無しさん:2009/06/25(木) 13:32:25 ID:???
>>126
F=d(mv)/dt の方が基本で、dm/dt=0 のときに限り F=ma が導出できるということ。
F=ma は基本的な運動方程式ではないという立場から始める。
139ご冗談でしょう?名無しさん:2009/06/25(木) 13:41:16 ID:???
空間に、真空空間、亜空間を入れたら、宇宙で浮きますか?ちょっとした疑問でごめんw
140ご冗談でしょう?名無しさん:2009/06/25(木) 13:49:16 ID:???
真空空間の作り方は、ブラックホールを内部で作るお
141ご冗談でしょう?名無しさん:2009/06/25(木) 13:57:33 ID:???
>>134
大部分は、な。ただし、一次反応として光電効果だけを考えても、
光電効果で蹴飛ばされた電子の占めていた軌道に上から電子が落ちて
くるときに出るアルミの特性X線とか、蹴飛ばされた電子自体が
制動輻射で出すX線もあるから100%ではない
142ご冗談でしょう?名無しさん:2009/06/25(木) 14:10:56 ID:???
亜空間の作り方も同じおw
143ご冗談でしょう?名無しさん:2009/06/25(木) 14:24:38 ID:???
ベクトルで計算すれば、理解できる
144ご冗談でしょう?名無しさん:2009/06/25(木) 14:31:39 ID:???
空間砲をつくっては、いけないおww
145ご冗談でしょう?名無しさん:2009/06/25(木) 14:45:12 ID:???
おw とかキモ
146ご冗談でしょう?名無しさん:2009/06/25(木) 15:22:17 ID:???
防御としての空間板もできるな。ミサイルの防御として使えるな。
147ご冗談でしょう?名無しさん:2009/06/25(木) 15:54:24 ID:???
>>141
なるほど、どうもです。
そういった現象がどんな割合で起こるのかとかも、知りたいのですが、
wabサイトで詳しいところなんて無いですよね?
148ご冗談でしょう?名無しさん:2009/06/25(木) 16:04:39 ID:eTbYyEWV
>>138
ありがとう
すっきりした
149ご冗談でしょう?名無しさん:2009/06/25(木) 18:48:00 ID:CvQAE4Dj
http://www.dotup.org/uploda/www.dotup.org168799.jpg.html
誰かこの[問53]教えて下さい・・
150ご冗談でしょう?名無しさん:2009/06/25(木) 18:50:44 ID:lEt7UCIZ
重力が高ければ、高いほど、時間の進み方が遅いってことは地球の六分の一の重力の月は時間の進み方が速いんですか?
それと、重力を生む装置ってありますか?
そういうの使っていけば、自分の時間遅らせることってのは可能でしょうか?(重力を生む装置の中で、寝て起きた時には8時間しか寝てないつもりなのに、外では80年たってるとか)

頭悪いんで、馬鹿馬鹿しい質問かもしれないですけど答えくれるとうれしいです。
151ご冗談でしょう?名無しさん:2009/06/25(木) 19:00:23 ID:???
重力発生させなくても、君が動いてるだけで、一応君の時間は周りで静止してる奴の時間より進み方は遅れてるぞ
152ご冗談でしょう?名無しさん:2009/06/25(木) 19:00:48 ID:???
>>149
>>1
宿題を聞くときは、どこまでやってみてどこが分からないのかを書くこと。
丸投げはダメだからね

>>150
>地球の六分の一の重力の月は時間の進み方が速いんですか?
一応そういうことになっている、もちろん人間の生きるスケールからは無視できる程度のずれだが
>それと、重力を生む装置ってありますか?
莫大なエネルギー(質量もその一種だけど)を集めれば、重力は発生する
ただ、局所的に重力を産むのは難しいだろうね、たとえ発生させても周りの時間もみんな一緒に遅れるとか
そもそもそんなエネルギーをどこから持ってくるのかって話もあるし
一応、重力の代わりに加速・減速を繰り返しても同じ効果が得られるけど、
無視できない程度ずらすには人間が到底耐えられないくらいの加速度が必要
153149:2009/06/25(木) 19:18:35 ID:CvQAE4Dj
>>152
(227)を出そうとすると、符号が逆になるんです・
分配されたもので、jの数が違うもの同士では消えるのはわかってて、jが同じのだけを計算していけばいいのはわかるんです

で、例えばj=1の時
<dq1∧dp1|(dp1/dt)(∂/∂p1)> + <dq1∧dp1|(dq1/dt)(∂/∂q1)>

これが、
(dq1/dt)dp1−(dp1/dt)dq1
になってほしいんですが、この過程がわかりません

dq1∧dq1=−dq1∧dp1の関係はわかるんですけど・・
154ご冗談でしょう?名無しさん:2009/06/25(木) 19:28:03 ID:lEt7UCIZ
>>152
もしも地球より重力が高い惑星(それがあるかどうかは知らないんですけど)に住んでいれば
無視できる程度だとしても、地球の人より老化は遅くなりますか?

>無視できない程度ずらすには人間が到底耐えられないくらいの加速度が必要
光に近い速度でぐらいですか?そうすれば他の人たちより歳をとらないですか?
155149:2009/06/25(木) 19:34:59 ID:CvQAE4Dj
>>153の最後ミスってました・・
dq1∧dp1=−dp1∧dq1でした

あと
<dq1∧dp1|∂/∂p1>=dq1

<dq1∧dp1|∂/∂q1>=<−dp1∧dq1|∂/∂q1>=−dp1
となるのもわかるんです

問題は、偏微分の左にある(dp1/dt)や(dq1/dt)の扱いをどうすればいいのかわからないんです

156ご冗談でしょう?名無しさん:2009/06/25(木) 19:40:51 ID:???
>>155
内部積というやつだね。符号逆じゃないか?
>問題は、偏微分の左にある(dp1/dt)や(dq1/dt)の扱いをどうすればいいのかわからないんです
内部積の定義知ってれば分かるはずだけど、そのまま外に出せばいい
157149:2009/06/25(木) 19:44:16 ID:CvQAE4Dj
>>156
だから、そのまま外に出すと符号が逆になってしまう・・
だからよくわからないんです・・
んん〜?
158ご冗談でしょう?名無しさん:2009/06/25(木) 19:46:41 ID:???
だから
<dq1∧dp1|∂/∂p1>=−dq1
<dq1∧dp1|∂/∂q1>=dp1
でしょう。
159ご冗談でしょう?名無しさん:2009/06/25(木) 19:53:28 ID:???
ただ単に、合成関数の微分(いわゆるチェーンルール)がわかってないだけでしょ。
学部一年で理解しておかなければいけない基本事項。
160149:2009/06/25(木) 19:55:54 ID:???
え?
<dq1∧dp1|∂/∂p1>=dq1
<dq1∧dp1|∂/∂q1>=−dp1
はあってるんじゃないですか?
161ご冗談でしょう?名無しさん:2009/06/25(木) 20:08:52 ID:???
>>159
違うでしょう。

>>160
授業でどう習ったの?
数学だと i_{∂/∂p} dq∧dp = -dq とか書く。
http://ja.wikipedia.org/wiki/%E3%83%AA%E3%83%BC%E5%BE%AE%E5%88%86
の真ん中辺り。
162ご冗談でしょう?名無しさん:2009/06/25(木) 20:16:05 ID:1gPtvXGX
理想電流源を直列につなぐとどうなるんですか?

例えば1Aの電流源、2Aの電流源、1Ωの負荷をつないだとき、負荷電流はいくらになるんでしょうか?
163149:2009/06/25(木) 20:41:21 ID:???
>>161
<dq1∧dp1|∂/∂p1>=dq1
<dq1∧dp1|∂/∂q1>=−dp1
このままヒントとして与えられたんですよ

あと
<dqj|∂/∂qk>=δjk   j≠kなら0、j=kなら1
<dqj|∂/∂pj>=0
というのも教わりました

ん〜物理数学なんで以外とさらっとしかやらないから・・
164ご冗談でしょう?名無しさん:2009/06/25(木) 20:55:29 ID:???
スレ独占しないでくれよ・・・
165ご冗談でしょう?名無しさん:2009/06/25(木) 21:30:20 ID:???
C=0になる?
166ご冗談でしょう?名無しさん:2009/06/25(木) 21:31:17 ID:???
こんにちわ
167ご冗談でしょう?名無しさん:2009/06/25(木) 21:31:54 ID:???
ふとした疑問です。
この板的には糞な質問かと思いますが教えて下さい。
http://p.pita.st/?m=7cwumvvg

ふしぎ!
168ご冗談でしょう?名無しさん:2009/06/25(木) 21:48:10 ID:Tjln/E8i
http://www.youtube.com/watch?v=zjvlPHw0xT8

こういうのってなんていう現象なのでしょうか。エネルギー保存の法則ですか?

複数の玉を並べて、一番端に玉をぶつけると反対側の端の玉がひとつだけ飛び出す法則のことです。
169ご冗談でしょう?名無しさん:2009/06/25(木) 21:48:17 ID:s+kBW/aI
屈折
170ご冗談でしょう?名無しさん:2009/06/25(木) 21:54:16 ID:???
>>168
エネルギーの保存と運動量の保存が両方とも成立するには、同じ量の玉が同じ速度で飛び出さなければならない。
171ご冗談でしょう?名無しさん:2009/06/25(木) 22:10:28 ID:???
>>170
エネルギー保存と運動量保存だけではその結果は導けないよ
ぶつかった方の玉が衝突後静止する、って条件入れれば導けるけど。
172ご冗談でしょう?名無しさん:2009/06/25(木) 22:15:34 ID:???
>>171
他の玉に行く手を阻まれてるんだから静止するのは自明。考察が足りない。
173ご冗談でしょう?名無しさん:2009/06/25(木) 22:15:54 ID:???
>>171
ちゃんと式立てて確認してみな。
174ご冗談でしょう?名無しさん:2009/06/25(木) 22:16:30 ID:oz1Ut8ao
L字型のいたのつりあいについて教えてください。

┏━━━━━
┃ △


△の位置のだしかたがわかりません。
縦6横12です。直角に曲がってるところからいくら???モーメントでだすのでしょうか???
175ご冗談でしょう?名無しさん:2009/06/25(木) 22:17:24 ID:???
>>172
おまえもちゃんと式立てて確認してみろ。球の質量が同じでなければ、
静止しない場合もあるから「静止するのは自明」なんて大法螺。
176ご冗談でしょう?名無しさん:2009/06/25(木) 22:18:47 ID:???
>>174
モーメントで出せ。
177ご冗談でしょう?名無しさん:2009/06/25(木) 22:20:50 ID:???
>>162
基本的に理想定流電源を開放したのと同じ
理想電源でそのような状況は考えられない
178ご冗談でしょう?名無しさん:2009/06/25(木) 22:22:20 ID:oz1Ut8ao
>>>176やりかたがわかりません。先生に質問してもヒントしか教えてもらえずさっぱりです。

179ご冗談でしょう?名無しさん:2009/06/25(木) 22:22:34 ID:???
A=d/dx B=x(d/dx)

この場合この交換関係[A,B]ってのは、どうなるんですか?
d/dxであってます?
180ご冗談でしょう?名無しさん:2009/06/25(木) 22:24:06 ID:???
>>178
ヒントもらったらじゅーぶんじゃねーか。じゃあ俺もヒントやる。

縦線の部分と横線の部分にわけてそれぞれのモーメント考えろ。
181ご冗談でしょう?名無しさん:2009/06/25(木) 22:24:39 ID:???
>>175
球の質量がまちまちだと?バカか。
問題としてカオス過ぎんだろうが。
182ご冗談でしょう?名無しさん:2009/06/25(木) 22:27:19 ID:???
>>181
まちまちにしてカオスにしろなんて誰も言ってない。カオスってほどの
カオスでもないがな。

>>172が「他の玉に行く手を阻まれてるんだから静止するのは自明」
なんてスカを言っているからそれはスカだと言っている。
>>172の言うこと信用すれば、1kgの物体が前を阻んでいたら1tの物体が
やってきても行く手を阻まれて静止するのが自明らしい。
183ご冗談でしょう?名無しさん:2009/06/25(木) 22:27:44 ID:???
球の質量がまちまちだからといって「問題としてカオス過ぎ」って、どんな低レベルな物理しかやってないの?w
184ご冗談でしょう?名無しさん:2009/06/25(木) 22:29:30 ID:???
>>182
おまえ動画観ずに書いてるだろ。
動画観た上でそのコメントなら、科学の素養がない。
185ご冗談でしょう?名無しさん:2009/06/25(木) 22:30:28 ID:???
確かに動画は見てないが、>>172が「自明」と述べていることがちっとも
自明じゃないのはわかる。
186ご冗談でしょう?名無しさん:2009/06/25(木) 22:32:37 ID:???
もしかして、動画は球がたくさんあるから、とか思っているのか???
(いやまだ見ていないが)

それにしたって、衝突は一個ずつ逐次的に起こるんだが。

187ご冗談でしょう?名無しさん:2009/06/25(木) 22:34:11 ID:oz1Ut8ao
>>>180すいませんヒントの意味が全くわからず……


答えお願いいたします。
188ご冗談でしょう?名無しさん:2009/06/25(木) 22:34:58 ID:???
ちなみに「たくさんの球の一団」と一個の球の衝突がいっきに起こっている
なら、一個目の球は静止せず跳ね返るよ。それも運動量保存とエネルギー
保存で出せる。
189ご冗談でしょう?名無しさん:2009/06/25(木) 22:38:26 ID:???
丸投げ君は完全無視。
190ご冗談でしょう?名無しさん:2009/06/25(木) 22:38:43 ID:???
>>188
衝突が一個ずつ逐次起こってるから止まるんだろ。
191ご冗談でしょう?名無しさん:2009/06/25(木) 22:39:31 ID:???
だから>>186でそう言っているじゃん。>>190
192ご冗談でしょう?名無しさん:2009/06/25(木) 22:41:03 ID:???
>>183
おまえも観察力の残念なバカか。低レベルな物理の問題なんだよ。
193ご冗談でしょう?名無しさん:2009/06/25(木) 22:41:55 ID:jlpjyQp9
>>179お願いします!!
194ご冗談でしょう?名無しさん:2009/06/25(木) 22:44:44 ID:???
あってるよ。>>179
195ご冗談でしょう?名無しさん:2009/06/25(木) 22:45:49 ID:???
>>179
単なる計算じゃねえか
答え出てんのになにが疑問なんだよ
196ご冗談でしょう?名無しさん:2009/06/25(木) 22:49:12 ID:???
>>194>>195
ありがとうございます
ちょっと自身がなかったので、確かめたかっただけです
197ご冗談でしょう?名無しさん:2009/06/25(木) 22:51:48 ID:???
>>187
(横スレだが)ヒント・・・左の縦線から支点(△)までの距離をXとおく。
そうすると、それぞれ力の大きさと作用点が計算できるから、あとは
支点に関してモーメントの釣り合いを考えればOK。
198ご冗談でしょう?名無しさん:2009/06/25(木) 22:53:12 ID:???
「答えお願いします」なんて奴に親切にするこたあないのに。>>197
199170:2009/06/25(木) 22:53:42 ID:???
>>171-172

同じ質量 m の玉 n 個が並んで静止しているところに、もう一つ同じ質量の玉が速度 v1 で
衝突する。衝突後、ぶつかった方の玉は速度v1'で運動し、並んでいた n 個の玉は一斉に
速度 v2' で運動する、という条件をつけると、

  v1' = -(n-1)/(n+1)・v1
  v2' = 2/(n+1)・v1

とすれば、運動量と運動エネルギーは保存する (疑うなら自分で計算して見れ)

v1' は v1 と符号が逆 (n≧2のとき)、つまりぶつかった方の玉ははじき返されるので
「他の玉に行く手を阻まれ」ることなどない。

こういう解があることは計算するまでもなく自明だよね。
質量 m の物体と質量 nm (nは2以上の整数) の物体の弾性衝突を考えればいいだけなんだから。

>>175はそういう意味で言ったんだと思うが、>>181はそれを「並んだ玉自体の質量がまちまち」と
しか受け止められなかったらしい。
200199=171≠170:2009/06/25(木) 22:55:11 ID:???
レス番がずれてた。
>>199>>171で、アンカーの>>171-172>>172-173 の間違い。
201ご冗談でしょう?名無しさん:2009/06/25(木) 22:56:16 ID:???
よーし、じゃあ次は一様な密度を持った球の衝突で考えてみよう!
202ご冗談でしょう?名無しさん:2009/06/25(木) 22:59:54 ID:???
>>199
観察結果とあってないから、その条件は棄却な。
203ご冗談でしょう?名無しさん:2009/06/25(木) 23:08:16 ID:???
v1 = 1, n = 2, m = 1とする

v1' = -(2-1)/(2+1)*1 = -1/3
v2' = 1/(2+1)*1= 1/3

p1 = 1*1 = 1
p2 = 1*(-1/3)+2*1*(1/3) = 1/3
p1 ≠ p2

E1 = 1*1^2/2 = 1/2
E2 = 1*(-1/3)^2/2+2*1*(1/3)^2/2 = 1/6
E1 ≠ E2
204ご冗談でしょう?名無しさん:2009/06/25(木) 23:12:50 ID:???
> v2' = 1/(2+1)*1= 2/3
205ご冗談でしょう?名無しさん:2009/06/25(木) 23:13:26 ID:oz1Ut8ao
>>>197ありがとうございます。

206ご冗談でしょう?名無しさん:2009/06/25(木) 23:43:05 ID:???
一所懸命長い式書いたのに、間違っていたなんて可哀想>>203
207ご冗談でしょう?名無しさん:2009/06/25(木) 23:46:53 ID:???
> 観察結果とあってないから、その条件は棄却な。

その通りだが。
単に>>170の言うように「運動量と運動エネルギーの保存」だけでは説明できない、というだけの話だ。
208ご冗談でしょう?名無しさん:2009/06/26(金) 00:59:22 ID:???
{(A×D)+[(F−A)×E]}÷F=外気注入後の宇宙船の船内気圧

Aは船内気圧、Dは船内気温、Eは船内気温 Fは船外気圧

これで間違いありません。スレッドを混乱させたお詫びをします尚絶対零度は存在
しないと言ってる人は別人です
209ご冗談でしょう?名無しさん:2009/06/26(金) 01:15:04 ID:???
>>207
逐次的に1対1衝突が起こっていると考えれば(というか実際そういう衝突
が起こっているんだが)、運動エネルギーと運動量の保存だけで話すむぞ。
210ご冗談でしょう?名無しさん:2009/06/26(金) 01:15:21 ID:???
>>208 Eは外気温のまちがいでしたたびたびすいません
211ご冗談でしょう?名無しさん:2009/06/26(金) 01:16:01 ID:???
>>208
式の次元ぐらい合わせられるようになってから一昨日こい。
212ご冗談でしょう?名無しさん:2009/06/26(金) 01:47:39 ID:???
>>208
A=1、D=100、E=200、F=0.01を代入した時、
外気注入後の宇宙船の船内気圧が絶対温度を下回るー9800になることは自明ですね
すみません
213ご冗談でしょう?名無しさん:2009/06/26(金) 01:48:22 ID:???
>>212 絶対温度は絶対零度のまちがいでしたたびたびすいません
214ご冗談でしょう?名無しさん:2009/06/26(金) 02:30:57 ID:???
>>212それは私がした投稿ではありません
215ご冗談でしょう?名無しさん:2009/06/26(金) 03:22:25 ID:UjsHOYs5
>>213何者かが投稿した。書き込みをスクリプが勘違いして私が書いたものと載せた
ものなんですよ。
216ご冗談でしょう?名無しさん:2009/06/26(金) 04:13:03 ID:???
>>208
このスレ、専門内知識のみ人間のオナニースレだから質問の仕方に
気をつけないとスルーどころか袋叩きにあうから気をつけてね。
217ご冗談でしょう?名無しさん:2009/06/26(金) 05:53:03 ID:???
まあ、そういう馬鹿が増えてしまったってことだ。
218ご冗談でしょう?名無しさん:2009/06/26(金) 06:37:53 ID:???
精子と精子の核をつかって子供ができるって知ってた?EUの研究中だけど。
219ご冗談でしょう?名無しさん:2009/06/26(金) 07:47:33 ID:ViRRlAbP
衝突振り子って面白いな。
特に両方から互いにぶつけてみたときの反応が意外だ。
反発係数が高いんだな。
見た目はステンレスみたいに見えるけど、特殊なものなのかな。
反動がすごい。
220ご冗談でしょう?名無しさん:2009/06/26(金) 07:57:48 ID:???
>>211
> 式の次元ぐらい合わせられるようになってから一昨日こい。

文字通り一昨日来てるな。
まあ、飽きずにあちこちのスレに粘着してるだけのことなんだが。
221ご冗談でしょう?名無しさん:2009/06/26(金) 08:15:00 ID:???
>>212 は船内気圧が負となることの証明でしたすみません
これはマイナスエネルギーの一つの証明だと思います
マイナス質量です
222ご冗談でしょう?名無しさん:2009/06/26(金) 10:21:16 ID:oiwNntSe
蜂ってなんで理論上は飛べないなんて言われてたんだっけ
223ご冗談でしょう?名無しさん:2009/06/26(金) 11:50:29 ID:???
「ロケットは真空中で飛べない。高校物理の知識もないのか」
とニューヨークタイムズがゴダードを批判したりしてるしな
224ご冗談でしょう?名無しさん:2009/06/26(金) 12:09:35 ID:rkO5KcSn
>>223
>ロケットは真空中で飛べない
たしかに。
もしかして、宇宙空間にも大気があるの?
225ご冗談でしょう?名無しさん:2009/06/26(金) 12:16:44 ID:???
そっちへいっちゃったかw
226ご冗談でしょう?名無しさん:2009/06/26(金) 13:36:56 ID:???
ロケットは空気を蹴って進むんだからね。
227ご冗談でしょう?名無しさん:2009/06/26(金) 13:55:39 ID:???
大阪のレーザー研危ないから、注意しておけ。
228ご冗談でしょう?名無しさん:2009/06/26(金) 14:40:43 ID:???
ラウス関数を導入する必要性とはなんですか?
ハミルトニアンのままでも同一の関係式に至れるような。
一応、「変換」したよーって事で持ってきてるだけなのかな。お願いします
229ご冗談でしょう?名無しさん:2009/06/26(金) 15:20:17 ID:rkO5KcSn
ロケットには羽がないのに何故に飛べるの?
230223:2009/06/26(金) 15:34:05 ID:???
ノリツッコミを期待してノリの部分を提供したら
ツッコミが来ずにノリノリな香具師が来ちゃったorz
231ご冗談でしょう?名無しさん:2009/06/26(金) 15:46:52 ID:VFHgPFtC
一次元=素粒子の世界
二次元=原子の世界
三次元=分子の世界
四次元=俺達の世界
五次元=宇宙じゃないの?
232ご冗談でしょう?名無しさん:2009/06/26(金) 15:58:42 ID:???
>>230 平日の今頃はバカの跳梁跋扈する時間帯なんだよ。

NYTのネタにはついてけねーよ。
233ご冗談でしょう?名無しさん:2009/06/26(金) 16:03:03 ID:0zNkXpYo
370nmの光のエネルギーっていくらですか。
E=hvで計算したら5×10^-19Jとかでたけど明らかに小さいですよね。

それともプランク定数が10^-34で光速が10^8、ナノが10^-9だからこんなもんなんでしょうか。

水素のイオン化エネルギーということで求めてるんですがこれじゃイオン化しほうだいじゃないですか。

誰か助けてください。
234ご冗談でしょう?名無しさん:2009/06/26(金) 16:25:55 ID:???
>>233
ざっと見た感じ桁は合ってんじゃねーの。
それより「イオン化しほうだいじゃないですか」と思う方が謎だ、
光のエネルギーが低すぎると思うんなら「イオン化なんて出来ないじゃないですか」と思うんじゃね?
235ご冗談でしょう?名無しさん:2009/06/26(金) 16:35:18 ID:0zNkXpYo
必要エネルギーが低い=しほうだいじゃないですかどこが違いますか?


問題のほうは原子一個だしこんなもんかと思うことにしました。
236ご冗談でしょう?名無しさん:2009/06/26(金) 17:23:41 ID:???
>>235
必要エネルギーはイオン化エネルギーのほうであって光のエネルギーのほうではない
237ご冗談でしょう?名無しさん:2009/06/26(金) 17:26:46 ID:???
>>129

ああそっか!もうすでにそんなこともできたんでした!
ありがとうございます。
238ご冗談でしょう?名無しさん:2009/06/26(金) 18:47:32 ID:???
370nmっていえば可視光域ぎりぎりぐらいの短波長だから、エネルギー高い
んだけどな。それで「しまくり」でも普通じゃん。
>>233
239ご冗談でしょう?名無しさん:2009/06/26(金) 19:39:54 ID:???
>>233
水素原子のイオン化エネルギーは 14eV だから、SI だと
2*10^(-18)J
くらい

相当する光の波長は 90nm (リュードベリ定数の逆数)
240ご冗談でしょう?名無しさん:2009/06/26(金) 20:55:49 ID:???
>>239
370nm はバルマー系列だろう
241ご冗談でしょう?名無しさん:2009/06/26(金) 23:15:49 ID:???
電磁波の力Fを知りたいんですがどう計算するのでしょうか?
例えば1THzの赤外線で振幅はええっと
赤外線他電磁波の振幅って単位は何でしたっけ?
すいません質問が複数になってしまいました。
電磁波の力Fの求め方と可視光や赤外線の振幅の呼び名です。
それはプランク定数hの整数倍ですか。そうすると周波数は単位時間あたりの
エネルギーで力Fということかな・・・すいません助力ください。
242ご冗談でしょう?名無しさん:2009/06/26(金) 23:17:40 ID:???
>電磁波の力
がそもそも意味不明
243ご冗談でしょう?名無しさん:2009/06/26(金) 23:18:13 ID:???
>>241
ちゃんと高校の授業受けたほうがいいよ
244ご冗談でしょう?名無しさん:2009/06/26(金) 23:25:46 ID:???
宇宙空間で電磁波を当てたら加速しますよね?
エネルギーはhνってのはわかるんですが
周波数と力の関係があはふやなまま生きてきました。
245ご冗談でしょう?名無しさん:2009/06/26(金) 23:30:12 ID:???
>>244
電磁波が「何」に及ぼす力なのかによって、答が全然違う。
電場と磁場が電荷に及ぼす力の式使うだけのこと。

246ご冗談でしょう?名無しさん:2009/06/26(金) 23:32:42 ID:???
>>245
例えば赤外線1テラヘルツが質量1gを押す力です。
247ご冗談でしょう?名無しさん:2009/06/26(金) 23:40:05 ID:???
>>246
0
248ご冗談でしょう?名無しさん:2009/06/26(金) 23:43:47 ID:???
>>247
地球のような重力と空気抵抗と摩擦のある場所でなく
宇宙空間でも加速されませんか?
249239:2009/06/26(金) 23:59:37 ID:???
>>240
別に 370nm については自分は何も書いてない
そもそも、質問者はイオン化エネルギーを出そうとして、
間違って 370nm と計算したんじゃないのか?
250ご冗談でしょう?名無しさん:2009/06/27(土) 00:09:23 ID:???
>>248
バカなこと言ってないで早く寝ろよ
251ご冗談でしょう?名無しさん:2009/06/27(土) 00:31:10 ID:???
他板から誘導されて来ました

量子論では無が無ではない、という事で、真空状態の宇宙でも、衛星が半永久的に公転しているのは、エネルギー保存の法則に反しないですか?
252ご冗談でしょう?名無しさん:2009/06/27(土) 00:37:47 ID:???
>>251
摩擦が無ければ動き続けてもエネルギーは失われない
253ご冗談でしょう?名無しさん:2009/06/27(土) 00:51:36 ID:???
>>251
エネルギーが保存してるから半永久的に公転してるんだろ
254ご冗談でしょう?名無しさん:2009/06/27(土) 00:53:53 ID:???
>>251
> 量子論では無が無ではない、という事で、



> 真空状態の宇宙でも、衛星が半永久的に公転しているのは、エネルギー保存の法則に反しないですか?

にどういう関係があると考えているの?
255252:2009/06/27(土) 01:03:39 ID:???
>>251
ちょっと言い方がまずかった
無が無でないといっても真空では運動に対するいわゆる摩擦はないからエネルギーは散逸しない
256ご冗談でしょう?名無しさん:2009/06/27(土) 01:06:17 ID:UuJC55js
量子力学の勉強を始めたのですが、この初めの原則にE=hω/2π がありますよね
しかし調和振動子のエネルギーが E=(n+1/2)hω/2π
となっているのは、波動関数のωとはまた別のものだからなのでしょうか?
257ご冗談でしょう?名無しさん:2009/06/27(土) 01:10:40 ID:???
>255
なるほど、分かりました
258ご冗談でしょう?名無しさん:2009/06/27(土) 01:13:59 ID:???
>>256
エネルギーEの状態の時間発展はexp(-iEt/h_bar)で与えられてこの振動数が下記のω。
>量子力学の勉強を始めたのですが、この初めの原則にE=hω/2π がありますよね
調和振動子のエネルギーの表式に出てくるωはポテンシャルωx^2/2に出てくるωで
E=hω/2π のωとは全くの別物
259ご冗談でしょう?名無しさん:2009/06/27(土) 01:20:29 ID:???
黒体輻射のエネルギー量子=調和振動子のエネルギー準位差
のことなんじゃなかろうか。
260ご冗談でしょう?名無しさん:2009/06/27(土) 01:40:24 ID:???
>>248
読解力がないにも程があるな。電磁波が力を及ぼすのは電荷に対して。

1gって何が1gなんだよ。電気を持っていないなら0だよ。
261ご冗談でしょう?名無しさん:2009/06/27(土) 01:47:05 ID:???
262ご冗談でしょう?名無しさん:2009/06/27(土) 01:48:32 ID:???
>>248
光が物質に吸収されるとその光の波長に応じた運動量が与えられるから加速はするかもしれない
力は求まらないと思う
263ご冗談でしょう?名無しさん:2009/06/27(土) 01:51:35 ID:???
>>261
放射圧のことか。それなら力は計算できるな。
1フォトンが当たったときのこと考えてたわ
264256:2009/06/27(土) 02:05:10 ID:???
>>258
即レスありがとうございます。助かります。
265ご冗談でしょう?名無しさん:2009/06/27(土) 11:11:23 ID:???
吸収された光が、熱になるのか運動量になるのか光電効果起こすのか・・・、
分からないしなぁ。
266ご冗談でしょう?名無しさん:2009/06/27(土) 11:20:39 ID:???
>>265
それは択一じゃないだろ。運動量には必ずなるんだから。
267ご冗談でしょう?名無しさん:2009/06/27(土) 13:05:10 ID:???
熱って、加熱する光の入射方向にかならず放射するの?知らなかった。
268ご冗談でしょう?名無しさん:2009/06/27(土) 13:11:13 ID:???
>>267はどのレスを誤読してんだ?
269ご冗談でしょう?名無しさん:2009/06/27(土) 13:40:15 ID:keqr0sDr
>>244
しますよ。すり抜けないなら。
どれだけの量の光か書いてないので勝手に光子単位で説明すると
光子が吸収された場合、光子の持っていた運動量分だけ物体は動きを変える
光子が反射された場合、光子と物体の全運動量が保存するように物体は動きを変える

光子の振動数とエネルギーと運動量の関係は自分で調べろ
運動量の変化を力に直したければ単位時間当たりに衝突する
光子の量を適当に設定して力に直せ
270ご冗談でしょう?名無しさん:2009/06/27(土) 13:49:14 ID:6ir0jce6
もの凄く大きい三角形を書いて計ると内角の和は180度になりますか?
271ご冗談でしょう?名無しさん:2009/06/27(土) 13:52:57 ID:???
>>270
ここは物理板です
272ご冗談でしょう?名無しさん:2009/06/27(土) 13:55:15 ID:???
>>270
宇宙の曲率がわかってないので不明。
273ご冗談でしょう?名無しさん:2009/06/27(土) 14:17:01 ID:???
地球の表面に大きな三角形を書くと・・・ってよくあるやつじゃね?
274ご冗談でしょう?名無しさん:2009/06/27(土) 15:13:38 ID:6ir0jce6
>>271
へー、なに板の質問ですか?
>>272
ありがとうございます。
>>273
地球の表面に書いた三角形は三角形じゃありませんよ?wwww
275ご冗談でしょう?名無しさん:2009/06/27(土) 15:25:39 ID:eFfpS66t
ガウスは実際に測定して三角形だと確認しましたがなにか?
276ご冗談でしょう?名無しさん:2009/06/27(土) 15:28:20 ID:6ir0jce6
ガウスは三角形の大きさが小さかったから測定できなかったんですがなにか?
277ご冗談でしょう?名無しさん:2009/06/27(土) 15:31:31 ID:???
数学板に行くべきだと思います
278ご冗談でしょう?名無しさん:2009/06/27(土) 15:38:14 ID:???
わからないことを質問したいんじゃないのならスレ違いでしょう。
279ご冗談でしょう?名無しさん:2009/06/27(土) 16:08:10 ID:???
わからないならわからないと言え
280ご冗談でしょう?名無しさん:2009/06/27(土) 16:12:34 ID:???
物理版としての答えなら>>272で終わりだろ。
281ご冗談でしょう?名無しさん:2009/06/27(土) 17:26:11 ID:???
>>272
話が逆
282ご冗談でしょう?名無しさん:2009/06/27(土) 17:30:30 ID:???
>>281
質問者が納得しているのに、理由も説明せず、茶々を入れるのは馬鹿の証拠のようなものだ。
283ご冗談でしょう?名無しさん:2009/06/27(土) 20:21:49 ID:???
スピン1と1/2の粒子をa,bの角運動量演算子をsaとsbするとsa+sbやsa・sbなどの固有値を考えたりする場合、
スピン1の角運動量演算子の成分は3×3の行列で,1/2の運動量演算子の成分は2×2の行列でなので、
sa+sbやsa・sbは計算出来なくないですか??
計算出来ないけど固有値や固有状態は存在するんですか?
そのへんが分かりません!良かったら教えて下さい。
284ご冗談でしょう?名無しさん:2009/06/27(土) 20:24:34 ID:???
>>283
角運動量の合成とか量子力学の教科書に載ってないか?
285ご冗談でしょう?名無しさん:2009/06/27(土) 21:21:44 ID:???
>>281
残念!だったな。
俺は、4代前から、東京生まれの東京育ちの、正真正銘の日本人だ。
お前こそ、朝鮮で生まれて、朝鮮で育った、正真正銘のチョンだろうが。
286ご冗談でしょう?名無しさん:2009/06/27(土) 21:26:36 ID:???
読点多すぎw
287ご冗談でしょう?名無しさん:2009/06/27(土) 21:42:47 ID:???
それ以前にアンカー先と繋がらなすぎ。誤爆か?
288ご冗談でしょう?名無しさん:2009/06/27(土) 21:52:16 ID:???
江戸っ子が誤爆したということでつね。
289ご冗談でしょう?名無しさん:2009/06/27(土) 22:05:59 ID:ZNVTIv1/
黒/鏡/真空/黒/鏡/真空/黒/鏡/真空/黒/鏡/真空/
と多層に作った断熱ポットのような密閉容器
の中と外では、放置すれば自然に温度差ができるでしょうか?
黒い部分と鏡は密着接触し非常に薄いと仮定します。

何年も、いまのところ誰も答えを示してもらえないので転載です。
290ご冗談でしょう?名無しさん:2009/06/27(土) 22:18:07 ID:???
>>289
できない。
外部から何もしないで温度差が生じる理由がない。

仮に温度差が生じるなら、そこからエネルギーを汲み出せる。
つまり、永久機関。これはムリ。

したがって、温度差が生じるとする仮定が棄却される。
291ご冗談でしょう?名無しさん:2009/06/27(土) 22:18:14 ID:???
黒ってのが意味わからん
292ご冗談でしょう?名無しさん:2009/06/27(土) 22:22:08 ID:???
表が黒で裏が鏡。熱を捕まえるけど逃がさない、一方通行のフィルターみたいなもんを夢想してるんだろう。

双方向なんだけどね。
293ご冗談でしょう?名無しさん:2009/06/27(土) 22:22:20 ID:???
>>289
ただの高級魔法瓶だろ。何が言いたいの?
直射日光に晒したら、当然表面の黒は熱くなって内部と温度差が生じるし
常温で冷暗所においておけば内外の温度変化はない。
一体何が知りたい思考実験なんだ?イミフ
294ご冗談でしょう?名無しさん:2009/06/27(土) 22:25:37 ID:???
太陽光という外部エネルギーを認めてる時点で
永久機関とか関係なくね?
295ご冗談でしょう?名無しさん:2009/06/27(土) 22:27:42 ID:???
>>294 289は太陽光とか言ってないが?
296ご冗談でしょう?名無しさん:2009/06/27(土) 22:28:26 ID:???
これ永久機関のつもりなの?益々イミフ
297ご冗談でしょう?名無しさん:2009/06/27(土) 22:29:42 ID:???
カシミール効果に仕事させれば
298ご冗談でしょう?名無しさん:2009/06/27(土) 22:33:59 ID:???
真空は熱を通さないし、鏡がどんなものかしらんが
赤外線を反射してそれが凹面などで局所集中するならば熱は均一には
ならないような気がする。
熱には方向性がないが電磁波には方向性があり、進行方向を
反射や誘導することは可能という意味ね。
299ご冗談でしょう?名無しさん:2009/06/27(土) 22:35:13 ID:???
>>298
もうちょっと詳しく
300ご冗談でしょう?名無しさん:2009/06/27(土) 22:37:22 ID:???
>の中と外では、放置すれば自然に温度差ができるでしょうか?
どうでもいいが、289は謙虚に断言などしていないのに
断言したように幻想し、俺正義しているDQNがいるのが笑えるw
301ご冗談でしょう?名無しさん:2009/06/27(土) 22:37:40 ID:TQf5b1eH
>>284
乗ってますけど分からないです。すいません。
302ご冗談でしょう?名無しさん:2009/06/27(土) 22:42:11 ID:???
> 断言したように幻想し、俺正義しているDQNがいるのが笑えるw

むしろこれ自体が幻想だな
303ご冗談でしょう?名無しさん:2009/06/27(土) 22:45:48 ID:???
局所温室って話じゃないの?
熱が均衡して一定になるのはマクロ的に見たときだけの話で
局所では熱が逆に力に変化することは実験で証明されているし。
物理法則に違反はしてないだろ。

【熱力学】ぶつけた速度よりも速く跳ね返る“常識覆すボール” 熱力学の根本に迫る
http://gimpo.2ch.net/test/read.cgi/scienceplus/1241043018/l50
304ご冗談でしょう?名無しさん:2009/06/27(土) 22:46:45 ID:???
300が一番痛いな
いったい何が見えているのやら
305ご冗談でしょう?名無しさん:2009/06/27(土) 22:48:17 ID:???
本人が必死になっているのが見える
誰とは言わないが、反感もったやつが本人である
306ご冗談でしょう?名無しさん:2009/06/27(土) 22:49:48 ID:???
>>303
それニュース自体に信用性がない。
熱が力に変化する?
そんなこと言うと304にキチガイ扱いされるよ。
307ご冗談でしょう?名無しさん:2009/06/27(土) 22:51:18 ID:???
なるほど必死だな。
308ご冗談でしょう?名無しさん:2009/06/27(土) 22:52:47 ID:???
どうでもいいことに粘着するな無視できない時点で
二度と来なくていいよ。カエレ!
309ご冗談でしょう?名無しさん:2009/06/27(土) 22:53:21 ID:???
>>289
> 何年も、いまのところ誰も答えを示してもらえないので転載です。

本当に? ここで一瞬で(>>290)答えが示されてるのに?
「自分が思っている答えを誰も示してくれない」じゃないとすれば、これまで質問した場所が悪すぎたんだろうね。
310ご冗談でしょう?名無しさん:2009/06/27(土) 22:54:03 ID:???
>>306
リンク先の話は直観的には「当たり前」な結果だと思うが?
311ご冗談でしょう?名無しさん:2009/06/27(土) 22:55:25 ID:???
>>306
ブラウン運動で推進するのと同じような原理だよ
312ご冗談でしょう?名無しさん:2009/06/27(土) 22:56:11 ID:ZNVTIv1/
>>298
ありがとうです。質問を変えて他で聞いて見ます。
313ご冗談でしょう?名無しさん:2009/06/27(土) 22:58:38 ID:???
どうやらここでも気に入る答えは得られなかったらしいw
314ご冗談でしょう?名無しさん:2009/06/27(土) 23:00:11 ID:???
↓粘着君の最後の教育的指導
315ご冗談でしょう?名無しさん:2009/06/27(土) 23:01:24 ID:???
>>298
>真空は熱を通さないし、鏡がどんなものかしらんが
>赤外線を反射してそれが凹面などで局所集中するならば熱は均一には
>ならないような気がする。
>熱には方向性がないが電磁波には方向性があり、進行方向を
>反射や誘導することは可能という意味ね。
つまり熱を自然に温度差を作ることが可能ってこと?
316ご冗談でしょう?名無しさん:2009/06/27(土) 23:05:21 ID:???
うーむ。つまり鏡を適当に配置すれば自然に温度差が生じるので(第二種)永久機関が作れると
317ご冗談でしょう?名無しさん:2009/06/27(土) 23:06:51 ID:???
>>315
それができなければ虫眼鏡で極所を焼き焦がすことは無理
類似した密度での熱均衡で考えるから温度差は発生しないと
思い込むだけだろう。輻射熱が全て全方向へ均衡に広がる
としか判断できないだけでしょうね。
318ご冗談でしょう?名無しさん:2009/06/27(土) 23:11:14 ID:???
>>317
ネタにしてもこれはひでぇ
319ご冗談でしょう?名無しさん:2009/06/27(土) 23:13:02 ID:???
>>318
君の発言が消防並みに見えるのは漏れだけ?
320ご冗談でしょう?名無しさん:2009/06/27(土) 23:15:18 ID:???
最初に質問落とした子よりも香ばしいのが涌いてるね。

自分の好きなように条件変えてるし。
321ご冗談でしょう?名無しさん:2009/06/27(土) 23:15:35 ID:???
>>317
虫眼鏡でいくら光を集めても、光源より温度が高くならないもんなんだが。
322ご冗談でしょう?名無しさん:2009/06/27(土) 23:16:13 ID:???
>漏れだけ?

外部の熱源持ち出したらダメだってことがわからない奴は君だけであって欲しいな。
323ご冗談でしょう?名無しさん:2009/06/27(土) 23:16:26 ID:???
熱のバランスが崩れるとして、
輻射でその状態へ至るには、輻射熱から考えて
とてつもなく時間がかかるはずだろう。
そのできた温度差は伝送路で減衰するだろうから
計れるほどのエネルギーが発生することはないと思われる。
324ご冗談でしょう?名無しさん:2009/06/27(土) 23:17:56 ID:???
>>321
> 虫眼鏡でいくら光を集めても、光源より温度が高くならないもんなんだが。
痛いな、それは間違え。
温度はどんどん溜まる。
光源と同じ熱量の放射密度までしか高くならないと言い直せ
はずかしすぎるぞ
325ご冗談でしょう?名無しさん:2009/06/27(土) 23:18:53 ID:???
>>324
光学系の相反定理とか、輝度不変の法則とか知っている??
326ご冗談でしょう?名無しさん:2009/06/27(土) 23:19:38 ID:???
>>324
> 温度はどんどん溜まる。

うわあ、こいつ本物だ。
327ご冗談でしょう?名無しさん:2009/06/27(土) 23:19:45 ID:???
>>325
熱と輝度を混同するDQN?
328ご冗談でしょう?名無しさん:2009/06/27(土) 23:20:33 ID:???
>>324
温度が違って熱量の放射密度が同じってどういう状況?
329ご冗談でしょう?名無しさん:2009/06/27(土) 23:21:12 ID:???
>>324
たぶんこの人は、虫眼鏡で温められた部分が逆に熱放射するということを
無視している。
330ご冗談でしょう?名無しさん:2009/06/27(土) 23:21:17 ID:???
太陽光でマグネシュームをプラズマ化させるときは3万度ぐらいの
温度までになるんですが。
331ご冗談でしょう?名無しさん:2009/06/27(土) 23:21:34 ID:???
>>324
> > 虫眼鏡でいくら光を集めても、光源より温度が高くならないもんなんだが。
> 痛いな、それは間違え。

キミの方が間違えてるよ。引用元が正しい。
332ご冗談でしょう?名無しさん:2009/06/27(土) 23:23:32 ID:???
>>331
量子力学を学んできたほうがいい。
333ご冗談でしょう?名無しさん:2009/06/27(土) 23:24:27 ID:???
>>330
素の太陽光で??太陽光で励起したレーザを使うとかいう話じゃなくて??

それはすごいな。いつのまにか熱力学第2法則が破れていたのか。
334ご冗談でしょう?名無しさん:2009/06/27(土) 23:25:21 ID:???
>>332
目下の話題に量子力学を持ち込む必要は皆無だけど、それでごまかせるとでも思ってんの?
335ご冗談でしょう?名無しさん:2009/06/27(土) 23:26:00 ID:???
虫眼鏡を使うだけで低温から高温へ熱が移動できるって、

「なんかおかしい」

と思わないのかね??
336ご冗談でしょう?名無しさん:2009/06/27(土) 23:27:15 ID:???
>>330
そいつはムリだろ。別の過程がなにか入ってるよ。輻射だけでは絶対不可能。
337ご冗談でしょう?名無しさん:2009/06/27(土) 23:30:53 ID:???
普通に太陽光レーザーで数万度まで集光させることは可能です
単純な屈折だけとしか考えられないのかな。
338ご冗談でしょう?名無しさん:2009/06/27(土) 23:32:52 ID:???
外部から加熱しつづけて外に熱を逃がさないものなら
熱は上がり続ける。もし上がらないのなら外部から
供給されたエネルギーは消滅することになる。
339ご冗談でしょう?名無しさん:2009/06/27(土) 23:33:19 ID:???
>>337
そんなんできて当たり前だ。目下のやりとりにノイズを入れるな。
340ご冗談でしょう?名無しさん:2009/06/27(土) 23:33:59 ID:???
> 外部から加熱しつづけて外に熱を逃がさないものなら

外に熱を逃がさないものを加熱することは出来ません
341ご冗談でしょう?名無しさん:2009/06/27(土) 23:34:38 ID:???
>>338 底なしのバカだ。輻射は双方向だ。
342ご冗談でしょう?名無しさん:2009/06/27(土) 23:35:05 ID:???
>>338
それは正しい。

が、今話題になっている、虫眼鏡で何かを温めるという状況は
「外に熱を逃がさないもの」という条件に反する。
343ご冗談でしょう?名無しさん:2009/06/27(土) 23:37:27 ID:???
342だが、すまん、確かに>>340>>341の言う通り、そもそも外に熱を逃がさない
ものは温めることすらできんな。

というわけで、>>342の2行目の「それは正しい。」は撤回する。

344ご冗談でしょう?名無しさん:2009/06/27(土) 23:42:00 ID:???
>>342
外に熱が逃がせない仕組みは存在しないと証明してください。
どんな方法にしろ例え励起にしろ仕組みがあれば熱は溜まります。
345ご冗談でしょう?名無しさん:2009/06/27(土) 23:43:13 ID:???
>>344
すごいところをwwww
346ご冗談でしょう?名無しさん:2009/06/27(土) 23:43:28 ID:???
悪魔の証明ktkr
347ご冗談でしょう?名無しさん:2009/06/27(土) 23:44:04 ID:???
>>344
つ熱力学第2法則

「他に何の影響も残さず、低温の物体から高温の物体に熱を移動させることは
不可能である」

 あなたの主張は、多くの経験から得られたこの法則に喧嘩を売ってます。
348ご冗談でしょう?名無しさん:2009/06/27(土) 23:44:14 ID:???
動揺のあまりアンカーまで間違えたか
349ご冗談でしょう?名無しさん:2009/06/27(土) 23:44:40 ID:???
>>344
確かに6000℃の熱エネルギーから他のエネルギーに変換できない
ならば熱は保存されないが、変換する前提があるならそういえるな。
350ご冗談でしょう?名無しさん:2009/06/27(土) 23:44:47 ID:???
次は「熱力学第2法則を証明してください」とか言い出しそうだが・・・・
351ご冗談でしょう?名無しさん:2009/06/27(土) 23:45:20 ID:???
>>349
太陽光レーザーで普通に数万度にまであげている。
352ご冗談でしょう?名無しさん:2009/06/27(土) 23:46:33 ID:???
>>351
なるほどそういう裏技があるのか
353ご冗談でしょう?名無しさん:2009/06/27(土) 23:47:18 ID:???
>>351
そうだねwww
太陽熱をエネルギー源として発生する雷も数万度の温度に達するよねwwww
354ご冗談でしょう?名無しさん:2009/06/27(土) 23:47:18 ID:???
外に熱が逃がせない仕組み・・・マックスウェルの悪魔ですか。
355ご冗談でしょう?名無しさん:2009/06/27(土) 23:48:13 ID:???
>>353
頭が固いのはそんなことすら、嘘だと言い張る。
356ご冗談でしょう?名無しさん:2009/06/27(土) 23:49:26 ID:???
どっかにいたか?「雷は嘘だ」とか言ってた奴??>>355

「太陽光レーザが嘘だ」と言った奴もいない。
357ご冗談でしょう?名無しさん:2009/06/27(土) 23:49:46 ID:???
そもそも太陽光が反射され輻射され物質と関係を持ったときに
太陽光のままの性質だと勝手に思い込むのがバカの始まり
358ご冗談でしょう?名無しさん:2009/06/27(土) 23:50:29 ID:???
>>355
へ? 雷が数万度に達することを? それとも雷のエネルギー源が太陽熱だってことを?
誰がそんなこと否定するの?
359ご冗談でしょう?名無しさん:2009/06/27(土) 23:51:33 ID:REk6JImd
高校生で卒論的なる探究活動があるのですが、発電系で何か案がほしいです。
360ご冗談でしょう?名無しさん:2009/06/27(土) 23:51:41 ID:???
>>357
誰かが虫眼鏡でどんどん高温が作れるとか言い出すからこんな騒ぎに
なっちゃっているんだが。
つまり話の発端は太陽光だけの話。レーザも雷も出てこない。

361ご冗談でしょう?名無しさん:2009/06/27(土) 23:52:05 ID:???
つまり励起や電気等に変化したりして性質が変われば
最終的に熱が外に逃げずに貯める仕組みは存在しえる。

太陽電池で発電したのをバッテリーで貯めて
バッテリーをショートさせればいいだけですね。
わかります簡単ですね。
362ご冗談でしょう?名無しさん:2009/06/27(土) 23:52:59 ID:???
>>360
発端は太陽光じゃなくて温度差が作れるか、話を摩り替えるな。
363ご冗談でしょう?名無しさん:2009/06/27(土) 23:54:58 ID:???
>>362
まぁ大本はそこだが、どっちの問題も「熱力学第2法則無視すんじゃねぇ」
という問題である点では一緒じゃん。
364ご冗談でしょう?名無しさん:2009/06/27(土) 23:56:39 ID:???
川の流れで水車を回して、その力で川の水を水源より高い場所に揚げることは可能だよね。

川の水の「ごく一部」であれば。
365ご冗談でしょう?名無しさん:2009/06/27(土) 23:57:41 ID:???
>>363
そこに298が条件を追加した話をし、条件が違うなら熱力学だけの話ではない
という流れだろう。熱力学だけの話ではなくなったのに熱力学だけの
話に固執しているのは明白だ。最初の話を否定するのはいいが、
その後の流れは条件を限定はしていない、お前の思い込み。
366ご冗談でしょう?名無しさん:2009/06/27(土) 23:59:10 ID:???
>>364
条件が違えば、海の水が雨がとなり可能となる。
条件を追加した時点で、追加する前と同じだと主張するのは池沼でしかない
367ご冗談でしょう?名無しさん:2009/06/28(日) 00:00:30 ID:???
そうだよな。科学なら条件を厳密に限定するのは必須条件であり
それを定めずに何を言っても空論でしかない。
368ご冗談でしょう?名無しさん:2009/06/28(日) 00:01:15 ID:???
>>365
熱力学第2法則に反するからダメ、と主張したら、熱力学に固執している
ことになるの???

熱力学だけの話じゃなくなったら、第2法則は効力を失うとでも??
369KY:2009/06/28(日) 00:01:48 ID:???
370ご冗談でしょう?名無しさん:2009/06/28(日) 00:03:09 ID:???
言葉のすれ違い、日本語でおk
371ご冗談でしょう?名無しさん:2009/06/28(日) 00:03:29 ID:???
>>367
とりあえず、最初の変な魔法瓶の話も、虫眼鏡の話も、十分問題を限定した
話だから、「温度差は生じない」とか「どこまでも高温にはならない」と
結論することは空論ではないと思う。
372ご冗談でしょう?名無しさん:2009/06/28(日) 00:04:59 ID:???
「太陽光レーザ太陽光レーザ」とか言っている奴は空論だが。
373ご冗談でしょう?名無しさん:2009/06/28(日) 00:05:07 ID:???
俺定義=俺正義=俺限定

俺が全て限定したと言った(はず=脳内)だから俺に従え
374ご冗談でしょう?名無しさん:2009/06/28(日) 00:05:55 ID:???
>>373
皮肉はやめとけ
375ご冗談でしょう?名無しさん:2009/06/28(日) 00:07:00 ID:???
>>366-367
話の発端は>>289の魔法瓶だろ。

>>289の魔法瓶で自然に温度差が生じないなら虫眼鏡で高温を作ることも出来ない、と>>317が言い出した
ことが発端。

つまり、>>317の主張は「魔法瓶で温度差を作ることが出来る」であって、その根拠として虫眼鏡を持ち出
したのだろ。

だから、どう条件を変えてもいいけど、それが「魔法瓶で温度差を作れる」の根拠にならないのなら無意味。

レーザーを持ち出しても電池やら雷やらを持ち出してもいいけど、それで魔法瓶の中に温度差が作れると思うのかな。
376これはどの板でも共通:2009/06/28(日) 00:07:02 ID:???
いいから無視すればいいじゃん
無視できない奴=荒らし
377ご冗談でしょう?名無しさん:2009/06/28(日) 00:08:23 ID:???
>>376
痛いのは相手にしないことにした。
378ご冗談でしょう?名無しさん:2009/06/28(日) 00:10:38 ID:???
>>375
宇宙線などの力で電気を貯めることは理論的に可能。

例えば電気石などで自然に

379ご冗談でしょう?名無しさん:2009/06/28(日) 00:13:21 ID:???
>>378
トルマリン?
トルマリンなら赤外線でも電気になるよ
380ご冗談でしょう?名無しさん:2009/06/28(日) 00:14:29 ID:???
なんで>>375を読んだ後で宇宙線だのトルマリンだのの話ができるんだか、、、
381ご冗談でしょう?名無しさん:2009/06/28(日) 00:14:41 ID:???
>>378
それのどこが>>375への返答になってるの?
それで「魔法瓶の中に温度差を作れる」と思うのか?
382ご冗談でしょう?名無しさん:2009/06/28(日) 00:15:14 ID:???
>>359
ゼーベック効果でもやっとけ。
383ご冗談でしょう?名無しさん:2009/06/28(日) 00:15:37 ID:???
もしかして「外部の熱源 (宇宙線とかも当然含まれる)」を導入したらダメ、ってことすらわかってないのか。
384ご冗談でしょう?名無しさん:2009/06/28(日) 00:15:56 ID:???
もっと冷静になるのをお勧めします
385ご冗談でしょう?名無しさん:2009/06/28(日) 00:16:54 ID:???
っていうより、こりゃ総員撤退!の時間じゃね?
386ご冗談でしょう?名無しさん:2009/06/28(日) 00:16:56 ID:???
>>384
余計なこと言うな
387ご冗談でしょう?名無しさん:2009/06/28(日) 00:22:11 ID:c5vhvYck
熱からエネルギーを確率的に取り出せるのは>>303でファイナルアンサーである。
終わり
388ご冗談でしょう?名無しさん:2009/06/28(日) 00:24:00 ID:???
>>387
嘘クセェ
389ご冗談でしょう?名無しさん:2009/06/28(日) 00:25:16 ID:???
>>387
熱からはエネルギーは取り出せません

それは熱力学ではできないことになっている。
390ご冗談でしょう?名無しさん:2009/06/28(日) 00:26:16 ID:???
>>389
頭固すぎるよ。教科書に書いてあるのを鵜呑みで
勝手に解釈しちゃだめ。
391ご冗談でしょう?名無しさん:2009/06/28(日) 00:28:16 ID:???
>熱力学第二法則では、外から手を加えずに熱が運動エネルギーなどの仕事に変わることを禁じている。
>熱から仕事が取り出せれば永久機関も実現できるからだ。
>>390
ちゃんと学校でた?熱から運動エネルギーは取り出せないと
教科書には載っているんだよ

「熱力学第二法則」を最初から勉強してこような。
392ご冗談でしょう?名無しさん:2009/06/28(日) 00:28:16 ID:???
>>303の話が仮に事実だとしても、「魔法瓶で…」という話とは無関係な話だな。
393ご冗談でしょう?名無しさん:2009/06/28(日) 00:28:18 ID:???
部分で見るとエネルギーが取り出せてるが、そもそも熱ってのは全体平均の物差しだしな
394ご冗談でしょう?名無しさん:2009/06/28(日) 00:29:30 ID:???
>>393
そのとおり、極所での話しと全体平均を区別できないようになった奴はシネ。
395ご冗談でしょう?名無しさん:2009/06/28(日) 00:31:05 ID:???
>>390
教科書を疑うのは結構だが、根拠のない話をされても困る。
レーザーにしても電池・雷・宇宙線(?)にしても、あるいは>>303にしても、
熱力学第二法則を破るものではない (>>303についてはそもそも疑わしいが)
396ご冗談でしょう?名無しさん:2009/06/28(日) 00:31:54 ID:???
>>392
>303の話が仮に事実だとしても、
http://gimpo.2ch.net/test/read.cgi/scienceplus/1241043018/l50
これは仮ではなく、実証された事実。頭わるいぞ。
397ご冗談でしょう?名無しさん:2009/06/28(日) 00:33:15 ID:???
>>390は相間と同じ臭いがするから気をつけた方がいい。
398ご冗談でしょう?名無しさん:2009/06/28(日) 00:33:26 ID:???
>>395
たぶん394は君にw
399ご冗談でしょう?名無しさん:2009/06/28(日) 00:34:27 ID:???
>>396
> 実証された事実

現実に観測されたわけでもなく、一部の研究者が「シミュレーションで」
見出した段階で「実証された事実」かよ

事実だとしても無関係なのに変わりはないしな。
400ご冗談でしょう?名無しさん:2009/06/28(日) 00:34:51 ID:???
http://gimpo.2ch.net/test/read.cgi/scienceplus/1241043018/493
これよめよ。303のURLが信じられない奴君。
401ご冗談でしょう?名無しさん:2009/06/28(日) 00:37:21 ID:???
まだバカが集まっているのか?
402ご冗談でしょう?名無しさん:2009/06/28(日) 00:37:38 ID:???
>>303の話自体は不思議じゃないし、信じるよ。これで熱力学第2法則破れた、
なんてことは全然思わないが。
403ご冗談でしょう?名無しさん:2009/06/28(日) 00:37:53 ID:???
信じる信じないの問題じゃないだろう…
404ご冗談でしょう?名無しさん:2009/06/28(日) 00:39:54 ID:???
この中には、熱力学第2法則すら「信じる信じないの問題」だと思っている
奴がいそうだな。。。。
405ご冗談でしょう?名無しさん:2009/06/28(日) 00:40:11 ID:???
>>400
それが何か? >>303が事実かどうかとなんら関係ないな。
>>303のようなことは「あってもおかしくない」とは思うが、現実に観測されるまでは
「実証された事実」とはとても言えないだろ。
406ご冗談でしょう?名無しさん:2009/06/28(日) 00:43:11 ID:???
質問です。
頂点を下に向け、軸のまわりに一定の角速度ωで回転するなめらかな放物線に束縛された質点の運動を調べよ。

ヒントに「仮想的な力として遠心力を考える」とあるのですが分かりません。よろしくお願いします。
407ご冗談でしょう?名無しさん:2009/06/28(日) 00:48:43 ID:???
>>406
「分かりません」って「遠心力を知りません」って意味か?
「遠心力は知っているけどどう使っていいかわかりません」って意味か?
408ご冗談でしょう?名無しさん:2009/06/28(日) 00:51:52 ID:???
>>407
遠心力はわかるのですが、式の立て方がよく分かりません・・・
409ご冗談でしょう?名無しさん:2009/06/28(日) 00:52:16 ID:KM/UFOr8
>>404
物理法則は本質的には「信じる信じないの問題」だよ。
というのは、結局、現象の解釈の問題だからね。
どの理論を信じるかは、普通、思考の経済学に従って、
仮定が少ない物が選ばれ「定説」とされ、教科書に
載せられているに過ぎない。しかし、同程度に正確に
現象を記述できる理論は、どちらが正しいとは言えな
いし、全ての理論は便宜的な仮説に過ぎない。仮説を
信じるのは、最終的には、個々人の信仰の問題。
410ご冗談でしょう?名無しさん:2009/06/28(日) 00:54:16 ID:c5vhvYck
>>409
UFOのIDキターオカルトオカルト
411ご冗談でしょう?名無しさん:2009/06/28(日) 00:58:01 ID:???
>>408
x軸とy軸で垂直抗力に気をつけながら運動方程式を立てればいいと思うが

しかしその放物線の係数って任意?
412ご冗談でしょう?名無しさん:2009/06/28(日) 00:59:24 ID:KM/UFOr8
>>410
>UFOのIDキター
ワーーイ、ウレチーーーー!
413ご冗談でしょう?名無しさん:2009/06/28(日) 01:08:07 ID:???
誰が何と言うと、海洋温度差発電は第2種永久機関だ!
414ご冗談でしょう?名無しさん:2009/06/28(日) 01:09:23 ID:???
誰が何といおうがビックバン宇宙は、永久機関である。
外部からエネルギーは一切補充されていないが動き続ける。
415ご冗談でしょう?名無しさん:2009/06/28(日) 01:10:12 ID:???
>>409
「熱力学第2法則」に「どちらが正しいとは言えない」と言えるほどの
対抗馬あるなら教えてくれ。

もしないんなら、今はそういう倫理社会の先生が言うような話はいらない。
物理には役に立たん。
416ご冗談でしょう?名無しさん:2009/06/28(日) 01:11:32 ID:???
>>414
永久機関と呼ぶためには、「エネルギーを取り出されつつも動く」って
のが大事だが、宇宙はいったいどこにエネルギーが取り出されているの?
417ご冗談でしょう?名無しさん:2009/06/28(日) 01:12:58 ID:???
>>416
無からエネルギーが生まれた時点で、取り出し続けても

元は無だろ?つまり無から無より小さくなるようにエネルギーは減らない。
418ご冗談でしょう?名無しさん:2009/06/28(日) 01:16:01 ID:???
>>417
 エネルギーが保存されない世界の話をしているわけね。じゃあそりゃ永久機関
だってなんでもありだわ。

 実際の宇宙論では、宇宙全体のエネルギーちゃんと保存しているから、
あなたの考えは机上の空論以下だけどね。
419ご冗談でしょう?名無しさん:2009/06/28(日) 01:16:19 ID:???
>>417
無からエネルギーが生まれたと定義している時点で
熱力学などはそれに通じるわけがない416はその根本すらわかっていない。

ビックバンが生まれる瞬間は物理法則は通じないとされている
ビックバンの定義では最初の段階では熱すらないのだから。

つまりビックバンという例外を議論してもまったく無意味である。
それができるなら、世界一の天才か、キチガイのどちらか。
420ご冗談でしょう?名無しさん:2009/06/28(日) 01:17:55 ID:???
>>418
残念ながら宇宙論はまだまだ1つにはまとまっていない。
派生の理論や勝手な解釈の仮説ばかりで、統一などまだまだ先の話
ということすら知らないようだな。
421ご冗談でしょう?名無しさん:2009/06/28(日) 01:19:45 ID:???
>>420
で、その中にエネルギー保存則が成立しないのはいくつあるの??

とりあえず一般相対論的宇宙論では保存するエネルギーを定義可能だよね。
422ご冗談でしょう?名無しさん:2009/06/28(日) 01:20:46 ID:???
果てしなく話をそらしている奴がいるな。

魔法瓶は温度差は作らないってのはもうFAってことでいいよな。
423ご冗談でしょう?名無しさん:2009/06/28(日) 01:21:04 ID:???
>>421
重力と電磁気の区別がない状態でエネルギー保存法則自体を語るのは
変じゃないの?
424ご冗談でしょう?名無しさん:2009/06/28(日) 01:23:11 ID:???
>>423
お前は何を言っているんだ。
425423:2009/06/28(日) 01:23:26 ID:???

http://homepage2.nifty.com/m-traveller/souteni.gif
この話をしているんだ。
426ご冗談でしょう?名無しさん:2009/06/28(日) 01:24:34 ID:???
で、なんで変なの??

重力と電磁気が区別なくなると、エネルギーが定義できなくなるの??

なんで???>>425
427ご冗談でしょう?名無しさん:2009/06/28(日) 01:25:15 ID:???
>>426
区別がなければ方程式すら成立しないだろ。
428ご冗談でしょう?名無しさん:2009/06/28(日) 01:25:59 ID:???
>>426
おまえは大統一理論が完成したとでもいいたいのか?
429ご冗談でしょう?名無しさん:2009/06/28(日) 01:26:37 ID:???
>>427
その絵を出すからには、統一理論があるって前提だろ。統一理論の方程式
があって、その方程式から時間並進の生成子作れば、エネルギーは定義
できるじゃん。
430ご冗談でしょう?名無しさん:2009/06/28(日) 01:27:30 ID:???
>>428
大統一理論は重力と電磁力は統一してないけど??
431ご冗談でしょう?名無しさん:2009/06/28(日) 01:27:45 ID:???
>>429
最初には重力しかない。故にエネルギー=重力で
それ以外はない。ごまかすな。
432ご冗談でしょう?名無しさん:2009/06/28(日) 01:29:01 ID:???
>>431
それなら、エネルギーは定義できているだろう?

なんで「エネルギーが定義できない」と言い切ることができるのかがわからん。

系に時間発展の演算子があれば、エネルギーは定義できる。

統一理論ができたとして、その理論に時間発展の演算子がないとは、
とても思えない。
433ご冗談でしょう?名無しさん:2009/06/28(日) 01:29:20 ID:???
最先端科学の仮説ですら、詳細が分からないとされているのに
力の関係が分かったというDQNがいるスレはここですか?
434ご冗談でしょう?名無しさん:2009/06/28(日) 01:34:08 ID:???
今北俺のために話の流れを産業でまとめてくれ
435ご冗談でしょう?名無しさん:2009/06/28(日) 01:42:10 ID:???
ビックバンの話を天才学者以外が語っても、
DQNでしかないのは明白。先端学者が語る物語を信じるだけでいい。

それを表現するときはソース必須な。
~~~~~~~~~~~~~~~~~~~~~~~~~~~~~~~~~~~
俺理論や俺定義など生ゴミに湧くウジムシと同じさ。
436ご冗談でしょう?名無しさん:2009/06/28(日) 07:01:35 ID:???
>>413 違うよ。太陽からエネルギーが供給されてるからね。

水力発電も一緒だ。
437ご冗談でしょう?名無しさん:2009/06/28(日) 08:04:07 ID:???
>>434
熱力学を良く理解してない奴らが熱力学について語っている。以上。
438ご冗談でしょう?名無しさん:2009/06/28(日) 08:35:34 ID:kLlAyICf
電気回路の電力計算で、無効電力は遅れ力率を正にするか、進み力率を正にするかで符号が変わってしまいますが、院試などでの解答方法では『ただし遅れ力率を正とする』などと事前に断っておけばどちらで回答してもよいのでしょうか?
439ご冗談でしょう?名無しさん:2009/06/28(日) 11:38:38 ID:???
どうせ受からないからどっちでもいいよ。
440ご冗談でしょう?名無しさん:2009/06/28(日) 12:27:15 ID:???
>>283
固有値は存在するよ。固有状態も!
441ご冗談でしょう?名無しさん:2009/06/28(日) 12:29:09 ID:???
442ご冗談でしょう?名無しさん:2009/06/28(日) 12:42:19 ID:???
>>440
どうしてですか??
443ご冗談でしょう?名無しさん:2009/06/28(日) 12:51:35 ID:???
ごめん間違えてた
444ご冗談でしょう?名無しさん:2009/06/28(日) 13:13:38 ID:kLlAyICf
>>439(`ω´)
445ご冗談でしょう?名無しさん:2009/06/28(日) 13:34:39 ID:???
盛り上がってる中すみません。
地球は自転しています。
地球の北緯60゚ところの長さは赤道の半分ですよね?
つまり赤道では40000kmが北緯60゚では20000kmの距離が自転として毎日移動してる訳ですが(公転を除くと・・・)
なんか慣性の法則とか、赤道→北緯60゚の地点に移動したときに成り立つのはどうしてですか?
446ご冗談でしょう?名無しさん:2009/06/28(日) 13:42:21 ID:???
>>445
せっかくの長文だが肝心の質問部分がちょっと曖昧で分からん。
赤道の方が自転の遠心力は大きいよ。だから体重も少し軽くなる。
そういう事聞きたいんじゃなくて?
せっかく前半4行が具体的・定量的なのに次の行に繋がらんと思う。
447ご冗談でしょう?名無しさん:2009/06/28(日) 13:55:44 ID:???
>>445
自転の角速度が小さく従ってその加速度も小さく、地表の動きがほとんど等速直線運動とみなせるから。
448ご冗談でしょう?名無しさん:2009/06/28(日) 14:01:39 ID:???
>>447
その論調でまさか、赤道で慣性の法則成り立たないとでも教える気か?
(厳密には)どっちでも成り立たない。or(誤差の範囲で)どっちでも成り立つ、
と言わないとまずいでしょ
449ご冗談でしょう?名無しさん:2009/06/28(日) 14:03:23 ID:???
俺が言いたいのは、結局北緯何度とかいう問題と質問の慣性の法則との繋がりがイマイチ
450ご冗談でしょう?名無しさん:2009/06/28(日) 14:04:18 ID:???
>>447-448
北緯60゚でも赤道でも自転の角速度は同じ。
451ご冗談でしょう?名無しさん:2009/06/28(日) 14:47:32 ID:???
この問題の(1)について教えて下さい。
http://up2.viploader.net/pic/src/viploader1066267.jpg

(a)はm(dv/dt) = -mg-kmvだと思います。
(b)でこれをtで積分するとどうなりますか?
452ご冗談でしょう?名無しさん:2009/06/28(日) 14:57:26 ID:???
a = -g-kv
dv/dt = -g-kv

dt=-dv/(g+v)
∫dt=-∫{1/(g+v)}dv
453ご冗談でしょう?名無しさん:2009/06/28(日) 14:59:55 ID:???
これはひどい
454ご冗談でしょう?名無しさん:2009/06/28(日) 15:00:41 ID:???
もとい、これもひどい
455ご冗談でしょう?名無しさん:2009/06/28(日) 15:03:44 ID:???
>>452
書いてあることは正しいのに書き方だけでここまで馬鹿っぽく見えるものなんだな
456ご冗談でしょう?名無しさん:2009/06/28(日) 15:20:37 ID:???
>>452
その式だと、重力mg と同じ方向に 抵抗kmv が働くことになって変だよ。
457ご冗談でしょう?名無しさん:2009/06/28(日) 15:21:49 ID:???
それは451に言ってくれ
458ご冗談でしょう?名無しさん:2009/06/28(日) 15:24:40 ID:???
式を吟味せずに解くバカ。しかも計算間違ってる上に、済んでもいない。
459ご冗談でしょう?名無しさん:2009/06/28(日) 15:43:47 ID:wSUjgoXv
「相補性」って言葉は今でも使われてるの?
460ご冗談でしょう?名無しさん:2009/06/28(日) 15:46:24 ID:???
>>451
問題文の「速度vに比例した抵抗-kmv(k>0)」は速度と抵抗の向き
が逆だと言ってるだけ。この場合速度は下向きだから抵抗の向き
は上向き。
運動方程式は(y軸は鉛直上方が+)

m(dv/dt) = -mg+kmv


・・・>>452さんのような感じで計算してけばおk。
461ご冗談でしょう?名無しさん:2009/06/28(日) 15:51:33 ID:???
>>458
> 計算間違ってる

というと?
462ご冗談でしょう?名無しさん:2009/06/28(日) 15:57:28 ID:???
>>451も俺もそうなんだけど
物理で一番つまずきやすいのって符号だと思うんだよ
速度とか速さとかで混乱する
何かコツとかないかな?
463ご冗談でしょう?名無しさん:2009/06/28(日) 15:58:55 ID:???
kが消えてるし
464ご冗談でしょう?名無しさん:2009/06/28(日) 16:42:04 ID:???
>>460
問題文をよく見ると、「速度vに比例した抵抗-kmv」のvはボールドになって
いて、つまりベクトルなんだ。だから問題文は悪くない。
465ご冗談でしょう?名無しさん:2009/06/28(日) 16:59:28 ID:???
>>456,460
お前ら落ち着け。m(dv/dt) = -mg-kmv で合ってるぞ。

> この場合速度は下向きだから抵抗の向きは上向き。

で鉛直上方を正にとるんだから、

> m(dv/dt) = -mg+kmv

これだと

  v<0 (速度は下向きなんだから)
  ∴ kmv<0  抵抗も下向き

になるぞ。
466451:2009/06/28(日) 18:22:17 ID:???
そうやって積分するんですね。ありがとう
467ご冗談でしょう?名無しさん:2009/06/28(日) 20:03:04 ID:???
ライフルの弾は何で重力に影響されずにまっすぐ飛ぶのですか?
弾速は1km/sとすると、(実際に大体その位だそうです)、単純
に計算すると1km先の標的にたどり着く前に約5mも落ちてしま
うように思えるのですが、重力を計算して撃つという話はあまり
聞かないので疑問に思いました。
468ご冗談でしょう?名無しさん:2009/06/28(日) 20:05:38 ID:???
>>467
重力もコリオリ力も計算して撃ちます。
469ご冗談でしょう?名無しさん:2009/06/28(日) 20:11:44 ID:???
そもそも1km先の標的をねらうって話をあまり聞かない
470ご冗談でしょう?名無しさん:2009/06/28(日) 20:14:26 ID:???
>>467はゴルゴの読み過ぎ
471ご冗談でしょう?名無しさん:2009/06/28(日) 20:14:56 ID:???
>>467
まっすぐ飛ぶのではなく、ちゃんと落ちている。
それを見込んで標的より上を狙うようになっている。
照準装置は銃身の向きと平行でなく、銃身の向きより下を向いている。
それで標的を狙うと、銃身は標的より上を向く事になる。
その向き加減を標的までの距離で調整する。
472ご冗談でしょう?名無しさん:2009/06/28(日) 20:26:52 ID:???
>>467
風速だって考慮するぞ。プライベートライアンって映画を見てみろ。
473ご冗談でしょう?名無しさん:2009/06/28(日) 20:28:52 ID:???
>>467
地球は丸いんだからまっすぐ飛ばすと上に行っちゃうだろ?
地球の丸みに沿って落ちてるんだよ。
474ご冗談でしょう?名無しさん:2009/06/28(日) 20:31:21 ID:???
ヒント
地球の曲率
475ご冗談でしょう?名無しさん:2009/06/28(日) 20:56:27 ID:3AkUSX2x
>>459
「ヱヴァンゲリヲン新劇場版:破」には相補性が出てきたな。
476ご冗談でしょう?名無しさん:2009/06/28(日) 21:02:49 ID:???
スレチ
477ご冗談でしょう?名無しさん:2009/06/28(日) 21:03:54 ID:???
>>473
軌道速度に全然及ばないって。
478ご冗談でしょう?名無しさん:2009/06/28(日) 21:10:23 ID:???
>>467
ジョジョの「ハンティングに行こう」 を読むべき
479ご冗談でしょう?名無しさん:2009/06/28(日) 22:12:41 ID:???
変分法における試行関数とはどのようなものなのでしょうか
作用積分の被積分関数とは違うのですか?
480ご冗談でしょう?名無しさん:2009/06/28(日) 22:32:58 ID:???
0 < x < a の井戸型ポテンシャルの中の波動関数
ψ(x) = C(1-cos(4πx/a))
において、
1)この波動関数を規格化し、規格化因子Cを求めよ
2)規格化された波動関数をエネルギーの固有関数φn(x)=√((2/a)sin(nπx/a))
を使ってψ=Σ[i=1,n]c(i)φ(i)と展開したときのc(i)を求めよ
3)E=E1,E2,E3が測定される確率を求めよ

1)は自力で解いてC=√(2/3a)はでたのですが、
2)以降の解き方が分かりません。よろしくお願いします。
481ご冗談でしょう?名無しさん:2009/06/28(日) 22:39:25 ID:???
sinとcosの直交性を利用
482ご冗談でしょう?名無しさん:2009/06/28(日) 22:49:44 ID:???
>>479
全く異なる

『与えられたHamiltonian Hに対し、固有エネルギーEを最小とするような固有関数ψが「一意的に」存在するという』という定理があるので、
「任意の」波動関数の中で固有エネルギーを最小にするような波動関数を求めてやれば、
定義からしてそれが基底状態の波動関数であることが分かるというのが変分原理

で、その最小となる波動関数を求めるわけだけど、
関数ψ(x)を動かしたときにエネルギーEが最小になる時を考えればよいわけで、
それは経路x(t)を動かしたときに作用Sが最小になるときを考えるのと同様、
変分法を使ってやればよいというだけのこと

変分法自体は、具体的な物理の系とは全く関係ない、純粋に物理数学の問題だよ
483ご冗談でしょう?名無しさん:2009/06/28(日) 23:25:17 ID:???
>>482
細かいところだけど
>『与えられたHamiltonian Hに対し、固有エネルギーEを最小とするような固有関数ψが「一意的に」存在するという』という定理があるので、
基底状態に縮退がなければね
484ご冗談でしょう?名無しさん:2009/06/28(日) 23:43:04 ID:???
>>483
縮退はないよ

エネルギー最小となる関数ψとψ'があったら(ψ≠ψ')、
適当に線型結合をとれば恒等的に0ではなくかつ零点を持つ新たな波動関数が作れちゃうでしょ
基底状態は零点を持たないのでそれはおかしい
485ご冗談でしょう?名無しさん:2009/06/28(日) 23:46:22 ID:???
>>484
局所ポテンシャルの1粒子系と多体のボーズ系ならそうだけど
多体のフェルミオン系では基底状態は必ず零点を持つ
486ご冗談でしょう?名無しさん:2009/06/29(月) 00:48:22 ID:???
基底状態が縮退してる系なんていくらでもあるじゃん
487ご冗談でしょう?名無しさん:2009/06/29(月) 04:36:47 ID:CV3sU64s
>>482
勘違いもほどほどにしろ
488ご冗談でしょう?名無しさん:2009/06/29(月) 11:24:32 ID:???
解析力学での変分法の場合には試行関数という言い方はしないと思う。
試行関数という言葉が出て来る変分法は、ある程度物理的な考察の元に
適当な関数形を仮定して試行関数とし、その中のパラメータの変動に
対してエネルギーが動かないようにする、というもの。必ずしも最小値に
なっているとは限らないし、そもそも試行関数が任意の波動関数を
表現しているとは限らないので、厳密解に行き着く保証もない。

試行関数が厳密解と同じ形をしていれば変分法によって得られたものは
厳密解に一致するけど、そもそも厳密解がわかっていれば変分法なんて
する必要がないわけで、変分法は厳密解がわからないときに固有エネルギーの
近似値を得たいというときによく用いられる手法。
解析力学での変分法とは必ずしも一致しない。
489ご冗談でしょう?名無しさん:2009/06/29(月) 14:00:22 ID:???
>>482-488
回答ありがとうございました!
波動関数の所は浅学の為よく理解できませんでしたが
気になっていた部分は>>488氏の説明のおかげで解決しました。
490ご冗談でしょう?名無しさん:2009/06/29(月) 14:03:12 ID:CV3sU64s
近似値を近似値とみなすが誤差のカタマリとみなすか。

DQNは誤差だらけなのに、近似値だと思い込むところに問題がある
491ご冗談でしょう?名無しさん:2009/06/29(月) 22:13:59 ID:aA0KgtH6
軍事板の質問スレで「高速増殖炉もんじゅ」絡みで揉めているから詳しい人来て
492ご冗談でしょう?名無しさん:2009/06/29(月) 22:25:05 ID:???
>>491
お前が連れてこい馬鹿。
493ご冗談でしょう?名無しさん:2009/06/29(月) 22:30:13 ID:???
つうか物理板の範疇じゃない。
494ご冗談でしょう?名無しさん:2009/06/29(月) 22:30:22 ID:???
連れてくんな
495ご冗談でしょう?名無しさん:2009/06/29(月) 22:33:25 ID:???
>>491
ここは出された質問に対して、気が向いた人がいれば答えるというスレ。

勝手に何かアグレッシブな行動を期待したり、求めたりするのは筋違い。
496ご冗談でしょう?名無しさん:2009/06/29(月) 22:44:17 ID:???
いつだったかこの手の出張依頼に応じてしまったら向こうからもやって来て、
質問スレ1個丸々潰すくらいに暴れまくっていったことがあったな。
497ご冗談でしょう?名無しさん:2009/06/29(月) 22:49:17 ID:???
件のスレのID:/3yJ+2ZT はあちらの知識も
物理の知識も無い中途半端なヤツということは
分かった。
498助けて!!:2009/06/29(月) 22:52:37 ID:N2jcRuRc
 中学3年の期末テストが迫っています。

次の問題の解き方、どっちが正しいのでしょうか。

問題 400gの長方形の箱が水平な床に置かれている。箱と床が接する面は、
  縦20cm、横10cmの直方体である。床にかかる圧力はいくらか。

 学校では、400gの物体に4Nの圧力が働くから、分数の上を4Nにして、分数の下のところを
0.2×0.01にして計算して、200haと習いました。

 塾では、原子は400gで、分母は10×20して、あとで100倍して、
200haにしろと習いました。

 親に聞いたら、答えは2ミリバーだとか言われました。
どのやり方が正しいのですか。

499ご冗談でしょう?名無しさん:2009/06/29(月) 23:02:14 ID:???
宿題は自力でw
500ご冗談でしょう?名無しさん:2009/06/29(月) 23:02:21 ID:???
>>498
とりあえず、「ミリバー」という単位はない。「ミリバール」ならあるが。
親は信用しない方がいいだろう。

学校の説明が一応まともだが、20cm×10cmが、どうして0.2×0.01に
なっちゃってるんだ?? 0.2×0.1だろ。


501ご冗談でしょう?名無しさん:2009/06/29(月) 23:04:35 ID:???
よく見たらhaって単位も変だな。というわけでどれも正しく
ねーよ。>>498
502ご冗談でしょう?名無しさん:2009/06/29(月) 23:07:22 ID:???
とりあえず「ha」という単位は面積の単位であって圧力の単位ではない。
「Pa」なら圧力だが
503ご冗談でしょう?名無しさん:2009/06/29(月) 23:09:18 ID:???
しかし理由も説明せずに「後で100倍しろ」と塾が教えているんなら、
塾は糞だな。

>>498が先生が説明している時に聞いてなかった可能性が大だが。
504ご冗談でしょう?名無しさん:2009/06/29(月) 23:12:23 ID:???
>>498
まず落ち着け。箱が長方形で、箱と床が接する面が直方体だとか問題文からして変だ。
505ご冗談でしょう?名無しさん:2009/06/29(月) 23:22:16 ID:???
その床は地球じゃないな。とチャチャいれても仕方ないか。

上のレスをまとめると、
長方形 <-(入れ替わり)-> 直方体
ha -> Pa
0.01 -> 0.1
ミリバー -> ミリバール
こいつらは多分、板書を書き写し間違えたんだろ。


圧力の単位Paだけど、これはN/m^2という物理量を合成した単位だ。
これがキモとなる知識。

まず、その問題では重力加速度を 10 m/s^2 としていると思える。(これは前提としてある?)
そこの場所でなら、1kgの重量は10Nの力を床に及ぼしている。
これから400gの場合なら4Nという数字になる。
次に m^2 は面積で平方メートル。つまり 1m x 1m の面積だ。
これは 20cm x 10cm は 0.2 m x 0.1m = 0.02 m^2 という数字になる。

以上を合わせれば単位が一致する。
4 / 0.02 [N/m^2] = 200 [N/m^2] = 200 [Pa] ということ。(1 [Pa] = 1 [N/m^2])


塾のは真似しないほうがいい。

親は答えしか言っていない。(1 [ミリバール] = 100 [Pa])
506ご冗談でしょう?名無しさん:2009/06/29(月) 23:25:21 ID:???
今の中学理科では、「1kgの物体に働く重力は10N」が
力の定義のように教えられているんだ。というわけで、>>498が中学生なら
その点は前提必要ない。
507ご冗談でしょう?名無しさん:2009/06/29(月) 23:27:02 ID:???
>>505
確かに、大気圧は無視しなさい、ってのも前提なのかね??
508ご冗談でしょう?名無しさん:2009/06/29(月) 23:30:04 ID:N2jcRuRc
>>505

 どうもありがとうございます。しっかりその内容を読んで勉強します。分数や小数の計算は、自分ではできないので、親に聞きながら計算しています。
509ご冗談でしょう?名無しさん:2009/06/29(月) 23:31:58 ID:???
2ミリバールと即答する親もたいしたもんだな
510ご冗談でしょう?名無しさん:2009/06/29(月) 23:34:01 ID:???
>>506
有効数字一桁なのか…。π=3と同類か
511ご冗談でしょう?名無しさん:2009/06/29(月) 23:35:03 ID:???
^ が累乗というのは中学生に伝わるのかな・・・中学生の知識範囲わかんないや。
m x m を m^2 というように、掛け合わせる個数を書く表記法。
>>3 だと基礎として使っていて、説明が無いな。
紙になら右上に小さく書けるけど、掲示板とかでは書けないから。
512ご冗談でしょう?名無しさん:2009/06/29(月) 23:40:52 ID:???
>>498
>  学校では、400gの物体に4Nの圧力が働くから、分数の上を4Nにして、分数の下のところを
> 0.2×0.01にして計算して、200haと習いました。

これは

 物体が床を押す力 400gf = 0.4kgf ≒4N  (中学理科って g = 10m/s^2 で教えてるのか)
 底面の面積 20cm×10cm = 0.2m×0.1m = 0.02m^2

 だから圧力は 4N/0.02m^2 = 200 N/m^2 = 200Pa

>  塾では、原子は400gで、分母は10×20して、あとで100倍して、
> 200haにしろと習いました。

これは

  物体が床を押す力 400gf
  底面の面積      20cm×10cm = 200cm^2

  だから圧力は 400gf / 200cm^2 = 2gf/cm^2

  で、1gf/cm^2 ≒ 0.01N/(0.01m)^2 = 100N/m^2 = 100Pa だから
  2gf/cm^2 = 200Pa

単位の変換をあらかじめやっておくか後でまとめてやるかの違いだけ。
513ご冗談でしょう?名無しさん:2009/06/29(月) 23:56:50 ID:???
>>508
>分数や小数の計算は、自分ではできないので、親に聞きながら計算しています。

おい、本気でがんばれよ!
どんなに時間がかかっても、分数の概念はしっかり身につけろ。
計算の手順だけじゃなくて、何をやっているのかという意味がわかるように。
そうすれば、単位の合成とか、確率とか、、、世の中の仕組みがわかって世界が広がるぞ。
大げさじゃなく。
514ご冗談でしょう?名無しさん:2009/06/29(月) 23:59:00 ID:N2jcRuRc
みなさま、498の父です。お忙しい中、息子の質問への回答どうもありがとうございます。

 私の世代は、学校で圧力の単位といえば、バール、ミリバール、キログラム重毎平方メートル、
グラム重毎平方センチメートル、トル、アトム、ミリメートルHgばかりで、ニュートン毎平方メートルすなわち
パスカルというのを使いませんでした。

 しかし、上の子(高校2年)が生まれたころから、台風情報でミリバールという単位を聞かなくなり、ヘクトパスカルに
なっていましたが、忙しくて気にとめていませんでした。
 
 今回の件で、息子が200haなんてノートに書いていたものだから、2ミリバールと書かないと、
駄目だろうって注意したところ、このような結果になってしまい、誠に申し訳ありません。

 圧力のことも久しぶりに勉強できて、わたくしとしても良かったです。

 乱文、失礼いたしました。
 
515ご冗談でしょう?名無しさん:2009/06/30(火) 00:02:53 ID:???
>>514
ha (ヘクタール・面積の単位)はどっちにしても大間違いです
あと息子さんは分数・小数の計算が出来ないといってますが、中学三年生でそれはとてもとてもヤバイです。
516ご冗談でしょう?名無しさん:2009/06/30(火) 00:09:20 ID:???
やべーなこりゃ
517ご冗談でしょう?名無しさん:2009/06/30(火) 00:21:28 ID:???
2chで「○○の父です」って書き込みで実物が出てきたの初めて見たw
518ご冗談でしょう?名無しさん:2009/06/30(火) 00:24:22 ID:???
>>517
 いや、本当に実物の父だって保証ないし。
519ご冗談でしょう?名無しさん:2009/06/30(火) 00:26:39 ID:???
義父かもしれんからな
520ご冗談でしょう?名無しさん:2009/06/30(火) 00:37:02 ID:Rg7Z7zNw
514の続き

 お騒がせしております。単身赴任が長かったために、低学力に気づきませんでした。

このあいだの中間テストは、成績表?によると英語100点、国語32点、社会17点、理科5点、数学0点で、英語以外は、大変悪いうえ、
テスト範囲表によると今日(30日)提出するべき数学や理科の問題集・プリントも手つかずのようで、家中で大慌てです。計算が絡むものは、
小学生時代から黒板をところどころ写すくらいしかできていないようで、38×4などの計算もどうしたらよいかわからない有様です。(分数・小数ばかりか
整数の計算もあまりできません。)家の者が総出で対応していますが、朝までに、数学の平方根・因数分解、理科の圧力(1年の復習)・物体の運動(速さ、力)・力学的エネルギーなどをどうしようかと悩んでいます。このままでは評定が1になってしまいます。
521ご冗談でしょう?名無しさん:2009/06/30(火) 00:39:54 ID:???
なんぼなんでもネタだろう。ほんとにそんな状態だったら、まず息子が
>>498を書き込んだ時点でおかしい。

ネットなんかに頼らずにちゃんと勉強させろ、あんたが親なら。>>520
522ご冗談でしょう?名無しさん:2009/06/30(火) 00:43:28 ID:???
「父親」はさすがにネタだとは思うが、>>508の「分数や小数の計算が出来ない」が本当ならいずれにしても
えらいことなので何とかしないといけないな。
523ご冗談でしょう?名無しさん:2009/06/30(火) 00:44:15 ID:???
>>498
> 分数の上を4Nにして、分数の下のところを

この時点でヤバいと思ったんだ
524ご冗談でしょう?名無しさん:2009/06/30(火) 00:49:55 ID:+KtqNBDU
>>514>>520じゃIDが異なる。
525ご冗談でしょう?名無しさん:2009/06/30(火) 00:51:42 ID:???
日付が変わったらIDも変わるんだぜ
526ご冗談でしょう?名無しさん:2009/06/30(火) 00:51:48 ID:+KtqNBDU
日付の所為だったか。親の台詞じゃないよな。
527ご冗談でしょう?名無しさん:2009/06/30(火) 01:06:59 ID:???
>>524
お前アホだろ
528ご冗談でしょう?名無しさん:2009/06/30(火) 12:29:16 ID:???
>>520
ネタだと思うが、本気で朝までに何とかなる状態だと思っているなら能天気にもほどがある。
潔く1の評定を受けたほうがよい
529ご冗談でしょう?名無しさん:2009/06/30(火) 13:01:58 ID:???
子も子なら、親も親だよな。
家族揃ってるんだ。
530ご冗談でしょう?名無しさん:2009/06/30(火) 13:02:50 ID:???
だが、まだ子のほうが救いようがある。
531ご冗談でしょう?名無しさん:2009/06/30(火) 13:04:22 ID:???
おまいら、釣られ杉
532ご冗談でしょう?名無しさん:2009/06/30(火) 13:11:21 ID:GZ2j6+V8
マンガでたまに見る描写に、「真空の力によって物体を切断もしくは破壊する」というシーンがあります。
「カマイタチ」という現象の原因も、自然に発生した真空(に近い低圧状態)であるという話も聞きます。

でも、なんかウソくさい話だ。真空にそんな力が本当にあるのでしょうか?
真空の力って、真空そのものの力ではなく、大気圧との差によって生じる力なんじゃないですか?
だったら、地球上ではどんなにがんばっても1気圧分の力しか働かないと思うのです。
真空に、物体を切断する力が本当にあるのでしょうか?
533ご冗談でしょう?名無しさん:2009/06/30(火) 13:27:47 ID:???
嘘です。
534ご冗談でしょう?名無しさん:2009/06/30(火) 13:44:27 ID:???
深海だとキャビテーションがスクリューを破壊したりするが
535ご冗談でしょう?名無しさん:2009/06/30(火) 13:46:22 ID:???
>>532
自分で正解出してるから特に補足も必要ないようだ

>でも、なんかウソくさい話だ。真空にそんな力が本当にあるのでしょうか?
ない
>真空の力って、真空そのものの力ではなく、大気圧との差によって生じる力なんじゃないですか?
yes
>だったら、地球上ではどんなにがんばっても1気圧分の力しか働かないと思うのです。
yes
>真空に、物体を切断する力が本当にあるのでしょうか?
ない

君の唯一の考え違いは、大気圧1気圧の物凄さを過少評価してるだけだ
536ご冗談でしょう?名無しさん:2009/06/30(火) 13:49:22 ID:???
>>534
キャビテーションは水蒸気の泡で、真空じゃない罠
537ご冗談でしょう?名無しさん:2009/06/30(火) 13:52:52 ID:???
キャビテーションの破壊力は、泡が潰れるときに集中する圧力による破壊だしな
538ご冗談でしょう?名無しさん:2009/06/30(火) 14:01:09 ID:???
圧力だけでなく集中した海水によるウォーターハンマー効果もバカにならん。
539ご冗談でしょう?名無しさん:2009/06/30(火) 15:51:35 ID:???
前もどこかで尋ねたんですがよくわからなかったので再度質問します。
強い力の重力を1とした相対的強さは10の40乗で
弱い力は10の15乗ということですがこの弱い力の量は
どういう意味ですか?粒子を変化させるのに強弱があるのですか?
また湯川ポテンシャルで現される距離とポテンシャルの関係も
強い力はわかりますが弱い力には何の意味があるのかわかりません。
分かる方おりましたらお教えください。
540ご冗談でしょう?名無しさん:2009/06/30(火) 17:56:57 ID:???
>>539
> 強い力の重力を1とした相対的強さは10の40乗
まずこれはどういう意味で言われているの?
541ご冗談でしょう?名無しさん:2009/06/30(火) 18:05:57 ID:???
>>539
素粒子の世界でも慣習的に「力」と呼んでいるけど、正確には相互作用。
で、相互作用の大きさを与える結合定数という量があって、その比が
(低いエネルギー領域では)それくらいのオーダーになっている、と。

>ポテンシャル
前もどこかで答えたんですがw、弱い相互作用はあまりポテンシャルで
記述しません。だから何の説明もなく弱い力(相互作用)のポテンシャルと
言われても、そもそも何を指しているのかわからない。そちらが
出くわしたのであろう弱い力のポテンシャルというものが
どういう導入の元に出てきたのか説明してもらわないことには、
それがどういう意味のものかなんて誰にもわからない。
542ご冗談でしょう?名無しさん:2009/06/30(火) 19:56:50 ID:CKFGuI2w
柿から取れる酵素って、ダイエットに効くよ。売ってるがな。バストアップにも繋がるよ。でも、飲みすぎないようにね。
543ご冗談でしょう?名無しさん:2009/06/30(火) 20:09:03 ID:???
こと健康に関して、見知りもしない人間から聞いた民間療法を信じるヤツは情弱。

そもそもスレチ。よそでやれ。
544ご冗談でしょう?名無しさん:2009/06/30(火) 20:17:59 ID:???
いつもの人だよw
545ご冗談でしょう?名無しさん:2009/06/30(火) 20:25:30 ID:???
量子井戸と超格子の違いって、波か、粒子の違いだけですか?
546ご冗談でしょう?名無しさん:2009/06/30(火) 21:23:39 ID:???
>>540
>> 強い力の重力を1とした相対的強さは10の40乗
>まずこれはどういう意味で言われているの?
京極一樹氏が書いた本に距離1fmに置かれた陽子同士で
重力の10の38乗の力と書いています。40乗というのはそれと違う距離での
力だと思います。
547ご冗談でしょう?名無しさん:2009/06/30(火) 21:24:28 ID:???
>>541
すいません。以前答えてもらって分からなかったのが僕だと思います。
>どういう導入の元に出てきたのか説明してもらわないことには
U=-g^2/4π×e^kr/r {1/k=(h/2π)/mc}
だそうですがどういう導出かはわかりません。すいません。
結合定数については勉強してみます。ありがとうございます。
548ご冗談でしょう?名無しさん:2009/06/30(火) 21:29:57 ID:???
>>545
はぁ?
549ご冗談でしょう?名無しさん:2009/06/30(火) 21:41:11 ID:???
本にこんな式も出てたのですが意味がわかりません。
強い力の式みたいです。すいませんテンソルの記述方をしりません。
Lqcd=-1/4F上μv下aF上a下μv+Σψf(i∂-M+gA上aT下a)ψ
F上a下μv=∂μA上a下μ+g下sf上a下bcA上b下μA上c下v
550ご冗談でしょう?名無しさん:2009/06/30(火) 22:11:22 ID:???
↑↑↓↓←→←→BA
551ご冗談でしょう?名無しさん:2009/06/30(火) 23:15:11 ID:???
>>546
> 距離1fmに置かれた陽子同士で
そんな長距離では弱い相互作用ははたらかないも同然。
552ご冗談でしょう?名無しさん:2009/06/30(火) 23:33:56 ID:eZCK8Rfz
電磁気の質問おねがいします。
点電荷と設置した導体球の問題で半径aの導体球を接地して球の中心から距離r_1の点に点電荷qを置いて静電ポテンシャルから電場を求め、球面上の全電荷を求めたいんですが
最後の導体球面上の積分がよくわかりません。

球面に誘導される電荷密度σ=ε_0E_nは球面上の電場の法線方向成分

E_n=−бφ(r,θ)/бr|_r=a =−q/4πε_0a×r_1^2−a^2/(a^2+r_1^2−2ar_1cosθ)^3/2
で与えられ、球面上の全電荷は
2πa^2∫(0,π) dsinθσ=−aq/r_1

と答えはなってます。
なぜこの式になるんでしょうか?
553ご冗談でしょう?名無しさん:2009/07/01(水) 00:00:24 ID:???
>>551
>> 距離1fmに置かれた陽子同士で
>そんな長距離では弱い相互作用ははたらかないも同然。
540の強い力についての回答です。
554ご冗談でしょう?名無しさん:2009/07/01(水) 00:59:37 ID:???
>>553
> 強い力の重力を1とした相対的強さは10の40乗で
> 弱い力は10の15乗ということですが
この出典ないし出てきた文脈はないの?
555ご冗談でしょう?名無しさん:2009/07/01(水) 01:19:39 ID:???
>>552
どこがわかってどこがわかんないのかわかんない
556ご冗談でしょう?名無しさん:2009/07/01(水) 01:23:19 ID:BhzC79wM
>>555
返事ありがとうございます。

取り出すと
なんで球面上の全電荷は
2πa^2∫(0〜π) dθsinθσ
で与えられるかがわかりません;
計算は行い、答えと同じになりましたが腑に落ちません
557ご冗談でしょう?名無しさん:2009/07/01(水) 01:27:23 ID:???
>>552
球面上、θ〜θ+dθの部分の面積を考えてみよう
558ご冗談でしょう?名無しさん:2009/07/01(水) 01:27:40 ID:???
>>556
回転体の表面積って高校数学でやらないか?
559ご冗談でしょう?名無しさん:2009/07/01(水) 01:28:46 ID:???
δ(r-a)dxdydz = sinθdθdφ、
∫dφ = 2π
とかそんな感じで
560ご冗談でしょう?名無しさん:2009/07/01(水) 01:33:47 ID:BhzC79wM
>>558
うちの高校はレベル低いのか回転体の体積しか記憶にないっす;
561ご冗談でしょう?名無しさん:2009/07/01(水) 01:35:14 ID:BhzC79wM
>>559
回答ありがとうございます。
でもδ(r-a)dxdydz = sinθdθdφがさっぱりです
562ご冗談でしょう?名無しさん:2009/07/01(水) 03:01:12 ID:8wGPktt9
中性子星が単独で十分時間が経って冷えたら、どんな状態になるのでしょうか?
不確定性の限界に近づいたら、何か奇妙なことが起きる・・・とか想像したのですが。
冷えても縮まない・・・ですよね?
小さな表面積から出て行く放射では、宇宙の年齢程度では冷えない?
563ご冗談でしょう?名無しさん:2009/07/01(水) 03:10:07 ID:AMG8zS6R
>>551
働くよ、馬鹿野郎!!
564ご冗談でしょう?名無しさん:2009/07/01(水) 03:15:19 ID:???
物理では、小さいことも見逃すなよ。当たり前だろ。
565ご冗談でしょう?名無しさん:2009/07/01(水) 03:34:28 ID:???
易しく書くと、小さな振舞いも見逃すな だよ。
566ご冗談でしょう?名無しさん:2009/07/01(水) 04:21:00 ID:AMG8zS6R
同級生のあいつが、医者やっていると思うと、ガクガクブルブルだろ。普通に考えようね。過去レスですが。
567ご冗談でしょう?名無しさん:2009/07/01(水) 04:34:27 ID:1G3wO/GD

             /)
           ///)
          /,.=゙''"/
   /     i f ,.r='"-‐'つ____   こまけぇこたぁいいんだよ!!
  /      /   _,.-‐'~/⌒  ⌒\
    /   ,i   ,二ニ⊃( ●). (●)\
   /    ノ    il゙フ::::::⌒(__人__)⌒::::: \
      ,イ「ト、  ,!,!|     |r┬-|     |
     / iトヾヽ_/ィ"\      `ー'´     /
568ご冗談でしょう?名無しさん:2009/07/01(水) 10:52:37 ID:???
>>547
e^kr/rはe^(-kr)/rの間違いだと思うけど、それ、強い力のポテンシャルじゃない?
そこに出てくるgが結合定数な。
569ご冗談でしょう?名無しさん:2009/07/01(水) 12:22:31 ID:???
>>568
強い力ならkrだ。遠方にいっても弱まらない。「核力」と間違えてないか?
570ご冗談でしょう?名無しさん:2009/07/01(水) 12:55:07 ID:???
sin波は純音なんですよね?
これを

サンプリング周波数10の離散時間波形にして
DFTすると

純音なのに2つのスペクトルがあらわれるのです。
なぜでしょうか?
571ご冗談でしょう?名無しさん:2009/07/01(水) 13:58:57 ID:???
>>570
極端な話、sin(2πt) を t=0,1,2,3… でサンプリングしたらどうなる?
572ご冗談でしょう?名無しさん:2009/07/01(水) 14:02:39 ID:???
10GHzの正弦波を、サンプリング周波数10/日でDFTとかしてないか?
573ご冗談でしょう?名無しさん:2009/07/01(水) 14:28:52 ID:???
>>554
>> 強い力の重力を1とした相対的強さは10の40乗で
>> 弱い力は10の15乗ということですが
>この出典ないし出てきた文脈はないの?
こんな感じで表で紹介されていたのを書きました。
http://www.netlaputa.ne.jp/~hijk/memo/quark.html
ここの表は弱い力は重力の35乗になってますが。
それ以上のことはわかりません。

>>568
>e^kr/rはe^(-kr)/rの間違いだと思うけど、それ、強い力のポテンシャルじゃない?
ここの↓{1/k=(h/2π)/mc}の部分のmがπだと
強い相互作用でwだと弱い相互作用みたいです。
>U=-g^2/4π×e^kr/r {1/k=(h/2π)/mc}

>そこに出てくるgが結合定数な。
これが結合定数ですか。どうも。
574ご冗談でしょう?名無しさん:2009/07/01(水) 14:52:13 ID:???
>>573
このサイトほんとフォント見にくいなw
(そりゃss替えりゃいいだけだけど)
575ご冗談でしょう?名無しさん:2009/07/01(水) 16:01:55 ID:???
>>573
こういう表は、ニュートン重力とクーロン静電気力の比較以外はほとんど意味がない。
「ステーキはラーメンの5倍うまい」と五十歩百歩。
576ご冗談でしょう?名無しさん:2009/07/01(水) 16:42:40 ID:???
>>571
1HZのところに線スペクトルがたって
なぜか9Hzのところにも線スペクトルがでてきます

どちらも振幅は10です。。

>>572
いちおうやってないです。
577ご冗談でしょう?名無しさん:2009/07/01(水) 16:51:53 ID:???
>>576
サンプリング周波数10と書いていて、単位を書かないという点を指摘しているんだと思うんだが。
それともどこかの業界だと単位は書かないで省略する風習があるのか?
578ご冗談でしょう?名無しさん:2009/07/01(水) 18:34:14 ID:Gnu8JXQb
インダクタンスやキャパシタンスが負の値を取ることってありますか?
579ご冗談でしょう?名無しさん:2009/07/01(水) 18:56:44 ID:GUNGN/HI
見かけじょうならww
580ご冗談でしょう?名無しさん:2009/07/01(水) 22:14:56 ID:???
>>576
ω = exp(2πi / 10) として、ω^9 = ω^(-1) なのは理解できる?
581ご冗談でしょう?名無しさん:2009/07/01(水) 23:21:39 ID:???
>>576
映画で車輪が逆回転して見える理屈だね
582ご冗談でしょう?名無しさん:2009/07/01(水) 23:28:21 ID:???
ロスアラモス研究所、光速を超える電波の送信装置の開発に成功
http://www.technobahn.com/cgi-bin/news/read2?f=200907012012


相変わらずテクノバーンお得意の糞翻訳なんだろうけど、これはどういう意味なんでしょ?
http://arxiv.org/abs/physics/0405062

We describe the experimental implementation of a superluminal ({\it i.e.} faster than light {\it in vacuo})
polarization current distribution that both oscillates and undergoes centripetal acceleration.
583ご冗談でしょう?名無しさん:2009/07/02(木) 09:44:26 ID:???
私達は、振動し中心加速度を受ける超高速の分極電流分布の
実験的実現を記述する。?
584583:2009/07/02(木) 10:02:50 ID:???
ていうか、その続きに書いてあるじゃん
Theoretical treatments lead one to expect
that the radiation emitted from each volume element of such a polarization current
will comprise a \v{C}erenkov-like envelope with two sheets that meet along a cusp.

分極電流から放射された電磁波がチェレンコフ光のようになる、
ってことじゃないの?
\v{C}erenkovの\v{C}ってのはどういう意味なのか、誰か教えてください。
585ご冗談でしょう?名無しさん:2009/07/02(木) 11:41:07 ID:???
>>584
Cの上にvが乗っかった拡張ラテン文字(C with caron)。
\v{C} が何の書式なんだかは知らないけど、とりあえずそういうこと。
586ご冗談でしょう?名無しさん:2009/07/02(木) 11:53:00 ID:???
サンプリング定理についてなんですが
原信号帯域幅の2倍より高い任意の周波数という表記もあれば
2倍以上
という表記があります

2倍より高いというのは2倍超過という意味ですよね?
587ご冗談でしょう?名無しさん:2009/07/02(木) 11:54:31 ID:???
すいません途中で送信してしまいました。
二倍超過と2倍以上ってどちらが正しいのでしょうか?
588ご冗談でしょう?名無しさん:2009/07/02(木) 12:06:26 ID:???
>>585
ありがとうございます
589ご冗談でしょう?名無しさん:2009/07/02(木) 12:15:03 ID:???
>>587
日本語じゃ同じ意味にしか考えられねぇじゃん。
590ご冗談でしょう?名無しさん:2009/07/02(木) 13:24:47 ID:wh4at0sd
(xp+px)/2がエルミート演算子になる理由教えて下さい。
591ご冗談でしょう?名無しさん:2009/07/02(木) 13:28:09 ID:???
>>590
エルミート演算子の定義は知ってるのか?
592ご冗談でしょう?名無しさん:2009/07/02(木) 13:35:18 ID:wh4at0sd
(Af,g)=(f,Ag)となるAをエルミート演算子
593ご冗談でしょう?名無しさん:2009/07/02(木) 14:01:47 ID:???
(AB)†=B†A†より自明
594ご冗談でしょう?名無しさん:2009/07/02(木) 14:02:47 ID:???
>>589
すいません
2.000000001倍(2倍超過)が正しいのでしょうか?
それとも2.00000倍(2倍以上)が正しいのでしょうか?
という質問です。
595ご冗談でしょう?名無しさん:2009/07/02(木) 14:04:56 ID:???
お前馬鹿だろ。
596ご冗談でしょう?名無しさん:2009/07/02(木) 14:15:06 ID:???
>>594
なんか切ないな
597ご冗談でしょう?名無しさん:2009/07/02(木) 14:52:42 ID:6v0VBrTV
気泡(空気or窒素)の存在するトルエン溶液に
磁石を近づけると、その気泡は磁石に近づくのでしょうか
それとも遠のくのか、変化はしないのでしょうか?
磁化率がポイントだと思うのですが、
わからないので教えてください。
598ご冗談でしょう?名無しさん:2009/07/02(木) 15:22:24 ID:pNcWgBVv
ベンチュリー効果?ベルヌーイの定理?
プラントの運転をしてる者なんですけど。
直径10cmくらいの管に水が20〜30㎥/h流れています。この管に直径2cmくらいの
管が付いていて薬品を注入しています。薬品の圧送ポンプの回転数は一定です。
しかし、薬品の消費量が毎日変わってしまいます。これは水の流量が20〜30㎥/hに
変化するのと関係がありますか?
599ご冗談でしょう?名無しさん:2009/07/02(木) 15:26:31 ID:???
>>598
ただの気温による熱膨張じゃねえの?
600ご冗談でしょう?名無しさん:2009/07/02(木) 15:49:23 ID:pNcWgBVv
>>599
それもあると思います。でも気温だけじゃなさそうなんです。
複合的な要因で起こってると思うので上記のような可能性もないかなと。
601ご冗談でしょう?名無しさん:2009/07/02(木) 15:51:38 ID:???
普通に関係あるだろ
602ご冗談でしょう?名無しさん:2009/07/02(木) 16:36:22 ID:wh4at0sd
>>593
どういう事ですか??
603ご冗談でしょう?名無しさん:2009/07/02(木) 16:45:05 ID:???
>>602
x、pがエルミートだから
604ご冗談でしょう?名無しさん:2009/07/02(木) 16:47:09 ID:wh4at0sd
どうしてエルミートなんですか??すいません。
605ご冗談でしょう?名無しさん:2009/07/02(木) 16:49:55 ID:???
>>604
xやpがエルミートじゃないなら(xp+px)/2もエルミートとは限らない。
で、xとpはそれぞれ何?
606ご冗談でしょう?名無しさん:2009/07/02(木) 16:51:46 ID:wh4at0sd
xは位置の演算子でpは運動量演算子です。
607ご冗談でしょう?名無しさん:2009/07/02(木) 17:34:37 ID:???
やりとりを見ていると、そもそもwh4at0sdはエルミートってなんだか
知らねーんじゃねーの、と思えてくる。
608ご冗談でしょう?名無しさん:2009/07/02(木) 21:05:17 ID:qvvKiXK6
3枚の導体板(面積S、厚さa) A、B、Cを互いに平行に間隔dで置く。Aに電荷Qa、Bに電荷Qb、Cに電荷Qcを与える。導体板A、B、Cの表面と裏面(左と右)に現れる電気量をそれぞれ求めよ。


お願いします
609ご冗談でしょう?名無しさん:2009/07/02(木) 21:12:37 ID:???
>>608
まぁ落ち着いて>>1を読め
610ご冗談でしょう?名無しさん:2009/07/02(木) 23:49:10 ID:???
>>594

例えば y = sin(x) を x = 0, π, 2π… でサンプリングしたらどうなるか考えて見れ。
611ご冗談でしょう?名無しさん:2009/07/03(金) 00:18:00 ID:???
612ご冗談でしょう?名無しさん:2009/07/03(金) 00:27:53 ID:???
高校物理Tの流体力学から質問お願いします。
アルキメデスの原理 F=ρVg=ρhSg(N) と、水圧 p=ρhg(Pa) と、水中での圧力 p'=p0+ρhg から、
Sp=ρVg=ρhSg=Sp'-p0 という式を構築することは可能ですか?
613ご冗談でしょう?名無しさん:2009/07/03(金) 00:30:39 ID:???
>>606
問題。
e^(aip/h)がユニタリー演算子である事を証明してみよ。
aは定数、 iは虚数単位、pは運動量演算子。
614ご冗談でしょう?名無しさん:2009/07/03(金) 00:47:33 ID:???
そういうのいいから
615ご冗談でしょう?名無しさん:2009/07/03(金) 01:18:14 ID:???
>>613
{e^(aip/h)}^{†}=e^{-ai(p^{†})/h}=e^{-aip/h}={e^(aip/h)}^{-1}
616ご冗談でしょう?名無しさん:2009/07/03(金) 09:03:05 ID:???
>>614
ごめん。
617ご冗談でしょう?名無しさん:2009/07/03(金) 10:54:38 ID:???
>>587
,担当教授に質問すれば?
618ご冗談でしょう?名無しさん:2009/07/03(金) 11:22:56 ID:???
>>598
そういうのは物理板ではなく機械・工学板で扱うこと。それはそれとして、
容積型でないポンプは回転数一定でも吸い込み口と吐き出し口の圧力差によって流量は変化する。
その関係は「揚程−流量」曲線で表される。本管の流量が変化するにつれて、その接続部分の圧力も
変化しているのだろう。
619ご冗談でしょう?名無しさん:2009/07/03(金) 18:00:40 ID:???
5kgの物体がマッハ1(大気中)で動いた時のエネルギー量はいくらですか?
あと10kgの物体がマッハで動いた時のエネルギー量は5kgの時の何倍ですか?
620ご冗談でしょう?名無しさん:2009/07/03(金) 18:26:41 ID:???
>>619
ざっと東京ドーム2杯分くらい。10kgならその倍。
621ご冗談でしょう?名無しさん:2009/07/03(金) 18:38:34 ID:???
質問間違えました
5kgの物体がマッハ1で静止してる物体に衝突した時のエネルギーはどれくらいですか?でした
622ご冗談でしょう?名無しさん:2009/07/03(金) 18:52:06 ID:???
>>621  2.89x10^5J

100gの牛乳から得られる熱量に匹敵する。牛乳えらい。
623ご冗談でしょう?名無しさん:2009/07/03(金) 19:02:40 ID:???
牛乳凄いですねありがとうございます
624ご冗談でしょう?名無しさん:2009/07/04(土) 09:41:14 ID:???
光子という粒は何種類もありますか?
例えば赤い光の光子と青い光の光子は別種類の光子ですか?
同じだけどエネルギーが違うということでしょうか?
それとも最初から別種類ですか?

たぶん赤外線の光子は弱くて、紫外線の光子は強いように思いますが、
なんでですか
625ご冗談でしょう?名無しさん:2009/07/04(土) 10:31:26 ID:???
あのー1種類ですが。
626ご冗談でしょう?名無しさん:2009/07/04(土) 10:40:59 ID:???
>>625
光電効果が起こる光子と起こらない光子の違いは何ですか?
627ご冗談でしょう?名無しさん:2009/07/04(土) 10:46:16 ID:???
光子のエネルギーですね。
628ご冗談でしょう?名無しさん:2009/07/04(土) 10:48:46 ID:???
>>626
基本的なところを理解してないみたいだから、教科書を最低でも3回は音読してからまたおいで。
629ご冗談でしょう?名無しさん:2009/07/04(土) 10:53:32 ID:zeoXiNxv
>>626
電子にも運のいい奴と悪い奴がいるのだよ。
630ご冗談でしょう?名無しさん:2009/07/04(土) 11:01:46 ID:yM8YJUzl
考え方等は解るのですが、
計算が解りませんので質問させて頂きます。


http://imepita.jp/20090704/394750
↑の画像の@を代入すれば、の下です
631ご冗談でしょう?名無しさん:2009/07/04(土) 11:04:11 ID:???
エネルギーが違うのは分かってるのですが、どう聞いていいか…

エネルギーの高さは振動数で、振動数のが高いと波長が短いので、
例えば赤外線より紫外線の方が波長長くて振動数高くて、
光電効果の結果の、光電子のエネルギーは高くなる。
それは知ってるんですが、その違いは何であるんですか?
632ご冗談でしょう?名無しさん:2009/07/04(土) 11:06:39 ID:???
>>630
代入するだけ
ルートの計算分からんなら中学の数学からやり直せ
633ご冗談でしょう?名無しさん:2009/07/04(土) 11:15:41 ID:???
打ちミスすみません
聞き方変えます。赤い光を青くしたり、青い光を赤くしたりできますか?
予算とか技術面で出来る出来ないかと、
出来ないとしても、理屈としては一粒の光子の振動数を変えるだけでいいんですか?
634ご冗談でしょう?名無しさん:2009/07/04(土) 11:24:19 ID:???
正値な演算子A、Bに対して、tr(AB)=>0となるのは何でですか?
635ご冗談でしょう?名無しさん:2009/07/04(土) 11:34:18 ID:???
とりあえず落ち着け
636ご冗談でしょう?名無しさん:2009/07/04(土) 11:59:13 ID:???
>>633
青い光に頑張って走って近付けば赤くなるよ
637ご冗談でしょう?名無しさん:2009/07/04(土) 12:08:05 ID:eqqcnK00
双極子モーメントは、何でモーメントというのですか?
638ご冗談でしょう?名無しさん:2009/07/04(土) 12:18:41 ID:???
なんか分布f(r)があった時 f(r) * r^n の積分をモーメントという
639ご冗談でしょう?名無しさん:2009/07/04(土) 12:19:51 ID:???
>>633
どういう立場の人なのかで答えが変わる
640ご冗談でしょう?名無しさん:2009/07/04(土) 12:24:51 ID:eqqcnK00
>638 双極子モーメントはその定義に当てはまっていますか?
641ご冗談でしょう?名無しさん:2009/07/04(土) 12:28:24 ID:???
少しは考えるといいと思うよ
642ご冗談でしょう?名無しさん:2009/07/04(土) 13:06:09 ID:???
>>633
> 聞き方変えます。赤い光を青くしたり、青い光を赤くしたりできますか?
> 予算とか技術面で出来る出来ないかと、
> 出来ないとしても、理屈としては一粒の光子の振動数を変えるだけでいいんですか?

できるよ。
紫外線は蛍光物質で波長を長くできる。
白色LEDも単波長の光を蛍光物質で多波長化した物が多い。
波長を短くするのはニオブ酸リチウムなどの非線形媒体。
波長が1/2とか1/3といった整数分の一になる。

端折って書いたけど、調べてみ。
643ご冗談でしょう?名無しさん:2009/07/04(土) 13:23:39 ID:???
>>634
Bの固有ベクトルから正規直交基底を作って
trの定義式で使えばいい
644ご冗談でしょう?名無しさん:2009/07/04(土) 15:06:22 ID:Vw+VxTey
>>643
すいません。もう少し詳しく教えて下さい。
645ご冗談でしょう?名無しさん:2009/07/04(土) 15:19:48 ID:???
>>644
どこまで分かってどこから分からないのかぐらいは礼儀として書くべき
646ご冗談でしょう?名無しさん:2009/07/04(土) 16:22:33 ID:???
私は分厚い色覚検査表で普通の人が分からないものが1多く見える。
2000人に一人ぐらいしかいないそうだ。昔は異常色覚とされていた。
647ご冗談でしょう?名無しさん:2009/07/04(土) 16:24:55 ID:???
+1ってwww www
648ご冗談でしょう?名無しさん:2009/07/04(土) 16:38:37 ID:???
そのためにニュープリント、称して放送される、映画の中に色のさめたものがけっこう多いということがわかる。
649ご冗談でしょう?名無しさん:2009/07/04(土) 18:26:55 ID:???
そやからどないやっちゅーねん。
650ご冗談でしょう?名無しさん:2009/07/04(土) 19:58:12 ID:???
前から思ってたんだけど>>640とか>>644とかみたいな人は
(´・ω・`)ショボーン まあもういいかなあどうせわかんないし
になるの?
651ご冗談でしょう?名無しさん:2009/07/04(土) 20:01:29 ID:???
結局その程度しか"知る"必要がなかったんじゃね
652ご冗談でしょう?名無しさん:2009/07/04(土) 20:36:14 ID:agyVPRj/
運動の比較をする場合に運動量と運動エネルギーではどちらが良いのでしょうか?

この疑問を持ったのは、テニスの最高速は230km/sで質量60g、野球の最高速は160で質量160g
この場合は運動的に強いのかと思いました
高校物理と簡単な大学物理は少し齧ったぐらいです
よろしくお願いします
653ご冗談でしょう?名無しさん:2009/07/04(土) 20:46:35 ID:???
>>652
状況による。

例えば「その球をキャッチした人が何センチ後ずさるか」を比較したいなら、
エネルギーの方がいい。「その球があたったらどれくらい痛いか」を比較したい
なら、運動量の方がいい。

case by case.
654ご冗談でしょう?名無しさん:2009/07/04(土) 20:48:56 ID:???
なるほど、652じゃないがアリガト
655652:2009/07/04(土) 20:52:06 ID:???
>>653
ありがとうございました
656ご冗談でしょう?名無しさん:2009/07/04(土) 21:04:54 ID:AJp8vojj
>>650
馬鹿の相手をすると荒れるだけだから。

サンプル
http://science6.2ch.net/test/read.cgi/sci/1237280643/
657ご冗談でしょう?名無しさん:2009/07/04(土) 21:28:42 ID:???
>>653
それ逆だろ。
658ご冗談でしょう?名無しさん:2009/07/04(土) 21:32:20 ID:???
>>657
おいおい、E = μPLだろ
659ご冗談でしょう?名無しさん:2009/07/04(土) 21:59:06 ID:???
>>653
運動量と運動エネルギー間違えてますよ。
660ご冗談でしょう?名無しさん:2009/07/04(土) 22:39:28 ID:???
逆だな
661ご冗談でしょう?名無しさん:2009/07/04(土) 23:15:17 ID:???
いや、その逆だ!
662ご冗談でしょう?名無しさん:2009/07/04(土) 23:21:08 ID:???
いま考えていることの逆が正解だ
663ご冗談でしょう?名無しさん:2009/07/04(土) 23:24:31 ID:???
エネルギーは仕事と関係し、運動量は力積と関係する。
664652:2009/07/04(土) 23:32:09 ID:???
表現が曖昧すぎたことを謝る
物体に当たった時の衝撃力で比較するのか、球の威力で比較するのかで
話は変わってきますね。物理屋はエネルギーで工学系の人は衝撃力で考えるために
運動量で比較するのかな?実際は分かりませんが
665ご冗談でしょう?名無しさん:2009/07/04(土) 23:36:40 ID:???
弾丸の衝撃力は運動量で考えてたな
666ご冗談でしょう?名無しさん:2009/07/04(土) 23:45:23 ID:???
運動量は一定。
衝撃力はボールや衝撃を受ける側の材質とか様々な要因で変化する。
667ご冗談でしょう?名無しさん:2009/07/04(土) 23:45:56 ID:???
曖昧じゃなく内容で間違えてるがな。
668ご冗談でしょう?名無しさん:2009/07/04(土) 23:48:59 ID:???
>>667
そいつと同じスレにいたんだが、どういうことなんだぜ?
それと、こういう場合はどんな比較するべきなのか教えてくれ
669ご冗談でしょう?名無しさん:2009/07/04(土) 23:54:14 ID:???
運動量かエネルギーかなんてそんな単純な二択で「衝撃力」をうまく表現できるものなのか?
670ご冗談でしょう?名無しさん:2009/07/05(日) 00:04:53 ID:/yTeLDHh
水を入れたペットボトルが風呂に沈むのは、PETの比重が水より重いからですよね
とすると水より比重の軽い容器に水を入れたら沈まないんでしょうか
不思議です。いえ不思議じゃありませんね
671ご冗談でしょう?名無しさん:2009/07/05(日) 00:05:53 ID:???
>>669
簡単なモデルの場合だったら力積考えればいいんだから衝撃力は表現できるだろ
弾が物体に当たって運動量がゼロになったとして、その時の力が一定なら
FΔt = mv だからF=mv/Δt この時の力が衝撃力だろ
672ご冗談でしょう?名無しさん:2009/07/05(日) 00:06:48 ID:???
木の船に水が充満したら沈みますか?
673ご冗談でしょう?名無しさん:2009/07/05(日) 00:10:05 ID:???
>>671
物理的に定義されてる訳じゃない「衝撃力」をどう定義するかによるだろ。
674ご冗談でしょう?名無しさん:2009/07/05(日) 00:14:46 ID:???
結局>>652の球の運動はどう比較するべきなんだよ
675ご冗談でしょう?名無しさん:2009/07/05(日) 00:16:52 ID:???
>>673
そうだな。書き方が悪かった
俺は質問者が考えているであろう衝撃力で定義してみた
676ご冗談でしょう?名無しさん:2009/07/05(日) 00:17:09 ID:???
>>671
よく知らないんだが
「衝撃力」って短時間で散乱が起きたとしてそのときに受ける力の時間平均みたいな量で定義されてるんだっけ
もっと日常で言う「衝撃の度合い」みたいなものの話をしてるのかと思った
677ご冗談でしょう?名無しさん:2009/07/05(日) 00:19:53 ID:???
>>675
>>652
> 運動的に強い
をどう解釈したの?
678ご冗談でしょう?名無しさん:2009/07/05(日) 00:20:46 ID:???
真空がヒッグス粒子で埋まったというのは弱い力と電磁気力が
分かれたときですか?それとも重力と強い力&電弱力が分かれたとき
ですか?
また相転移を起こす理由は何ですか?できれば数式を絡めて
教えていただければありがたいです。
679ご冗談でしょう?名無しさん:2009/07/05(日) 00:21:09 ID:???
>>672
水面より上の部分にある船上の水と、水面下にある船体部分が受ける浮力が等しくなるまで沈む(はず)
船が沈没するかは船の形状や材質、水の比重などにもよる(海水か真水か)
680ご冗談でしょう?名無しさん:2009/07/05(日) 00:21:42 ID:???
>>674
だから結局「何」を比較したいかによるだろ。
あたったどちらが痛いか?とか、(モデルとしては複雑で生理学的問題も含み、物理で簡単に答えは出せないだろう)
1kgの鉄球にぶつけた時、より鉄球が早く動くのは?(簡単な物理モデルで答えも簡単に出る)
とか。
681ご冗談でしょう?名無しさん:2009/07/05(日) 00:22:39 ID:???
>>677
>>664
>物体に当たった時の衝撃力で
ここみたんよ
682ご冗談でしょう?名無しさん:2009/07/05(日) 00:23:14 ID:???
ダイナマイトとか爆弾の威力の単位(?)の「トン」ってどういう意味ですか?

また格闘家の蹴りの衝撃が一トンとか、車に衝突したときの衝撃とか、どういう計算で出るんでしょうか?
F=maですか?
683ご冗談でしょう?名無しさん:2009/07/05(日) 00:26:12 ID:???
>>678
相転移を起こす理由はHiggs粒子のない状態よりもHiggs粒子の存在する状態の方がエネルギーが低いからだね
場の量子論が分かっているなら数式も踏まえたもう少し真面目な説明もできる
684ご冗談でしょう?名無しさん:2009/07/05(日) 00:34:21 ID:Ro5J/Lgw
>>682
ダイナマイトは質量じゃないのかな

蹴りが1トンとかいうのは、計算ではなく測定だと思われる

車の衝突時も実験や、そこからの推測ではなかろうか
計算するんなら F=ma
685ご冗談でしょう?名無しさん:2009/07/05(日) 00:35:38 ID:???
>>682
>ダイナマイトとか爆弾の威力の単位(?)の「トン」ってどういう意味ですか?

核爆弾のイールド(威力)の単位に良く使われるkt(キロトン)とかなら、エネルギーのTNT火薬換算量。

>また格闘家の蹴りの衝撃が一トンとか、車に衝突したときの衝撃とか、どういう計算で出るんでしょうか?
>F=maですか?

これは使われ方をきちんとみないと断言できないが、
おそらく 衝突で受けた最大「力」だろう、単位をNなどでなく重量(やはり力)としてt重で表してる。
また 衝突全体を通しての「力積」かもしれない、その場合やはり単位をkgm/sでなくtm/sで表してる事になるのかな。
686ご冗談でしょう?名無しさん:2009/07/05(日) 00:37:40 ID:???
>>683
街で聞いてもまず答えが返ってこない問いが返ってくるネットってほんとすごいです。
回答者に感謝致します。
>相転移を起こす理由はHiggs粒子のない状態よりもHiggs粒子の存在する状態の方がエネルギーが低いからだね
相転移が起きる前はヒッグス粒子が存在しない状態の方がエネルギーが
低いってことですか?その境目ってのはどう導出されるんですか?
>場の量子論が分かっているなら数式も踏まえたもう少し真面目な説明もできる
消滅生成演算子どうこうで発散してしまうケースが出るので
組み込みで解決する云々というのを少し知っている程度です。
私は実際それらに数値を代入してどうこうできるレベルの知識は
ありませんが少し数式で説明してもらえれば後学のために大変ためになります。
687ご冗談でしょう?名無しさん:2009/07/05(日) 00:43:36 ID:tNT3QFx3
>>684
>ダイナマイトは質量じゃないのかな
普通は、NTT火薬の重さに換算した表現。
688ご冗談でしょう?名無しさん:2009/07/05(日) 00:45:56 ID:???
撃力っぽい力の瞬間最大値って測定しにくそう
バネで測るとバネの特性に応じた平均値しか得られないだろうから
測定器ごとに値がばらばらだろうし
689ご冗談でしょう?名無しさん:2009/07/05(日) 00:48:14 ID:???
>>687
そうらしいね、すまん
TNTだけど
690ご冗談でしょう?名無しさん:2009/07/05(日) 00:48:32 ID:???
まあ、蹴りの力が1トンとかってのはあんまり意味のない話だろうな。
691ご冗談でしょう?名無しさん:2009/07/05(日) 01:09:21 ID:SGpLKVk2
エヴァ作るのって可能ですか?
692ご冗談でしょう?名無しさん:2009/07/05(日) 01:30:43 ID:???
無理。
693ご冗談でしょう?名無しさん:2009/07/05(日) 01:36:29 ID:???
TNT換算トンはエネルギーの単位で、1TNT換算トン=10億カロリー
694ご冗談でしょう?名無しさん:2009/07/05(日) 02:17:49 ID:???
>>683
今のところ「Higgs粒子」ってのは「Higgs機構で自発的対称性を破る時に
出てくる粒子」の総称だから、電弱が分かれる時にも、強と電弱が分かれる
時にも「Higgs粒子」は出てくる。別種類だが同じ名前で呼ぶ。
 まぁ、大統一理論は今のところまだ確実じゃないから、ふつ〜「Higgs粒子」
と言われたら思い浮かべるのは電弱が分かれる時の奴だと思う。
695ご冗談でしょう?名無しさん:2009/07/05(日) 02:26:05 ID:???
二つのヒッグス場のことかね
696ご冗談でしょう?名無しさん:2009/07/05(日) 02:29:34 ID:???
> 自発的対称性を破る時に

「自発的対称性の破れ」の「自発的」は「破れ」の方に掛かってるのだと思ったが、
「自発的対称性を破る」はおかしくないか?
697694:2009/07/05(日) 02:43:44 ID:???
>>696
うん、ごめん書き間違い。「自発的に対称性を破る時に」と書いたつもりで、
「に」が抜けてた。

698ご冗談でしょう?名無しさん:2009/07/05(日) 02:49:07 ID:???
区別つける時は、GUT-Higgs と electroweak-Higgsとか言う。
699ご冗談でしょう?名無しさん:2009/07/05(日) 02:59:01 ID:???
LHCの当初の予定だと5月中には実験結果出るはずが事故で遅れてるのか?
700ご冗談でしょう?名無しさん:2009/07/05(日) 07:25:21 ID:???
ヒッグス場は二つあったのですね!標準理論は理解しなくちゃならないことが
たくさんあって何冊か本を読んでるんですが気がつきませんでした。
今理解しようと思ってるのが計算式に数値を入れ何が発散して
どう組み込みするとそれが防げるのか。それとなぜ質量があると組み込み
不可能でヒッグス粒子が必要になるのかです。
701ご冗談でしょう?名無しさん:2009/07/05(日) 07:30:07 ID:???
>>700
場の量子論勉強しないと本当の意味での理解は出来ないよ。
道のりは長いと思うがあきらめずガンガレ
702ご冗談でしょう?名無しさん:2009/07/05(日) 07:33:53 ID:???
組み込みじゃなくて繰り込みだろ
703ご冗談でしょう?名無しさん:2009/07/05(日) 07:36:23 ID:tNT3QFx3
>>699
>LHCの当初の予定だと5月中には実験結果出るはずが事故で遅れてるのか?
LHCは国際的な反対運動の高揚の為、再開のめどが立たなくなりました。
予算の削減もあり、実験は規模縮小ないし中止の可能性もあります。
とりあえず秋までに改修工事は継続中だよ。
http://cdsweb.cern.ch/journal/article?name=CERNBulletin&issue=26/2009&number=4&category=News%20Articles&ln=fr
704ご冗談でしょう?名無しさん:2009/07/05(日) 09:41:25 ID:???
>>703
フランス語読めないので最後の「ln=fr」を「ln=en」に変えて読んだが
修理の進捗状況の報告だけで
>LHCは国際的な反対運動の高揚の為、再開のめどが立たなくなりました。
>予算の削減もあり、実験は規模縮小ないし中止の可能性もあります。
のような記述は見当たらないんだけど
705ご冗談でしょう?名無しさん:2009/07/05(日) 10:09:55 ID:???
706ご冗談でしょう?名無しさん:2009/07/05(日) 10:11:27 ID:tNT3QFx3
707ご冗談でしょう?名無しさん:2009/07/05(日) 10:49:05 ID:???
ラプラス変換って物理的にはどういう意味があるんですか?
フーリエ変換は波の分解とか重ね合わせみたいな意味だってことはわかるんですけど
708ご冗談でしょう?名無しさん:2009/07/05(日) 11:25:03 ID:???
物理的意味などない
709ご冗談でしょう?名無しさん:2009/07/05(日) 11:58:06 ID:tNT3QFx3
人生にも物理的意味などない。
710ご冗談でしょう?名無しさん:2009/07/05(日) 13:04:29 ID:???
>>701
エールありがとうございます。
道のりは長いですが頑張ります。

>>702
>組み込みじゃなくて繰り込みだろ
おはずかしい。学の無いところが出てしまいました。
711ご冗談でしょう?名無しさん:2009/07/05(日) 14:22:42 ID:dwzWxQtc
>>707線形空間に出来る。
712ご冗談でしょう?名無しさん:2009/07/05(日) 14:47:00 ID:???
もともと線形だし
713ご冗談でしょう?名無しさん:2009/07/05(日) 18:41:50 ID:GvtqbXHz
こんにちは。
非可換ゲージ理論における
ラグランジアンの不変性
について質問があります。

今、非可換ゲージ変換に関して
ラグランジアンが不変であることを
式で導こうとしているんですが、

gをLie群の元、r_{\mu \nu} をガンマ行列として

g^{-1} r_{\mu \nu} g = r_{\mu \nu}

が成立していないと、不変性が導けない
ことに気づきました。

ということは、Lie群の生成子はガンマ行列に対して
常に可換でなければならないことになりますが、

この考えは正しいのでしょうか?

場の理論に詳しい方のアドバイスをお持ちしております。


714ご冗談でしょう?名無しさん:2009/07/05(日) 18:44:41 ID:???
こんにちは。
非痴漢メコスージ理論における
メコスジアンの不変性
について膣悶があります。
715ご冗談でしょう?名無しさん:2009/07/05(日) 18:48:42 ID:???
質問です。
物理の力学が苦手です。
よく、物理はイメージだと言われますが、よくわかりません。
公式を覚えていても使いどころが分からないなどいろいろあります。

どうか回答をお願いします。
716ご冗談でしょう?名無しさん:2009/07/05(日) 18:51:26 ID:???
で、質問は?
717ご冗談でしょう?名無しさん:2009/07/05(日) 18:54:30 ID:???
具体的には、力のモーメントがよくわかりません。関連して重心もわかっていません。
M=fl
でモーメントの式が与えられているのはわかっていますが、いまいち使いどころがつかめません。
718ご冗談でしょう?名無しさん:2009/07/05(日) 18:55:52 ID:???
力学はまず作図することから始まる
公式の使いどころが分からないなら使いどころが分かるまで繰り返しやるしかない
719ご冗談でしょう?名無しさん:2009/07/05(日) 19:04:04 ID:???
>>715
「公式を覚える」
物理、特に力学好きな奴が絶対口にしない言葉(自分調べ
それぐらい力学みたいなストーリー性のある分野には似つかわしくない表現だな。
多分テストの点の事考える前に勉強の仕方考えた方がいいと思う。

あんたが文系だったら上記の事は全てあてはまらないのは認める。
それは俺が歴史を暗記科目と思って勉強してたからだ。
歴史なんて理解しなくてもとりあえず問題が解ければいいと思っていた。
720ご冗談でしょう?名無しさん:2009/07/05(日) 19:27:38 ID:???
>>717
てこの原理がわからないと?
721ご冗談でしょう?名無しさん:2009/07/05(日) 19:33:34 ID:tNT3QFx3
>>713
>この考えは正しいのでしょうか?
間違っています。
722ご冗談でしょう?名無しさん:2009/07/05(日) 19:37:46 ID:???
>>713
いいよ、というか最初からゲージ変換はそういうものとして定義されている。
723ご冗談でしょう?名無しさん:2009/07/05(日) 20:03:22 ID:???
>>713
ゲージ変換の定義次第だろ。
724ご冗談でしょう?名無しさん:2009/07/05(日) 20:09:16 ID:???
r_{\mu \nu}って[γ_μ,γ_ν]じゃないよな?
725ご冗談でしょう?名無しさん:2009/07/05(日) 21:22:30 ID:???
>>713
ゲージ変換の定義を書いてみて
726ご冗談でしょう?名無しさん:2009/07/05(日) 23:06:05 ID:???
地磁気の伏角を求めるにはどういう計算をしたらいいんですか?
ネットや図書室の参考書で調べてもわからなかったので
よろしくお願いします。
727ご冗談でしょう?名無しさん:2009/07/05(日) 23:07:50 ID:???
天文板で暴れてた奴かな
728ご冗談でしょう?名無しさん:2009/07/06(月) 00:15:34 ID:uIuumRFW
713です。

ゲージ変換の定義は
Φ  :dirac 場
A_{\mu}:非可換ゲージ場
g=exp(-θ^{a}T_{a}) :Lie群の元
(表現 T_{a}は具体的には与えない.)
として、

Φ’=gΦ
A_{mu}'=gA_{\mu}g^{-1}+g d_{\mu}g^{-1}

です。
729ご冗談でしょう?名無しさん:2009/07/06(月) 00:16:48 ID:uIuumRFW
>r_{\mu \nu}って[γ_μ,γ_ν]じゃないよな?

んなわけがない
730ご冗談でしょう?名無しさん:2009/07/06(月) 00:22:39 ID:???
壁を押した場合の作用反作用は感覚的に分かるんですが
物を投げる場合の作用反作用がいまいちピンときません
簡単な説明でもいいのでお願いします
731ご冗談でしょう?名無しさん:2009/07/06(月) 00:25:37 ID:???
>>730
よく転がる台車の上に乗って、ボウリングのボールでも投げてみろ
732ご冗談でしょう?名無しさん:2009/07/06(月) 00:28:51 ID:uIuumRFW
713です。コメントどうもです!
ちと、修正します.

ゲージ変換の定義は
Φ  :dirac 場
A_{\mu}:非可換ゲージ場
g=exp(-θ^{a}T_{a}) :Lie群の元
(表現 T_{a}は具体的には与えない.)
として、

Φ’=gΦ
\bar{Φ'}=\bar{Φ}g^{-1}
A_{\mu}'=g A_{\mu}g^{-1} +g ∂_{\mu} g^{-1}

です。
733ご冗談でしょう?名無しさん:2009/07/06(月) 00:30:44 ID:???
>>731
あーなるほど
すっきりしました。レスどうもです
734ご冗談でしょう?名無しさん:2009/07/06(月) 00:46:46 ID:???
>>729
じゃあ\mu,\nuってローレンツの添え字じゃないのか?
ディラックスピノールの添え字か?
735734:2009/07/06(月) 00:48:06 ID:???
訂正
ローレンツ→4元ベクトル
736ご冗談でしょう?名無しさん:2009/07/06(月) 00:54:47 ID:???
移流拡散方程式を離散化して
BiCGStab法で数値的に解こうとしているのですが、
対角成分にゼロがあって、ILU(0)の前処理がうまくいきません。

なにかよい方法はないでしょうか。
737ご冗談でしょう?名無しさん:2009/07/06(月) 02:17:20 ID:???
問題の切り分けが出来てないのでこれじゃ誰も答えないでしょう

シューマイを作ろうとしていなげやに行ったのですがグリーンピースが売ってませんでした
どうしたらいいでしょう
みたいな質問なので
738ご冗談でしょう?名無しさん:2009/07/06(月) 05:36:39 ID:???
>>737
とりあえず、置換行列で対角要素に大きい値をもってきてから
前処理をしてみます。

それでだめなら、離散化の方法を再検討かな。
739ご冗談でしょう?名無しさん:2009/07/06(月) 06:39:24 ID:nLACsKf0
反重力できたから、古川電工さんに、テンプルフロートつくれ 人数揃えてください 電話かけとけ 絶対命令 DC
740ご冗談でしょう?名無しさん:2009/07/06(月) 07:37:13 ID:???
冷やした液体で、冷蔵庫 凍った固体で、冷凍庫を作れ 大気汚染が防げる
                                             登記
741ご冗談でしょう?名無しさん:2009/07/06(月) 09:38:16 ID:Bc5w6M+e
>>734,735
>じゃあ\mu,\nuって4元ベクトルの添え字じゃないのか?

そうです。

その場合、
Lie群の生成子はガンマ行列に対して
常に可換でなければならないことになりますが、
この考えは正しいのでしょうか?

というのが、私の質問です。

理由など教えて頂けると、大変うれしいです。

(それが非可換ゲージ変換の定義だと思うのは
やさしいのですが、それだと、一般に型が異なる
行列の積を計算していることになります。
それが可換だと言われても、ちょっと気持ちわるくて...
例:SU(2)の基本表現であるパウリ行列とガンマ行列との
積など.)

場の理論はどうも、テクニカルな計算が多い気がして
なやまされていますm(_ _)m

場の理論に詳しい方のアドバイスをお持ちしております。

742ご冗談でしょう?名無しさん:2009/07/06(月) 10:01:01 ID:???
>>741
直積空間に作用しているんだから、交換するよ。
たとえば、SU(2)の2次元表現(u,d)があって
u,dはそれぞれディラックスピノールだとする。

SU(2)の元 gは、
g (u, d) = (g11 u+g12 d, g21 u + g22 d)
みたいに作用し、
ローレンツ群の元 hは
h (u, d) = (hu, hd)
みたいに作用する。

つまり、ほんとは、gは×を直積としてg×1と書くべきだし、
hは1×hと書くべきなのね。このふたつはもちろん交換する。
743ご冗談でしょう?名無しさん:2009/07/06(月) 10:26:04 ID:rRbh2jV0
>>742
>直積
テンソル積が正しい。
744743:2009/07/06(月) 15:52:54 ID:???
>>743
失礼しました
745742:2009/07/06(月) 15:54:28 ID:???
ありゃりゃ、またミスった。すみません。
>>744の発言者は>>742です。
746ご冗談でしょう?名無しさん:2009/07/06(月) 16:41:43 ID:???
Nintendogs というゲームがあるんですけど、
もう2年近くやってないんです。
このゲーム、あまりにも飼い主が構ってくれないと、犬が家出してしまうんです。

いまはまだ、犬が家にいるのか、家出しているのか未確定な状態です。
ゲームカセットをDSにセットして、電源を入れた瞬間に状態が決まってしまいます。
でも家出してたらどうしようと思うとなかなか電源を入れられません。

これって主レディンガーの猫と同じですよね
747ご冗談でしょう?名無しさん:2009/07/06(月) 17:12:22 ID:???
昔テレビで、地球に一番近い恒星が超新星爆発を起こしたとしたら、
数年後に地球に届く衝撃波によって
地球の大気がはぎ取られてしまうだろう...

... といっていたのですが、
そもそも衝撃波は真空中でも伝わるんですか?
なにを媒体にしているのでしょうか?
748ご冗談でしょう?名無しさん:2009/07/06(月) 17:21:37 ID:???
>地球に一番近い恒星
そりゃ太陽ですからねえ。
衝撃波で地球の大気が剥ぎ取られるどころの騒ぎじゃないですよ。
あはは。
749ご冗談でしょう?名無しさん:2009/07/06(月) 17:21:43 ID:???
>>747
地球に一番近い恒星は太陽なので、数年後というのは明らかに間違い
750742:2009/07/06(月) 17:22:33 ID:???
>>747
最初に光速でやってくるのは可視光やガンマ線
ほぼ光速でやってくるのはニュートリノとか。

おくれてやってくる衝撃波は、普通の物質です。
星を構成していたガスが飛んでくる。
751ご冗談でしょう?名無しさん:2009/07/06(月) 17:25:31 ID:???
>>746
犬が猫と同じわけないだろう
752747:2009/07/06(月) 17:40:18 ID:???
間違えた 太陽系に一番近い恒星 でした。
753ご冗談でしょう?名無しさん:2009/07/06(月) 17:45:56 ID:???
細い棒の一端を原点とし、z軸の正の方向を鉛直上方と定める。
z軸と棒のなす角度をαとし、棒はz軸まわりを角速度ωで回転しているとする。
この棒にそって滑らかに動く質量mの質点の運動を調べる。
原点から質点までの距離をr、重力加速度の大きさをgとするとき、rに関する
運動方程式を書き下せ。

計算してみた結果、r''=r・(sinα)^2・ω^2-g・cosα
となったのですが間違っていないでしょうか?
よろしくお願いします。
754742:2009/07/06(月) 17:53:36 ID:???
>>753
いいじゃないかな。
755ご冗談でしょう?名無しさん:2009/07/06(月) 18:04:34 ID:???
>>754
そうでしたか。
どうもありがとうございました。
756ご冗談でしょう?名無しさん:2009/07/07(火) 01:47:25 ID:ggXSCkR8
物質に反物質をぶつければ対消滅してエネルギーになる説明がある。
では?

どこまで近づけばその対消滅となる?
真空で物質から分離して磁気などで中に浮かせればという話もあるが。
真空なら反応しないの?
距離が原子1個の幅でも離れていれば反応しないんじゃないの?
距離が陽子1個の幅でも離れていれば反応しないんじゃないの?
757ご冗談でしょう?名無しさん:2009/07/07(火) 01:56:17 ID:cZe2V1Jn
ヤング率を求めるクントの実験で、棒状試料の支点を棒の端から三分の一のところを固定した場合どうなりますか?理由も教えてください!
758ご冗談でしょう?名無しさん:2009/07/07(火) 02:10:30 ID:cZe2V1Jn
棒状試料を硬く持って摩擦すると音がしないのは、なぜですか?
759ご冗談でしょう?名無しさん:2009/07/07(火) 02:10:58 ID:???
>>756
場の量子論使って反応確率を計算すればおk
760ご冗談でしょう?名無しさん:2009/07/07(火) 02:12:01 ID:cZe2V1Jn
ガラス棒の固定端のコルク栓がゆるい場合,あるいはコルク栓の代わりにゴム栓をつけた場合はどうなりますか?
761ご冗談でしょう?名無しさん:2009/07/07(火) 02:26:20 ID:???
NGID:ID:cZe2V1Jn
762ご冗談でしょう?名無しさん:2009/07/07(火) 02:31:24 ID:???
>>643

遅くなりました。Aの方はそのままで規格直行関係を使えばいいのですか??
763ご冗談でしょう?名無しさん:2009/07/07(火) 03:49:36 ID:???
>>752 その恒星なら超新星爆発はしないから、安心して。
764ご冗談でしょう?名無しさん:2009/07/07(火) 06:46:33 ID:???
>>763
通常の恒星の進化過程を経ればね。
765ご冗談でしょう?名無しさん:2009/07/07(火) 07:02:40 ID:???
>>764
そうそう。モノリスの介入も心配しないとな
766ご冗談でしょう?名無しさん:2009/07/07(火) 07:09:36 ID:???
>>765
そんなことじゃなく、銀河のこのあたりじゃまず起きないだろう衝突型超新星のことが例外。

ちなみにモノリスは超新星爆発なんか起こしてない。
767ご冗談でしょう?名無しさん:2009/07/07(火) 08:03:46 ID:???
何と衝突スンの? 放浪中性子星?
それなら太陽も衝突しないか心配だな
768ご冗談でしょう?名無しさん:2009/07/07(火) 08:15:30 ID:???
>>767
太陽近傍の恒星密度なら心配する必要はない。

あくまで可能性の低い仮定の話。
769ご冗談でしょう?名無しさん:2009/07/07(火) 09:04:00 ID:???
要するに、太陽系近傍でもしも超新星爆発が起こったらっていう話でしょこれ。
アルファ・健太売りが実際に超新星爆発起こしそうだとかそういう事じゃなくて。
770ご冗談でしょう?名無しさん:2009/07/07(火) 09:06:38 ID:???
>>769
起こる可能性がないに等しい仮定だと、入り込めないんだよ。
771ご冗談でしょう?名無しさん:2009/07/07(火) 09:08:56 ID:???

>>767  白色矮星でもBHでも中性子星でも。
772ご冗談でしょう?名無しさん:2009/07/07(火) 09:15:34 ID:jcWe42bI
シリウスBってもう直き超新星爆発するんじゃなかった?
773ご冗談でしょう?名無しさん:2009/07/07(火) 09:17:47 ID:???
>>772

白色矮星が超新星爆発?バカ?
774ご冗談でしょう?名無しさん:2009/07/07(火) 09:27:43 ID:???
>>772
釣られないぉw
775ご冗談でしょう?名無しさん:2009/07/07(火) 09:32:06 ID:???
ベテルギウスと間違えてるんだろ
776ご冗談でしょう?名無しさん:2009/07/07(火) 09:33:55 ID:???
シリウスAと間違えた訳じゃないんだw
777ご冗談でしょう?名無しさん:2009/07/07(火) 09:34:47 ID:???
ここ天文板じゃねえ
778ご冗談でしょう?名無しさん:2009/07/07(火) 09:35:26 ID:???
あれ、アンタレスって既に超新星爆発したって聞いた気がするけど知りようが無いよね実際は
779ご冗談でしょう?名無しさん:2009/07/07(火) 09:36:40 ID:jcWe42bI
>>776
連星系の白色矮星はIa型超新星して死ぬんだよ。
780ご冗談でしょう?名無しさん:2009/07/07(火) 09:39:40 ID:???
>>779
そうか、降着ガスが溜まって限界超えたらあぼーんしちゃう奴かな
781ご冗談でしょう?名無しさん:2009/07/07(火) 09:40:59 ID:???
それこそ釣りだな
782ご冗談でしょう?名無しさん:2009/07/07(火) 09:53:08 ID:???
まじ、ヤバいじゃん。
783ご冗談でしょう?名無しさん:2009/07/07(火) 10:01:46 ID:???
A×(B×C)=B(A・C)−C(A・B)
(A、B、Cはベクトル)

この式で「×」は外積 「・」は内積
でも右辺のBと(A・C)やCと(A・B)の間はどうなってるの?

今まで何も書いてないから内積だと思ってたけど内積だと(右辺)=0になるんだけど

784ご冗談でしょう?名無しさん:2009/07/07(火) 10:08:43 ID:???
>>783
内積はスカラー
785ご冗談でしょう?名無しさん:2009/07/07(火) 10:17:56 ID:???
>>784
ありがとう、言われて気づいたわ
こんな質問して恥ずかしい

今日は自主休講にして1日休む
786ご冗談でしょう?名無しさん:2009/07/07(火) 10:48:06 ID:VKQ36ksV
>>750
大丈夫か?重力波は光より速いぞ。衝撃波も速いものもある
787ご冗談でしょう?名無しさん:2009/07/07(火) 10:52:53 ID:???
スルーで
788ご冗談でしょう?名無しさん:2009/07/07(火) 11:19:00 ID:???
担当教授に聞いてみなさい。
789ご冗談でしょう?名無しさん:2009/07/07(火) 12:29:36 ID:???
786は一般相対論から導かれる重力波とは別の概念について語っているらしいw
790ご冗談でしょう?名無しさん:2009/07/07(火) 12:35:45 ID:???
釣られんなよw  >>俺もだ
791ご冗談でしょう?名無しさん:2009/07/07(火) 12:39:17 ID:???
  ┌──────────────────
─┼○ 変な俺理論厨が去りますように
  └──────────────────
792ご冗談でしょう?名無しさん:2009/07/07(火) 12:39:29 ID:???
でも、太陽の光って中心から出てくるのに200万年かかるっていう話あるじゃん。
それから類推すると、超新星爆発が観測される数百万年前にニュートリノの異常
増大とかが観測されるんだろうね。それで爆発が観測される前に分かると。
793ご冗談でしょう?名無しさん:2009/07/07(火) 13:30:35 ID:???
>>792
太陽内部で紆余曲折しながら出てくる光の速度を、真空中の光速と同様に
「これより速いものはないっ!」と勘違いしているのか??
まさか、超新星爆発が200万年かけてゆっくり恒星表面に出てくると
思っているとか??

超新星爆発なんて、始まったらあっという間。
794ご冗談でしょう?名無しさん:2009/07/07(火) 13:35:06 ID:???
また釣れた
795ご冗談でしょう?名無しさん:2009/07/07(火) 13:48:26 ID:???
釣りだったということにしないと精神の平静が保てないらしい。
796ご冗談でしょう?名無しさん:2009/07/07(火) 13:58:57 ID:???
自分で釣れたって言う奴は、釣りだったと言い訳して恥ずかしさを反らしていることが丸わかりだな。
797ご冗談でしょう?名無しさん:2009/07/07(火) 14:31:49 ID:???
【口封じ?】鳩山事務所「秘書は交通事故に遭い、現在生死の境を彷徨っており面会謝絶となっております」
http://tsushima.2ch.net/test/read.cgi/news/1246922364/
798ご冗談でしょう?名無しさん:2009/07/07(火) 17:34:03 ID:???
質問です。一次元調和振動子の固有関数を用いて、xの行列要素って計算可能ですよね??
このxって何でも良いんですか??
つまりどんな行列でも調和振動子の固有関数を用いれば、要素を計算できますか??

それとも何か条件がいりますか??
799ご冗談でしょう?名無しさん:2009/07/07(火) 18:04:28 ID:???
>>798
できる。
別に完全系を張ってなくても、内積の定義さえあれば
行列要素
<g|A|f>
は計算できる。
800ご冗談でしょう?名無しさん:2009/07/07(火) 20:32:27 ID:???
>>799
ありがとうございます。
801ご冗談でしょう?名無しさん:2009/07/07(火) 20:44:46 ID:???
遮蔽効果が完璧な際の電子の持つエネルギーを求める式として
E=-(z-N)^2/n^2×定数
(n=量子数、Z=元素番号、N=既に入っている電子数)という
式を与えられたのですが、遮蔽効果を考える時に最外殻の内側の軌道にある
電子全てが影響すると考えるんでしょうか?それとも1つ内側の殻のみと考えるんでしょうか?
Naを例にするとNに入れるのは8か10かで困っています。どなたか解りますか?
802ご冗談でしょう?名無しさん:2009/07/07(火) 21:20:13 ID:???
最外殻電子は遮蔽するけど内殻電子は遮蔽できない
と思う感覚が分からない
あと遮蔽は >>完璧 なんでしょ?
803ご冗談でしょう?名無しさん:2009/07/07(火) 21:55:20 ID:???
>>802
遮蔽効果は完璧となっています。と言うことはN=10でいいんでしょうか?
Mgの場合もN=10ということですか?
804ご冗談でしょう?名無しさん:2009/07/07(火) 22:13:30 ID:xEAln6OR
科学ニュース+での議論は非常に勉強になります。

【技術】光速を超える電波の送信装置の開発に成功
http://gimpo.2ch.net/test/read.cgi/scienceplus/1246501723/
805ご冗談でしょう?名無しさん:2009/07/07(火) 23:26:24 ID:???
等価原理のエレベータの説明がありますが、エレベータの中でも上と下で重量に違いがあるか計れば、重力か慣性力かはわかるんじゃないでしょうか?
806ご冗談でしょう?名無しさん:2009/07/07(火) 23:30:49 ID:???
>>805
等価原理ってのは、外の見えない大きさのないエレベータ内では
重力と慣性力は区別できないということですから。

大きさのあるエレベータを考えれば、そりゃ区別できちゃいます。
807ご冗談でしょう?名無しさん:2009/07/07(火) 23:32:08 ID:???
ヒント:局所的に
808ご冗談でしょう?名無しさん:2009/07/07(火) 23:47:14 ID:???
>>805
おっしゃる通りだよw
エレベータと言うのは言葉の綾だ。
厳密には質点のような体積無限少のエレベータじゃないと
等価原理は成り立たない。俺も高校の先生にあんたと同じ
質問ぶつけて笑われた苦い記憶があるぜw
809ご冗談でしょう?名無しさん:2009/07/07(火) 23:48:36 ID:???
誤字スマソ:少→小
810ご冗談でしょう?名無しさん:2009/07/07(火) 23:51:40 ID:???
局所的には区別がつかないが、有限の領域では区別がつく、という
ことこそが一般相対論の基礎になってる。
811ご冗談でしょう?名無しさん:2009/07/08(水) 00:02:02 ID:ox481ct+
教科書での話しなんですが
(d^2x/dt^2+x)
にdx/dtを掛けて変形すると
d/dt((dx/dt)^2/2+x^2/2)
となるのがわかりません。
812ご冗談でしょう?名無しさん:2009/07/08(水) 00:06:56 ID:???
下の式計算すればいいだけじゃん
813ご冗談でしょう?名無しさん:2009/07/08(水) 00:09:09 ID:???
d/dt((dx/dt)^2/2+x^2/2)を計算したら(d^2x/dt^2+x)(dx/dt)になるから
814ご冗談でしょう?名無しさん:2009/07/08(水) 00:26:05 ID:???
805です、806-810、ありがとうございました。
815ご冗談でしょう?名無しさん:2009/07/08(水) 00:43:56 ID:???
>>803
なんで最内殻の電子を考えないという発想になるのか分からんが
遮蔽と言ったら普通は内側の電子全ての分の影響を考えるね
だからNa、Mgの場合は両方ともN=10
816ご冗談でしょう?名無しさん:2009/07/08(水) 01:38:46 ID:???
オリフィス構造について教えて下さい。

流れる方向 ⇒ABA'⇒

A、A'は大きい
Bは小さい

この場合、流れる流速はA=A' or B=A'?

そしてその訳も教えて下さい。
817ご冗談でしょう?名無しさん:2009/07/08(水) 01:45:16 ID:???
こんな文章で自分の疑問点が相手に伝わって
誰か答えてくれるとでも思ってるのか
818ご冗談でしょう?名無しさん:2009/07/08(水) 01:48:46 ID:???
>>816
「私の説明不足の用語・概念はググってみて下さいね、回答者のみなさん(はぁと」
と書くのも面倒くさかったようだな
819ご冗談でしょう?名無しさん:2009/07/08(水) 01:54:32 ID:???
>>816
板違い。こんどから機械・工学板で質問すること。
A'がオリフィス板に近いと大体Bの流速に等しい。つまり流れは流路断面のすべてを
満たしているわけではない。こういう流れを噴流という。
厳密に言うとオリフィス板の直後では断面Bよりもさらに流れは細くなり流速は大きくなる。
噴流は周りの流体とエネルギーの交換をして圧力は低下する。
820ご冗談でしょう?名無しさん:2009/07/08(水) 04:17:07 ID:???
何で相対速度は光速を越える事はないのですか??(例えば0.5cと0.6cの相対速度は1.1cにならないのはなぜ?)中学、高校生にも分かるように誰か教えて下さい。
821ご冗談でしょう?名無しさん:2009/07/08(水) 05:43:01 ID:167ugBQQ
長さL、慣性モーメントJの導体棒の一端が原点Oに固定され、xy平面をω。で回転している。
導体棒のもう一端に接触するように半径Lの円形導体リングが設置されており、リングと原点Oの間に抵抗Rが接続されている。
t=0でz軸方向に磁束密度Bの磁界をかける。
導体棒のその後の回転角速度ωを求め、グラフに示せ。

という問題なのですが、まず棒に生じる起電力V=(BωL^2)/2から流れる電流I=V/Rを求め、棒に生じる力F=IBLを求めたのち、回転の方程式
I(dω/dt)=−N(Nはトルク)を解いて答えを求めるという流れだと思いますが、トルクの出し方に悩んでいます。
微小な長さに生じる力のトルクdN=xdF=IBx・dxを積分して求めるのでしょうか、N=LFで求めるのでしょうか?
822821:2009/07/08(水) 06:01:20 ID:167ugBQQ
すみません。解決しましたm(__)m
823ご冗談でしょう?名無しさん:2009/07/08(水) 06:59:57 ID:???
>>820
理系なら中学生・高校生でも特殊相対論を理解する事はさほど難しくない

>中学、高校生にも分かるように誰か教えて下さい
しかしこの様な事を言う場合はほぼ不可能と思った方が良い。(今後も)
・数式をさほど使わない啓蒙書を既に多数読んでいるはず (まさか読んでないとかw
・特殊相対論の根幹の概念を掲示板の数レスで理解出来ると本気で考えている
(まさか、本気で質問していないとかw

子供の頃沢山の啓蒙書を読んだが疑問が晴れる事は無かった。
数式を使わないで本当の意味の物理学の体系的理解は有り得ないと理解するまでに
俺も大分時間がかかった。
あきらめた方が良いから、良い小説を沢山読んで将来文系に進みなさい。
824ご冗談でしょう?名無しさん:2009/07/08(水) 07:05:51 ID:???
>>820
一応質問に真正面から答えておくと、【観測的事実】だ
これで納得してもらえる可能性が殆ど無いと判断したうえでの回りくどいレスになったのだ。
825ご冗談でしょう?名無しさん:2009/07/08(水) 08:24:11 ID:???
>>820
じゃあ俺も真正面から答えよう。

相対速度を求める公式は(u+v)/(1+uv/c^2)だ。

何、学校で(u+v)と習っただと。あれは近似だ近似。πを3.14とか「およそ3」
とか習うのと一緒だ。日常レベルでは(u+v)でも(u+v)/(1+uv/c^2)でも、
大して差がなくて、かつ(u+v)の方が簡単だからそう教えることになっている。
だが実際は相対速度の公式は(u+v)/(1+uv/c^2)だ。

この式で計算すれば、(0.5c+0.6c)/(1+0.5*0.6)=(1.1/1.3)cだ。ほら光速を
越えない。

何「そんな式常識外れだ。直観に合わない」だと。おまえの常識や直観は、
光速に比べて遅い速度で動いたことがないという狭い経験から作られた
ものだ。そういうことは光速に近い速度で動き回ることという経験を積んだ
後でもう一度考えろ。
826ご冗談でしょう?名無しさん:2009/07/08(水) 08:31:38 ID:???
>>825
式だけ示して「コレが正しい、そうなっている」と言っても説得力はないわな。

その式が導出される理由を説明しないとな。

ぶっちゃけ、知らない人間より痛い。
827825:2009/07/08(水) 08:50:31 ID:???
>>826
それが、俺自身頭の痛いことに、今時のゆとりさんはこの程度で「へぇ〜」
ボタン叩いて納得しちゃうのが多いんだよ。
説明している俺の方は納得いかないよ、もちろんこれじゃ。

828ご冗談でしょう?名無しさん:2009/07/08(水) 08:59:30 ID:???
でも論理的にこの流れで正しいでしょ。納得いかないじゃなくて。
観測的事実として>826の速度合成則が受け入れられて、従来あた
りまえとして考えられていた合成則がV<<Cでの近似式でしかなかった
という事が分かった訳だし。様々な歴史的実験をここで紹介する程
スペースがある訳でもないし。十分でしょ。
829ご冗談でしょう?名無しさん:2009/07/08(水) 09:19:34 ID:???
>>826
「何でその式が成り立つかは知りたきゃ教科書読め」ってだけのことだと思うが。
830ご冗談でしょう?名無しさん:2009/07/08(水) 09:26:24 ID:???
>>829
超人的な読解力だな。

予備知識ないと仮定して825読んで、そう思えるのか。
そうは書いてないだろう。

正解を知ってる人間が825読んで、うんうんと思うのはいいけどね。
831825:2009/07/08(水) 09:53:21 ID:???
>>830
>>823,>>824が既にある前提で>>825を書いたわけなんだがな。
その上で、「どうして相対速度はu+vじゃないんだ」という質問に真っ正面から
答えると>>825になる。「その理由は?」になんかあるらしいことは>>823,>>824
臭わせてくれているわけだし。

実際に中学生に説明する時はマイケルソン・モーレーの実験あたりから
ちゃんと説明するがね。

832ご冗談でしょう?名無しさん:2009/07/08(水) 12:47:20 ID:???
マイケル・ジャクソンの亡霊がどうのこうのって話かw
833ご冗談でしょう?名無しさん:2009/07/08(水) 14:53:14 ID:gCmhYuFh
生命の誕生なんかで使われる例えで卵が先か?ニワトリが先か?なんて質問ありますよね?
それみたいに水が先か?酸素が先か?だったらどっちが先できたんですか?
834ご冗談でしょう?名無しさん:2009/07/08(水) 14:56:53 ID:???
>>833
地球の話
・酸素より水が先(大昔の大気には酸素は殆ど無かったのだ)
ちなみに宇宙の話
・酸素より水素の方が先(宇宙出来た頃は酸素は無かった。水素とヘリウムだけ)
835ご冗談でしょう?名無しさん:2009/07/08(水) 15:00:01 ID:???
語弊を指摘される前に補足
・酸素より水が先→酸素(気体の)より水が先
に訂正
836ご冗談でしょう?名無しさん:2009/07/08(水) 15:00:09 ID:???
>>833
酸素が先。酸素は恒星の内部か、超新星爆発でできる。
水分子なんかもっと温度の低いトコじゃないとできない。
837ご冗談でしょう?名無しさん:2009/07/08(水) 15:01:05 ID:???
>>834
> ちなみに宇宙の話
すり替えよくない。
838ご冗談でしょう?名無しさん:2009/07/08(水) 15:01:59 ID:???
>>836
酸素原子はそうだろうけど、酸素分子だとどう?
839ご冗談でしょう?名無しさん:2009/07/08(水) 15:07:23 ID:???
>>838
実際に分子ができるのは、超新星爆発後の冷えた宇宙空間なんだよね。

圧倒的な割合の水素と、ごく微量の酸素。
酸素原子同士が結合するより、水分子ができり確率の方が高いだろ。
840ご冗談でしょう?名無しさん:2009/07/08(水) 15:43:31 ID:gCmhYuFh
>>833です
分子と原子でまた変わるようなんですね(^_^;)
重力と物質ではどちらが先ですか?
841ご冗談でしょう?名無しさん:2009/07/08(水) 15:54:48 ID:???
物質は本来は質量持ってない存在だからゴニョゴニョしたいけど、やっぱ同時でいいんでしょう
842ご冗談でしょう?名無しさん:2009/07/08(水) 16:03:21 ID:???
質量とエネルギーは等価らしいけど、電荷とエネルギーは等価?
等価なら電荷を質量に変えることも可能なの?
843ご冗談でしょう?名無しさん:2009/07/08(水) 16:06:06 ID:???
>>842
NO
844ご冗談でしょう?名無しさん:2009/07/08(水) 19:55:09 ID:bc6q6NEQ
C(コンデンサー)が直列でL(コイル)とR(抵抗)が並列になってる合成交流回路のインピーダンスをおしえてください。
複素電圧法でときたいのですが。
845ご冗談でしょう?名無しさん:2009/07/08(水) 19:57:17 ID:???
846めろん:2009/07/08(水) 19:59:24 ID:bc6q6NEQ
C(コンデンサー)が直列でL(コイル)とRが並列の合成交流回路のインピーダンスを教えてください。
複素電圧法でときたいのですが。
847めろん:2009/07/08(水) 20:05:22 ID:bc6q6NEQ
R(1-ω^2LC)+jωLになるらしいことはわかっているのですが。
自分で計算したのはは1/jωC+1/jωL+1/Rとなります。
848ご冗談でしょう?名無しさん:2009/07/08(水) 20:09:55 ID:???
>>847
お前の回答は次元がめちゃくちゃだろ、論外
849ご冗談でしょう?名無しさん:2009/07/08(水) 23:00:19 ID:???
>>816

マルチポスト市ねチンカス

もの凄い勢いで誰かが質問に答えるスレ@理系板@42
http://science6.2ch.net/test/read.cgi/rikei/1240551892/775
850ご冗談でしょう?名無しさん:2009/07/09(木) 00:11:25 ID:++ZqMLEm
X=πR 
851ご冗談でしょう?名無しさん:2009/07/09(木) 00:23:05 ID:???
>>846
R1が直列で、R2とR3が並列の
直流回路の合成抵抗Rはわかるのか?
852ご冗談でしょう?名無しさん:2009/07/09(木) 01:02:57 ID:???
スピンについて質問です

ttp://imagepot.net/image/124706782168.jpg

演算子s_zから固有ベクトルを求めた場合、
昇降演算子はs_±=s_x±i*s_yとおいて計算は↑の画像のようになりますが、
演算子s_xから固有ベクトルを求めた場合(固有ベクトルα,β=1/√2*(1,±1)となると思います)、
昇降演算子はどのようにおけばいいですか?
サイクリックにs_y±i*s_zと置いてみましたがうまくいかないみたいです
昇降演算子というのはどういう風に決めるものなんでしょう?
教科書は皆天下り的なので・・・
また、一つの演算子の固有ベクトルから他の演算子の期待値(例えば<α|s_y|α>)を求めると0になるのは
どういう意味があるのでしょう?
853ご冗談でしょう?名無しさん:2009/07/09(木) 01:46:30 ID:???
>>852
サイクリックに回せばうまく行くはずだが。
代数
[s_x, s_y±is_z] = ±(s_y±is_z)
が成り立つからね。
昇降演算子は、この関係が成り立つようにできてる。

最後の2行は、角運動量の対応物だと思えば自然かな。
z軸を軸として回ってる剛体の角運動量のy成分はゼロということ。



854ご冗談でしょう?名無しさん:2009/07/09(木) 01:53:16 ID:F3abZefw
崩壊分岐比ってどうやって計算するんだ?
855ご冗談でしょう?名無しさん:2009/07/09(木) 01:53:58 ID:???
ちょっとやってみたけど位相因子がつくのかもね
>>853の関係だけじゃ決まらない部分の

一番確実なのはWignerのD関数で演算子と状態を両方回すのかなあ
856ご冗談でしょう?名無しさん:2009/07/09(木) 02:02:30 ID:???
計算するとs_xの昇降演算子はs_+=s_z-isy,s_-=s_z+i_s_yになるな
857ご冗談でしょう?名無しさん:2009/07/09(木) 05:14:31 ID:???
ちょっとした疑問なのですが
光電効果で、当てる光子のエネルギーhνが一定以上じゃないと電子が金属から飛び出さないと聞いたのですが
それでも電子にエネルギーを与えた光子は消滅してるのですよね。
だったら飛び出さないまでも内部の電子にエネルギーが溜まって、光子を当て続ければいずれ飛び出すように思うのですが
↑の事は光子が波だけの性質を持つなら起こり得、事実そうでない事が粒子性の根拠となってるようですが・・・

素人で的外れな質問してるかもしれませんがどうかお願いします
858ご冗談でしょう?名無しさん:2009/07/09(木) 05:28:39 ID:???
電子が飛び出すようなエネルギーじゃない波は、何も無かったかのように通り過ぎる。
電子には何の影響も与えない。弱い影響を与えるなんてことはなくて、与えるか与えないか。
859ご冗談でしょう?名無しさん:2009/07/09(木) 05:47:49 ID:???
電子をちょっと励起させて、電子は電磁波放ってもとの位置に戻る
つまり光は反射してるんじゃないの
860ご冗談でしょう?名無しさん:2009/07/09(木) 05:48:56 ID:???
なるほど、そうなんですね。ありがとうございます
「波」とありますが、このときは波動性の側面が出てきて
電子にエネルギを与えるときは粒子性が出てくるということですか?
861ご冗談でしょう?名無しさん:2009/07/09(木) 05:51:34 ID:???
>>859
弱すぎて吸収されずに弾いてるみたいな感じですか。
解答ありがとうございます
862ご冗談でしょう?名無しさん:2009/07/09(木) 09:41:14 ID:???
準静的断熱過程においてPとTの関係とPとVの関係教えて下さい。
863ご冗談でしょう?名無しさん:2009/07/09(木) 09:49:47 ID:jdVdI3gA
いいよ
864ご冗談でしょう?名無しさん:2009/07/09(木) 10:03:26 ID:???
>>863
式で関係を表すとどうなりますか??
865ご冗談でしょう?名無しさん:2009/07/09(木) 10:47:34 ID:???
>>853>>855>>856
ありがとうございます
>>856としたらうまくいきますね
866ご冗談でしょう?名無しさん:2009/07/09(木) 11:43:14 ID:zGnJKlMb
>>841
ゴニョゴニョ詳しく教えてください
867ご冗談でしょう?名無しさん:2009/07/09(木) 14:08:04 ID:???
>>866
回答者のレヴェルを遙かに越える素粒子論の話だからゴニョゴニョにしといたのだorz
868ご冗談でしょう?名無しさん:2009/07/09(木) 14:12:47 ID:???
>>864
状態方程式は? 内部エネルギーの式は?

しかし、準静的断熱過程という言葉を知っているなら、
それを学んだ本に計算法がのってるだろうに。
869ご冗談でしょう?名無しさん:2009/07/09(木) 18:00:39 ID:???
>>862
dU(S(T, V), V)=PdV
870ご冗談でしょう?名無しさん:2009/07/09(木) 19:35:40 ID:???
>>867
俺はおまえ以上のレヴェルだからゴニョゴニョ言ってみろよw
871ご冗談でしょう?名無しさん:2009/07/09(木) 19:43:45 ID:???
>>870
レス良く嫁。質問者のレヴェルじゃない。俺のレヴェルだw
872ご冗談でしょう?名無しさん:2009/07/09(木) 20:18:50 ID:???
だから俺はおまえ以上のレヴェルだからゴニョゴニョの部分言えと
自分で分からないからゴニョゴニョにしてんだろw
873ご冗談でしょう?名無しさん:2009/07/09(木) 20:58:36 ID:???
>>872
オマエは少なくとも国語のレベルは相当低いな。
874ご冗談でしょう?名無しさん:2009/07/09(木) 21:52:42 ID:???
私のためにケンカしないで!
そして誰か質問に答えてください
875ご冗談でしょう?名無しさん:2009/07/09(木) 23:05:33 ID:???
いいよ
876ご冗談でしょう?名無しさん:2009/07/10(金) 15:31:20 ID:EaKvRYlQ
カメハメ波を人工的に作る事は可能ですか?
877ご冗談でしょう?名無しさん:2009/07/10(金) 15:39:49 ID:???
不可能
似たような事する中国人のおっさんなら居る
878ご冗談でしょう?名無しさん:2009/07/10(金) 15:42:50 ID:???
だったら可能なんじゃね
879ご冗談でしょう?名無しさん:2009/07/10(金) 15:43:49 ID:???
>>876
かめはめ波がそもそも何なのか分からんのだから作り様がない。
880ご冗談でしょう?名無しさん:2009/07/10(金) 16:48:41 ID:???
>>873
何でもいいからはやくゴニョゴニョ部分書けよカス
881ご冗談でしょう?名無しさん:2009/07/10(金) 17:16:30 ID:???
まだいたのか、粘着バカ。傍で見てると痛いんだよ。
882ご冗談でしょう?名無しさん:2009/07/10(金) 17:23:57 ID:???
>>876
陽電子砲とかレールガンとかでいいなら…
883ご冗談でしょう?名無しさん:2009/07/10(金) 17:39:52 ID:???
え?手から何か出す話じゃないの?
884ご冗談でしょう?名無しさん:2009/07/10(金) 18:37:56 ID:Zthd0cF1
磁化の定義がわからないんですけど、ウィキペディアを見たところ

ある場所の磁化 M は、そこの周囲に物質が何も無い真空であったと仮定した際の磁場 H0 (これはその場所の磁束密度 B を真空の透磁率 μ0 で割ったものに等しい)
から実際に観測された磁場 H を差し引いたものである。

とあり、ということは実際に観測される磁場Hの方が大きと 磁化Mはマイナスになりますよね??

でも磁化がマイナスになるのは、反磁性の物のときなんですよね?ということは観測されるHは小さくなりますよね?
さっぱりです(´;ω;`)
885ご冗談でしょう?名無しさん:2009/07/10(金) 18:41:01 ID:???
>>884
ここに書かなくていいからベクトルを図示化して
886ご冗談でしょう?名無しさん:2009/07/10(金) 18:46:42 ID:???
最小作用原理っていうのは一種の仮説ですか?
887ご冗談でしょう?名無しさん:2009/07/10(金) 18:47:10 ID:???
>>881
だからはやく書けやヴォケ死ね
888ご冗談でしょう?名無しさん:2009/07/10(金) 18:48:08 ID:???
そのとおりですが何か
889ご冗談でしょう?名無しさん:2009/07/10(金) 18:50:52 ID:???
>>887
おまえ日本語通じんの?「傍で見てると」て書いてあんだろ。

相手構わずか。ファビョるなよ、バーカ。
890ご冗談でしょう?名無しさん:2009/07/10(金) 18:58:47 ID:???
>>889
馬鹿はてめーだ
釣られてる時点でおまえしかいねーんだよ
シッタカのゴニョゴニョ野郎
891ご冗談でしょう?名無しさん:2009/07/10(金) 19:15:55 ID:???
>>886
「原理」や「法則」というのは他の定理等から導き出されない実験的・観測的事実を言うのだ
892ご冗談でしょう?名無しさん:2009/07/10(金) 19:18:55 ID:???
ある体系では原理でも、それを包含するより高位の理論体系では説明可能な定理であったりする
893ご冗談でしょう?名無しさん:2009/07/10(金) 19:29:48 ID:???
物理スレでバカが自演
894ご冗談でしょう?名無しさん:2009/07/10(金) 19:46:39 ID:???
2chの物理板は物理じゃなく哲学とか宗教
いかれた子供、ただのシッタカおっさん、宗教高校・大学生、6流大学の物理かじりとかが鼻高々w
895ご冗談でしょう?名無しさん:2009/07/10(金) 19:49:31 ID:???
そういう物理板が好きなんです><
896ご冗談でしょう?名無しさん:2009/07/10(金) 19:50:49 ID:???
新たな宗教でも作るつもりかよw
897ご冗談でしょう?名無しさん:2009/07/10(金) 19:58:03 ID:???
>>894
当たり前だろw
最前線で物理やってる職員や、一日中勉強してる一流大の勤勉学生が俺ら
みたいに一日中スレに張り付いてられねぇだろw
昔とった杵柄とか、学生時代は物理得意だったのに何故今フリーターなんだ
ろ、みたいな境遇の俺らだからアフォ質問者の相手できんだよw
俺だって本職の人に間違い指摘されたら直ぐに謝って訂正する事にしてるぜ
898ご冗談でしょう?名無しさん:2009/07/10(金) 20:07:13 ID:???
まあまあ、おちけつ。
難しい問題は一流の大先生にお任せして
またーり物理を楽しめたらそれでいいじゃん。
899ご冗談でしょう?名無しさん:2009/07/10(金) 20:11:15 ID:???
  ∧_∧
 ( ´∀`) おまえら
 /,   つ   おちけつ
(_(_, )
  しし'
900ご冗談でしょう?名無しさん:2009/07/10(金) 20:12:08 ID:???
♪ゴーニョ ゴーニョ ゴニョ さかなの子青い海からやってきた ♪♪ ゴーニョ ゴーニョ ゴニョ
   ふくらんだまんまるおなかの女の子 ♪♪ ペータペタ ピョーンピョン 足っていい
  なかけちゃお! ♪♪ ニーギニギ ブーンブン おててはいいなつないじゃお! ♪
  ♪ あの子とはねると 心もおどるよ パークパクチュッギュッ!パークパクチュッギュッ! ♪
  ♪ あの子が大好き まっかっかの ♪♪ ゴーニョ ゴーニョ ゴニョ さかなの子青い海から
  やってきた ♪♪ ゴーニョ ゴーニョ ゴニョ ふくらんだまんまるおなかの女の子 ♪
  ♪ フークフク いいにおい おなかがすいた 食べちゃおう!♪♪ よーくよく 見てみよう
  あの子もきっと 見ている ♪♪ いっしょに笑うと ホッペがあついよ   ワークワク
  チュッギュッ!ワークワクチュッギュッ! ♪♪ あの子が大好き まっかっかの ♪♪
  ゴーニョ ゴーニョ ゴニョさか./ ̄ ̄ ̄ ̄ ̄ ̄ ̄\なの子 崖の上にやってきた ♪♪
\ゴーニョ ゴーニョ ゴニョ女の|  うるさい黙れ  |子まんまるおなかの元気な子 ♪
   ̄ ̄ ̄ ̄ ̄ ̄ ̄ ̄ ̄ ̄ ̄ ̄ \          / ̄ ̄∨ ̄ ̄ ̄ ̄ ̄ ̄ ̄ ̄ ̄ ̄ ̄
                      ̄ ̄ ̄ ̄V ̄ ̄    (゚д゚ )
                          <⌒/ヽ-、__ノヽノ |
                        /<_/____/ < <
                         ̄ ̄
・・・おまえら、喧嘩はやめれ
901ご冗談でしょう?名無しさん:2009/07/10(金) 20:18:41 ID:???
>>898
うん落ち着くぉ@897
902ご冗談でしょう?名無しさん:2009/07/10(金) 20:39:46 ID:???
どうでもいいわ
>>841
まじでゴニョゴニョ説明しろよ
903ご冗談でしょう?名無しさん:2009/07/10(金) 20:54:44 ID:???
禁則事項です
904ご冗談でしょう?名無しさん:2009/07/10(金) 20:57:23 ID:???
ちょっとビール吹きかけた
905ご冗談でしょう?名無しさん:2009/07/10(金) 22:49:21 ID:U28PlXUr
どなたか宇宙空間で物質と重力どっちが先に出来たかわかる方いませんか?
ゴニョゴニョなしで
906ご冗談でしょう?名無しさん:2009/07/10(金) 22:52:23 ID:???
ボソンやらフェルミオンやらの物質の誕生の後に、GUT力と重力が別れたんだよ
907ご冗談でしょう?名無しさん:2009/07/10(金) 22:53:17 ID:???
だから同時だって言ってんじゃん
908ご冗談でしょう?名無しさん:2009/07/10(金) 22:56:24 ID:???
>>906
重力が分かれる前には重力は無かったっていう考え方した方がいいのか?
4つの力には区別が無かったつまり一つだったtって事で、その一つの力は
重力も兼ねてたわけだし。
909ご冗談でしょう?名無しさん:2009/07/10(金) 23:01:10 ID:???
場の量子論のかなり初歩的な部分が分からないです。
力学変数q、pを演算子化するのが量子力学で
それと同じように、場ψを演算子化するのが場の量子論、というところまでは分かったのですが
例えば、シュレディンガー場
ψ=Σa_k(t)e^{ik・q}
で、a_k(t)を演算子化すると、ψも演算子になるのは分かるのですが
平面波の中にある変数、qは演算子なのでしょうか?ただの数なのでしょうか?
教えてください。
910ご冗談でしょう?名無しさん:2009/07/10(金) 23:22:09 ID:???
ベクトルの添字のようなもの。
911ご冗談でしょう?名無しさん:2009/07/10(金) 23:41:09 ID:???
>>906
数学的には
GUT力はゲージ場(接続)だけど
重力は、計量テンソルだよね。
かなり異質なものに見えるのだが。

重力を他の力と統一しようとしている人は
そのあたりをどう考えているの?
912ご冗談でしょう?名無しさん:2009/07/10(金) 23:44:18 ID:???
>>911
>重力は、計量テンソルだよね。
少しはお勉強してね。
913ご冗談でしょう?名無しさん:2009/07/10(金) 23:45:41 ID:???
>>912
何を勉強したらいい?
914ご冗談でしょう?名無しさん:2009/07/10(金) 23:49:56 ID:???
位置と運動量の不確定性とエネルギーと時間の不確定性
は意味あいが違うんですか?
915ご冗談でしょう?名無しさん:2009/07/11(土) 00:00:18 ID:???
>>911
内山龍雄に叱られるぞ
916ご冗談でしょう?名無しさん:2009/07/11(土) 00:44:55 ID:???
>>915
SO(3,1)ゲージ理論とみる、ということ?
4つ足があったり、ふつうのゲージ理論とは
ずいぶん違うように思うのだが。
917ご冗談でしょう?名無しさん:2009/07/11(土) 01:03:18 ID:???
>>916
内山龍雄に叱られるぞ
918ご冗談でしょう?名無しさん:2009/07/11(土) 01:16:24 ID:???
>>971
どういう点が叱られるのか、具体的に書いてくれないか。
919ご冗談でしょう?名無しさん:2009/07/11(土) 09:31:55 ID:???
ロングパス乙
920ご冗談でしょう?名無しさん:2009/07/11(土) 14:27:35 ID:vOPp6uoi
実験物理学者の歴史がのってる本はありますか?
921ご冗談でしょう?名無しさん:2009/07/11(土) 16:04:27 ID:???
なんのために必要なのかのぅ
922ご冗談でしょう?名無しさん:2009/07/11(土) 20:00:46 ID:???
ttp://imagepot.net/image/124730970707.jpg
電子が一つの場合(s=1/2)スピン起動相互作用・弱い磁場によるゼーマン効果によって画像のように
準位が分裂すると思うのですがそれぞれの準位の量子数の組み合わせがよくわかりません。
スピン起動相互作用によって分裂した際2つの準位の量子数はどのようになるのでしょう
923ご冗談でしょう?名無しさん:2009/07/11(土) 20:33:38 ID:???
mってなんだ
924ご冗談でしょう?名無しさん:2009/07/11(土) 20:44:40 ID:???
>>922
角運動量の合成はできるの?
925ご冗談でしょう?名無しさん:2009/07/11(土) 22:41:34 ID:???
本当にくだらない質問なんですが、
2リットルのペットボトルに水をほぼ満タンにして凍らせて、
それを部屋においたら多少は涼しくなりますか?
部屋の広さは3畳ぐらいです
よろしくお願い致します。
926ご冗談でしょう?名無しさん:2009/07/11(土) 22:45:56 ID:???
>>925
部屋の中の冷蔵庫で凍らせるならば、
その際に冷蔵庫の放熱板から発生する熱の方が
多いのではないだろうか。

よそ部屋の冷蔵庫で凍らせて、自分の部屋に持ち込むなら
多少は涼しくなりそうだ。
927ご冗談でしょう?名無しさん:2009/07/11(土) 22:48:02 ID:???
ペットボトルの水を凍らせたら破裂するんじゃないか?
928ご冗談でしょう?名無しさん:2009/07/11(土) 22:48:20 ID:???
>>926
部屋の外の冷凍庫です
なるほど一応は効果あるんですね
ありがとうございます
929ご冗談でしょう?名無しさん:2009/07/11(土) 22:49:29 ID:???
体感できるほどの効果はあるかどうか
930ご冗談でしょう?名無しさん:2009/07/11(土) 22:51:50 ID:???
>>927
1割ぐらいは飲んでと破裂しないのですw

>>929
熱量としてはどれぐらいなんですかね
931ご冗談でしょう?名無しさん:2009/07/11(土) 23:11:10 ID:Qcb6GRPG
やってみりゃいいじゃん
932ご冗談でしょう?名無しさん:2009/07/11(土) 23:31:13 ID:???
雑誌のNewtonに『単極の磁石の合成に成功した』みたいな記事が載ってたんだけど、これってスゴイことじゃないの?


ググってもそのことがまったく話題になってないからすごく気になるんだが…
933ご冗談でしょう?名無しさん:2009/07/11(土) 23:32:32 ID:???
デマじゃね
934ご冗談でしょう?名無しさん:2009/07/11(土) 23:37:43 ID:???
なんだデマか
935ご冗談でしょう?名無しさん:2009/07/11(土) 23:38:28 ID:???
磁石はN極とS極があって初めて磁石として成立っています。
したがってN極のみ、S極のみといった単極磁石を作ることはできません。
http://www.26magnet.co.jp/database/qa/qa36.html
936ご冗談でしょう?名無しさん:2009/07/11(土) 23:39:31 ID:???
Newtonって専門誌風の見た目のくせに、
実は一番のトンデモ雑誌だからなw
937ご冗談でしょう?名無しさん:2009/07/11(土) 23:41:09 ID:???
>トンデモ雑誌
ソースは?
938ご冗談でしょう?名無しさん:2009/07/11(土) 23:42:18 ID:???
ソースというか、書いてあることの大半が子供や非専門家向けの方便でしょう
939ご冗談でしょう?名無しさん:2009/07/11(土) 23:43:15 ID:???
ニュートンはムーみたいなオカルト雑誌だと思っとけばいい
だいたい同じ棚にあるし
940ご冗談でしょう?名無しさん:2009/07/11(土) 23:43:19 ID:???
>>935
だからこそ発見されたらスゴイはずなのに話題になってないから不思議なんだよなぁ

>>936
そうなのか!
これから買うのやめようかな…
941ご冗談でしょう?名無しさん:2009/07/11(土) 23:44:09 ID:???
>>936
信憑性の低いニュースでも、面白記事だから載せただけさ
942ご冗談でしょう?名無しさん:2009/07/11(土) 23:45:52 ID:???
>>940
いや、まぁヨタ話のネタに読むぐらいはいいんじゃない?
絵とかCGはきれいだし
あんまり鵜呑みにすると危険だけど
943ご冗談でしょう?名無しさん:2009/07/11(土) 23:47:54 ID:???
日経サイエンスでも電磁波を重力波に変換する技術とかの速報有ったし、
ポピュラーサイエンス誌はそんなもん
944ご冗談でしょう?名無しさん:2009/07/11(土) 23:48:09 ID:???
>>942
たしかにイラスト綺麗だもんな
これからはNewtonの情報も鵜呑みにしないようにするよ
945ご冗談でしょう?名無しさん:2009/07/11(土) 23:51:06 ID:???
新聞の科学欄やテレビの科学番組も怪しいしね
専門家と記者が伝言ゲームになっちゃってるのが一番の原因なんだけどw
946ご冗談でしょう?名無しさん:2009/07/11(土) 23:51:21 ID:???
>>925
2kgの-10℃の氷と30℃の水のエネルギー差分はおよそ1MJ。
1000秒くらいで融かせば1kwくらいの冷却能力になるな。
947ご冗談でしょう?名無しさん:2009/07/11(土) 23:53:02 ID:???
融解熱を入れると結構エネルギーあるんだな
948ご冗談でしょう?名無しさん:2009/07/11(土) 23:57:44 ID:???
>>938
トンデモと方便はまったくの別物だろ
949ご冗談でしょう?名無しさん:2009/07/12(日) 00:05:34 ID:???
どっちでもいいよ
950ご冗談でしょう?名無しさん:2009/07/12(日) 00:07:58 ID:???
Newtonはときどき数式の間違いもあるし、解釈に矛盾がある場合もあるし。
でも難解な専門分野を平易に素人に分かりやすく解説してくれるのも事実。
これに限らず巷の情報を鵜呑みにする癖がなければOK。
951ご冗談でしょう?名無しさん:2009/07/12(日) 00:14:34 ID:???
同意
952ご冗談でしょう?名無しさん:2009/07/12(日) 00:17:20 ID:???
トンデモってwwww
953ご冗談でしょう?名無しさん:2009/07/12(日) 00:31:40 ID:???
水を沸騰させたときに、沸騰してる間に温度は上がりませんが、
その熱はどこに行ってしまったのですか?
954ご冗談でしょう?名無しさん:2009/07/12(日) 00:33:58 ID:???
蒸気が奪って行ってるんだよ
955ご冗談でしょう?名無しさん:2009/07/12(日) 00:36:14 ID:???
>>954
ってことは、蒸気を閉じ込めれば100℃以上になるってことですか?
956ご冗談でしょう?名無しさん:2009/07/12(日) 00:40:27 ID:???
当たり前じゃん
957ご冗談でしょう?名無しさん:2009/07/12(日) 00:41:50 ID:???
>>956
ってことは、氷を溶かして水になるときみたいに、
横軸を時間、縦軸を温度で取ったとき、
温度が一定になる時間はないってことですかね?
それとも、水が沸騰しきるまでは温度は一定ですか?
958ご冗談でしょう?名無しさん:2009/07/12(日) 00:43:09 ID:???
密封すれば順次圧が高まるからそれだけ沸点も上昇するだろ
959ご冗談でしょう?名無しさん:2009/07/12(日) 00:46:22 ID:???
高田純次 にみえた
960ご冗談でしょう?名無しさん:2009/07/12(日) 00:50:24 ID:???
>>950
>Newton
トンデモってか、ドカンチ大量製造雑誌ですな。
961ご冗談でしょう?名無しさん:2009/07/12(日) 01:25:47 ID:???
>>960
ではあなたが、数式や専門用語を一切使わずに素人に説明してください。
数学や物理が分からなくても科学に興味のある人はたくさんいます。
962ご冗談でしょう?名無しさん:2009/07/12(日) 01:30:32 ID:???
なんか一人むきになってるのがいるな
963ご冗談でしょう?名無しさん:2009/07/12(日) 01:44:18 ID:???
>>961
あなたがドカンチか
964ご冗談でしょう?名無しさん:2009/07/12(日) 08:20:41 ID:???
>>925
凍らせたペットボトルを床に置いておいてもあまり
有り難みはなさそう。冷たい空気が足下をはうだけ。

ひもをつけて天井からつり下げたら、頭上から冷たい
空気が降りてきて気持ちいいんじゃなかろうか。
965ご冗談でしょう?名無しさん:2009/07/12(日) 08:28:26 ID:???
>>964
PETボトルに風当てりゃいいだろ。
扇風機なんか強風にしたって50w程度しか喰わないから。

結露するから、下に受け皿置いておくがよろし。
966ご冗談でしょう?名無しさん:2009/07/12(日) 09:05:44 ID:???
>>961
何でこういう人って、
なんでもかんでも一言ぐらいで、
数式を用いずに簡単に説明できると思えるのかな。
967ご冗談でしょう?名無しさん:2009/07/12(日) 09:15:04 ID:???
>>966 読解力不足だな。

Newtonをこき下ろしてることに対しての反論だろ。
968ご冗談でしょう?名無しさん:2009/07/12(日) 09:25:11 ID:???
まぁ、小学生や出来の悪い中学生や妄想世界の大人が絵本に興味を示すのは理解できなくはないが。
しかし、理系の人間がお宝にして読んでシッタカしている姿は、いと哀れだね。
969ご冗談でしょう?名無しさん:2009/07/12(日) 09:32:44 ID:???
>>967
君の方が読解力不足だろ。
だからNewtonはダメだと言っているんだ。
無駄な誤解は弊害でしかない。
970ご冗談でしょう?名無しさん:2009/07/12(日) 09:38:16 ID:???
>>932
モノポール(磁気単極子)という仮想素粒子はあるけど。
まだ、発見されてないんじゃなかった?
971ご冗談でしょう?名無しさん:2009/07/12(日) 10:00:28 ID:???
仮説上の粒子と言ってくれ
972ご冗談でしょう?名無しさん:2009/07/12(日) 10:08:59 ID:???
そうだな。まだあると決まった訳じゃないね。
973ご冗談でしょう?名無しさん:2009/07/12(日) 10:12:30 ID:???
>>970
モノポールと違って化合物らしい
974ご冗談でしょう?名無しさん:2009/07/12(日) 10:16:30 ID:???
少なくとも大学生以上がニュートンの記事を信用してるようじゃだめだな
あれは批判的に読まないと
975ご冗談でしょう?名無しさん:2009/07/12(日) 11:04:17 ID:???

モノポール(lol)
976ご冗談でしょう?名無しさん:2009/07/12(日) 13:10:18 ID:???
Newtonで方便を超えた単なる誤りって、例えばどんなのがあるの?
977ご冗談でしょう?名無しさん:2009/07/12(日) 14:43:58 ID:???
Newtonは素人におおまかな雰囲気を伝えるものなんじゃないの?
978ご冗談でしょう?名無しさん:2009/07/12(日) 15:01:32 ID:???
Newton絶賛もNewton全否定も滑稽。
979ご冗談でしょう?名無しさん:2009/07/12(日) 15:04:07 ID:???
なにがよくないのか言えなくてただ駄目だっていうだけなったらただの
Newtonアンチとしか言いようがないんじゃないの?
980ご冗談でしょう?名無しさん:2009/07/12(日) 15:30:07 ID:???
Newtonはよく頑張ってるなあというのが普通の印象だと思うが

磁石の合成に成功ってそんなに凄いことなのか?
強磁性体なら種類の違う元素からの合成は普通にできるよな
どのレベルでの合成に成功したと言ってるんだろう
981ご冗談でしょう?名無しさん:2009/07/12(日) 15:41:48 ID:???
> 磁石の合成に成功ってそんなに凄いことなのか?

「磁石の合成」なら全然すごくないだろう…君の目は節穴か?
982ご冗談でしょう?名無しさん:2009/07/12(日) 15:46:04 ID:???
>>981
さんくす。思いっきり見逃してたわ
983ご冗談でしょう?名無しさん:2009/07/12(日) 16:11:52 ID:???
おいおい、単極磁石の合成が簡単だってか?
984ご冗談でしょう?名無しさん:2009/07/12(日) 16:43:11 ID:???
だから…
980は「単極」を読み落としてました、ってだけの話
985ご冗談でしょう?名無しさん:2009/07/12(日) 16:46:06 ID:???
>>983
お前は日本語が読めないのか?
986ご冗談でしょう?名無しさん:2009/07/12(日) 18:10:51 ID:???
>>985
わるい わるい(^^;;
下のレスを読まずに書き込んでしまった。
987ご冗談でしょう?名無しさん:2009/07/12(日) 18:15:04 ID:???
つか記事読んだ訳じゃないけど>>973によるとモノポール発見じゃないんでしょ。
モノポール発見なら大ニュースになるだろうけど。
でも化合物だとしてもどういう仕組みになってるんだろね
球殻状の物質でN極が外側向いててS極が内側向いてるとかならどうよ
988ご冗談でしょう?名無しさん:2009/07/12(日) 18:19:07 ID:Kfgb6vhY
>>987
棒磁石をそういう風に並べたらs-sの反発で飛び散るな
989ご冗談でしょう?名無しさん:2009/07/12(日) 18:23:01 ID:???
>>987  全立体角で積分したら磁束ゼロだろJK
990ご冗談でしょう?名無しさん:2009/07/12(日) 18:27:40 ID:???
ちょっと落ち着いて考えて欲しい
>>932の勘違いということもあるのではないだろうか
991ご冗談でしょう?名無しさん:2009/07/12(日) 18:30:25 ID:???
どうも「単分子磁石の合成に成功」の勘違いではなかろうかと。
992ご冗談でしょう?名無しさん:2009/07/12(日) 18:33:26 ID:???
>>988
サッカーボール状の恐ろしく強い構造とか
993ご冗談でしょう?名無しさん:2009/07/12(日) 18:33:44 ID:???
>>990
それは俺も思ってる。読み間違えたのではないかと…
今手元にないから全部さらしたりはできないから、どうしても気になるなら本屋にでも行ってくれ

>>987
正四面体の結晶で、ある結晶から磁力線の放出のみが、ある結晶からは磁力線の吸収のみが観測された


うろ覚えだけどこんな感じだった
994ご冗談でしょう?名無しさん:2009/07/12(日) 18:36:31 ID:???
>>993
難しくてわかんなぃorz
995ご冗談でしょう?名無しさん:2009/07/12(日) 18:44:45 ID:???
>>994
同じく…



ちなみに、自分でいうのもなんだけど『単極』って単語は本文に出て来たから使ってるわけだし、そのうえ何回も読み返したから間違えてる可能性は低いと思う
996ご冗談でしょう?名無しさん:2009/07/12(日) 18:45:22 ID:Yx9iz+IM
流れと全く関係ありませんが、

円管(層流)における局所混合平均温度を
グレツ数の関数で表わす式の導出が載っている本やサイト,もしくはやり方を知っていたら教えて頂けないでしょうか?
エネルギ方程式から算出しようとしたのですがうまく出来ませんでした。
997ご冗談でしょう?名無しさん:2009/07/12(日) 18:47:36 ID:???
流れと関係あるじゃねーかよ
998ご冗談でしょう?名無しさん:2009/07/12(日) 18:53:53 ID:???
山田君モノポール座布団>>997に1枚あげて!
999ご冗談でしょう?名無しさん:2009/07/12(日) 18:54:36 ID:???
お!まじモノポール発見!
http://www.jst.go.jp/pr/report/report360/
1000ご冗談でしょう?名無しさん:2009/07/12(日) 18:57:45 ID:???
山田君>>999の座布団全部持ってって!
10011001
このスレッドは1000を超えました。
もう書けないので、新しいスレッドを立ててくださいです。。。